Download as pdf or txt
Download as pdf or txt
You are on page 1of 192

CHARTERED ACCOUNTANCY PROFESSIONAL II

(CAP-II)

Revision Test Paper


June 2020

Education Department
The Institute of Chartered Accountants of Nepal

The Revision Test Papers are prepared by the institute with a view to assist the students in their study.
The suggested answers given here are indicative and not exhaustive. Students are expected to apply their
knowledge and write the answer in the examinations taking the suggested answers as guide. Due care has
been taken to prepare the revision test paper. In case students need any clarification, creative feedbacks or
suggestions for the further improvement on the material, or any error or omission on the material, they
may report to the email educationdepartment@ican.org.np at Education Department of the Institute.
Paper 1 : Advanced Accounting

Paper 1: Advanced Accounting

© The Institute of Chartered Accountants of Nepal 1


Paper 1 : Advanced Accounting

Revision Questions:
Nepal Financial Reporting Standards (NFRS)
Question No 1.
On 1 Shrawan 2075, SK Co. Ltd. began to construct a supermarket which had an estimated
useful life of 40 Years. It purchased a leasehold interest in the site for Rs 25 million. The
construction of the building cost Rs 9 million and the fixtures and fittings cost Rs 6 million.
The construction of the supermarket was completed on 31 Asadh 2076 and it was brought
into use on 1 Shrawan 2076.
SK Co. Ltd. borrowed Rs 40 million on 1 Shrawan 2075 in order to finance this project. The
loan Carried interest @ 10% p.a. It was repaid on 30 Aswin 2076
Required: Calculate the total amount to be included at cost in property, plant and equipment
in respect of the development at 31 Asadh 2076..
Question No.2
Value the following items of inventory
a) Materials costing Rs12000 bought for processing and assembly for a profitable special
order. Since buying these items, the cost price has fallen to Rs10,000.
b) Equipment constructed for a customer for an agreed price of Rs18,000. This has
recently been completed at a cost of Rs16,800. It has now been discovered that, in order
to meet certain regulations, conversion with an extra cost of Rs 4,200 will be required.
The customer has accepted partial responsibility and agreed to meet half the extra cost.

Question No. 3
V. Ltd sold 1 lakh vacuum pumps during the year 2075-76 with a condition to make good by
repair/replacement any manufacturing defects reported within 6 months from the date of sale.
Past experience in this regard showed that there were no replacements carried out, but
minor/major repairs were necessitated to the extent of 10%/5% respectively of the units sold.
The cost of such minor/major would amount to Rs 1,000/Rs 6,000 respectively. While
finalizing the accounts for the year, the company does not reflect provision in this regard.
Comment as per Nepal Accounting Standard.

Question No 4
Mini Ltd took a factory premises on lease on 1.4.2073 for Rs 200,000 per month. The lease is
operating lease. During Asadh,2074, Mini Ltd relocates its operation to a new factory building.
The lease on the old factory premises continues to be live up to 30.12. 2076.The lease cannot
be cancelled and cannot be sub-let to another user. The management insists that lease rent of
balance 33months up to 30.12.2076 should be provided in the accounts for the year ending
31.3. 2074.Mini Ltd seeks your advice.

© The Institute of Chartered Accountants of Nepal 2


Paper 1 : Advanced Accounting

Question No 5
At the end of the financial year ending on 31st Asasdh,2075, a company finds that there are
twenty law suits outstanding which have not been settled till the date of approval of accounts
by the Board of Directors. The possible outcome as estimated by the board is as follows:
Probability
Loss(Rs)
In respect of five cases(win) 100% -
Next ten Cases(Win) 60% -
Lose (low Damages) 30% 120,000
Lose (High Damages) 10% 200,000
Remaining five cases
Win 50% -
Lose (low Damages) 30% 100,000
Lose (High Damages) 20% 210,000
Outcome of each case is to be taken as a separate entity ascertain in the amount of contingent
loss and the accounting treatment in respect thereof

Question No 6
During 2071 a company discovered that certain items had been included in inventory at 31
Asadh 2070 at a value of Rs 2.5 million but they had in fact been sold before the year end.
The original figures reported for the year ending 31 Asadh 2070 and the figures for the current
year 2071 are given below:

2071 2070
Particulars Rs in Thousands Rs in Thousands
Sales 52,100 48,300
Cost of Sales (33,500) (30,200)
Gross Profit 18,600 18,100
Tax (4,600) (4,300)
Net Profit 14,000 13,800
The cost of goods sold in 2071 includes the Rs 2.5 million error in opening inventory. The
retained earnings at 1 Shrawan 2070 were Rs 11.2million.(Assume that the adjustment will
have no effect on the tax charges)
Show the 2071 statement of profit or loss with comparative figures and the retained earnings
for each year. Disclosure of other comprehensive income is not required.

© The Institute of Chartered Accountants of Nepal 3


Paper 1 : Advanced Accounting

Hire Purchase Accounting


Question No 7
A Ltd purchases a plant on hire purchase basis for Rs 100,000 and makes the payment in the
following order:
Down payment Rs 20,000
1st installment after one year 40,000
2nd installment after two year Rs 20,000
and the last installment after 3 years
The cash price of the plant is Rs 86,000
You are required to calculate (i) the total interest (ii) the interest included in each installment.
Question No 8
Arghakhachi Traders sells goods on hire purchase basis on the following terms:
Cost Rs 2,000 Selling Price Rs 3,000 Deposit Payable 20% Monthly Installment Payable 12
Monthly installment are payable on the last date of the month in which the sales takes place.
Other details are:
2071 2072
Sales 300Nos 600Nos
Installment Received Rs.375,000 Rs.10,70,000
Sales take place evenly throughout the year. In 2072, 40 items sold in 2068 on which a total
of 300 installments had been received were repossessed.
Required: Show ledger accounts for both the years in the books of Arghakhachi Traders.
Accounting for Amalgamation, Absorption and Corporate Restructuring
Question No 9
Small Limited and Little Limited, two Companies in the field of specialty chemicals, decided
to go in for a follow on public offer after completion of an amalgamation of their businesses.
As per agreed terms initially a new company Big Ltd will be incorporated on 1st Shrawan, 2075
with an authorized capital of Rs 2 crore comprised of 20lakh equity shares of Rs 10 each. The
holding company would acquire the entire shareholding of Small ltd & Little Ltd and in turn
would issue its share to the outside holders of these shares. It is also agreed that the
consideration would be a multiple of the average P/E ratio for the period 1st Shrawan,2014 to
31st Aswin,2074 times the rectified profits of each company, subject to necessary adjustments
for complying with the terms of the share issue.
The following information is supplied to you:

Particulars Small Ltd. Little Ltd.


Ordinary Shares of Rs. 10each(Nos.) 40lakhs 20Lakhs
10% Preference Shares of Rs 100
each(Nos.) 2 Lakh Nil

© The Institute of Chartered Accountants of Nepal 4


Paper 1 : Advanced Accounting

10% Preference Shares of Rs 10


each(Nos.) Nil 2 Lakhs
5% debentures of Rs 10 each (Nos.) 4Lakhs 4Lakhs
Investments Held
(a) 4 Lakh ordinary shares in small ltd - Rs 40 Lakhs
(b) 2 Lakh ordinary shares in little ltd Rs 20 Lakhs -
Profit before interest and tax(PBIT)
after
considering impact of Inter-Company
Transactions and Holdings. Rs 50 Lakhs Rs 25 Lakhs
Average P/E ratio Shrawan 2074-
Aswin 2074 10 8

The following additional information is also furnished to you in respect of adjustments required
to the profit figure as given above:
1. The profits of the respective companies would be adjusted for the half the value of
contingent liabilities as on 31st Asadh 2074.
2. Trade receivables of small Ltd include an irrecoverable amount of Rs 2 lakh against
which Rs 1 lakh was recovered but kept in advance account.
3. Little Ltd had omitted to provide for increased FOREX liability of US$ 10,000 on loan
availed in financial year 2073-74 for purchase of Machinery. The Machinery was
acquired on 1st Baisakh,2074 and put to use in financial year 2074-75. The additional
liability arose due to change in exchange rates and is arrived at in conformity with
prevailing provisions of NAS 21. The exchange rate is US $ 1=NRS 50.
4. Small Ltd. has omitted to invoice a sale that took place on 31st Asadh ,2074 of goods
costing Rs 250,000 at a markup of 15 percent instead the goods were considered as
part of closing inventory.
5. Closing Inventory of Rs 45 lakhs of Little Ltd. as on 31st Asadh ,2074 stands
undervalued by 10 percent.
6. Contingent liabilities of Small Ltd and Little Ltd.as on 31st Asadh,2074 stands at Rs 5
lakhs and Rs10 lakhs respectively.
The terms of the share issue are as under:
I. Shares in Big Ltd will be issued at a premium of Rs 13 per share for all external
shareholders of Small Ltd. The premium will be Rs 15 per share for shares in Big Ltd.
issued to all external shareholders of little ltd.
II. No shares in Big Ltd. will be issued in lieu of the investments (intercompany holdings)
of both companies. Instead the shares so held shall be transferred to Big Ltd at the close
of the financial year ended 31st Asadh,2075 at par value consideration payable on date
of transfer.
III. Big Ltd would in addition to the issue of shares to outside shareholders of Small Ltd
and Little Ltd make a preferential allotment on 31st Asadh,2075 of 2 lakhs ordinary
shares at a premium of Rs 28 per share to Virgin Capital Ltd (VCL). These shares will
not be eligible for any dividends declared or paid till that date.

© The Institute of Chartered Accountants of Nepal 5


Paper 1 : Advanced Accounting

IV. Big Ltd will go in for a 18 percent unsecured Bank overdraft facility to meet
incorporation costs of Rs 16 lakhs and towards management expenses till 31st Asadh ,
2075 estimated at Rs 14 lakhs. The overdraft is expected to be availed on 1st Jestha
,2075 and closed on 31st Asadh,2075 out of the proceeds of the preferential allotment.
V. It is agreed that interim dividends will be paid on 31.03.2075 for the period
Baisakh,2015 to Asadh,2015 by Big Ltd.at 2 percent; Small Ltd at 3 percent and Little
Ltd at 2.5 percent. Ignore dividend distribution Tax.
VI. The prevailing income tax is 25 percent.
You are required to compute the number of shares to be issued to the shareholders of each of
the companies and prepare the projected Profit and Loss account for the period from 1st
Baisakh,2075 to 31.03.2075 of Big Ltd and its Balance Sheet as on 31st Asadh 2075.

Question No 10
AB Ltd and MB Ltd. decide to amalgamate and to form a new company AM Ltd. The following
are their summarized balance sheets as at 31.3.2075:

Liabilities AB Ltd MB Ltd Assets AB Ltd MB Ltd


Share Capital( Rs 100
Each) 1,000,000 600,000 Fixed assets 750,000 200,000

General Reserve 100,000 50,000 Investments:

1500 shares in MB 350,000 -


Investment Allowance
Reserve 40,000 30,000 4000 Shares in AB - 500,000
12%
Debentures(Rs.100each) 300,000 100,000

Trade Payables 60,000 20,000 Current assets 400,000 100,000

1,500,000 800,000 1,500,000 800,000


Calculate the amount of purchase consideration for AB Ltd and MB Ltd and draw up the
balance sheet of Am Ltd after considering the following:
a. Assume amalgamation is in the nature of the purchase.
b. Fixed assets of AB Ltd are to be reduced by Rs 50000 and that of MB Ltd are to be
taken at Rs 300,000.
c. 12% debenture holders of AB Ltd and MB Ltd are discharged by AM Ltd By issuing
such number of its 15% debentures of Rs 100 each so as to maintain the same amount
of interest.
d. Shares of AM Ltd are of Rs 100 each.

Also show, how the investment allowance reserve will be treated in the financial statement
assuming the reserve will be maintained for 3 years.

© The Institute of Chartered Accountants of Nepal 6


Paper 1 : Advanced Accounting

Question No 11
The following are the summarized balance sheet of X. Ltd and Y. Ltd as on 31st Asadh, 2075
X. Ltd Rs. Y. Ltd Rs.
Assets

Fixed assets 700,000 250,000

Inventory 240,000 320,000

Trade Receivables 420,000 210,000

Cash At Bank 110,000 40,000


Investments in:

6000 shares of Y .Ltd 80,000

5000 shares of X. Ltd. 80,000

1,550,000 900,000
Liabilities
Share Capital:

Equity Shares of Rs. 10 each 600,000 300,000

10% preference shares of Rs. 10 each 200,000 100,000

Reserve And Surplus 300,000 200,000

12% debentures 200,000 150,000

Trade Payables 250,000 150,000

1,550,000 900,000

Details of Trade payables and Trade receivables:

X.Ltd Rs. Y.Ltd Rs.


Trade Payables

Bills payable 30,000 25,000

Sundry Creditors 220,000 125,000

250,000 150,000
Trade receivables

Debtors 360,000 190,000

© The Institute of Chartered Accountants of Nepal 7


Paper 1 : Advanced Accounting

Bills Receivables 60,000 20,000

420,000 210,000

Fixed assets of both the companies are to be revalued at 15% above book values and inventory
and debtors are to be taken over at 5% less than their book values. Both the companies are to
pay 10% equity dividends, preference dividends having been paid already.
After the above transactions are given effect to X Ltd will absorb Y.Ltd on the following terms:
I. 8 equity shares of Rs. 10 each will be issued by X Ltd at par against 6 shares of Y.Ltd.
II. 10% preference shares of Y.Ltd will be paid off at 10% discount by issue of 10%
preference shares of Rs. 100 each of X Ltd at par.
III. 12% Debenture holders of Y ltd are to be paid off at a 8% premium by 12%
debentures in X Ltd. issued at a discount of 10%.
IV. Rs. 30,000 to be paid by X.Ltd to Y.Ltd for liquidation expenses.
V. Creditors of Y Ltd include Rs. 10,000 due to X Ltd.
Prepare: (a) A statement of purchase consideration payable by X.Ltd
(b) A Balance Sheet of X Ltd after its absorption of Y.Ltd.

Branch Account
Question No 12
Subash Electricals has its branches at Chitwan and Palpa to whom goods are invoiced at cost
plus 25%. Following information is available of the transactions at Chitwan Branch for the year
ending 31st Asadh, 2072:
Balances As at 1.4.2071 As at 31.3.2072
Stock at invoice price Rs 40,000 ?
Debtors Rs 12,000 Rs 11,000
Petty Cash 150 250

Transactions during 2071-2072: Rs.


Goods sent to branch 4, 20,000
Goods returned to head office 15,000
Cash Sales 1, 05,000
Credit Sales 1, 80,000
Normal Loss(at cost) 280
Goods pilfered (at invoice price) 3,000

© The Institute of Chartered Accountants of Nepal 8


Paper 1 : Advanced Accounting

Goods lost in fire(at invoice price) 4,000


Insurance company paid to Head Office for loss by fire at Chitwan 3,000
Cash sent for petty expenses 32,000
Bad debts At Chitwan Branch 400
Goods Transferred to Palpa Branch under instructions From Head Office at
Invoice price 12,000
Insurance Charge Paid by Head Office 200
Goods returned by debtors 500
Note: Goods transferred to Palpa Branch(given above) were in transit on 31st Asadh,2072.
Prepare (i) Branch Stock Account (ii) Branch Adjustment Account (iii) Branch Profit and loss
account (iv) Stock Reserve Account
Question No 13
Sriniwash Ltd having head office in Kathmandu has branch at Mahendranagar where a
complete set of books is maintained. All purchases are made at Kathmandu and stock required
by the branch is invoiced to it by the head office at selling price less 15 percent. The branch
manager is entitled to a commission equal to one-third of the net profit earned by the branch
on the basis of invoice price. The following are the respective Trial balances of the head office
and the branch as on 31st Asadh, 2072.

Head Office Branch


Particulars Dr. Rs Cr. Rs Dr. Rs Cr.Rs
Capital 94,200
Sundry Debtors & Sundry Creditors 89,700 24,600 7,550 6,150
Stock at beginning of year at cost:
Head Office 124,150
Branch 32,150
Stock at Branch beginning of year as
invoiced 36,270 36,270
Purchases 307,250
Goods Invoiced during the year 102,500 102,500
Salaries 35,280 14,680
Sundry Expenses 16,170 2,290
Branch Account 23,230
Head Office Account 5,800
Cash on hand and at bank 24,800 3,260
Furniture 15,000 2,000
Sales 410,160 168,200
667,730 667,730 174,350 174,350

© The Institute of Chartered Accountants of Nepal 9


Paper 1 : Advanced Accounting

The cost Price of goods invoiced to Mahendranagar was Rs 81,900.The stock in hand at the
end of the year at Kathmandu was valued at Rs 45,250 at cost and at Mahendranagar Rs.14,160
at cost and Rs.17,660 at invoice price. The difference between the adjustment account is due
to a remittance in transit from the Branch to the Head Office.
Prepare the trading and profit and loss account for the year ending on 31st Asadh,2072(in
columnar form) and the consolidated Balance Sheet as at that date.
Partnership Account
Question No 14
A,B and K were carrying business as oil dealers and they share profits seven-fifteenth, five-
fifteenth and three-fifteenth respectively. On 31st Asadh,2071 they dissolve partnership. The
Balance Sheet of the firm on that date was as follows:

Liabilities Rs. Assets Rs.

Sundry Creditors 12,500 Land & Building 37,500

Loan from A at 5 percent 30,000 Plant 11,250

Capitals: Furniture 5,000

A 45,000 Stock 53,250

B 30,000 Sundry Debtors 15,000

K 12,500 Cash at Bank 8,000

130,000 130,000
A and B wanted K to join them in floating the concern as a Private Limited Company but K
refused and A and B arranged to pay K out and then form the company. The arrangements
between A,B and K were as follows:
A and B took over the liabilities at book figure plus Rs.2,500 allowed for realization expenses;
they also took over the Cash at Bank and the Furniture was sold for Cash realizing Rs 3,700.The
other assets A and B agreed to take over as follows:
Land and Building Rs.50,000; Plant less 10 percent; Stock less 20 percent; Debtors at
Rs.10,575
A and B having paid the realization expenses Rs.2,500 and having paid out Karun by providing
the required cash in the same proportion as they share the profits and losses interse proceed to
sell the assets as they stand to COJIG & Company Private Limited for Rs.60,000 cash and
Rs.60,000 in fully paid shares. The latter A and B take in the same proportions as they share
profits and losses.
A and B this pay off the liabilities and close the partnership books.
Required: Show (a) the Bank Account; (b) the Capital Accounts; (c) the Realisation Accounts;
and (d) COJIG & Company Private Limited Accounts.

© The Institute of Chartered Accountants of Nepal 10


Paper 1 : Advanced Accounting

Assume that all the above transactions are put through on Asadh 31,2071(Calculations to be
made to the nearest Rupee.)
Accounting for Debentures & Preference Shares
Question No 15
The summarized Balance Sheet of X Ltd, as on 31.03.2076 stood as follows:
Liabilities Rs. Assets Rs
500000 equity shares of Rs 10 each Fixed Assets(at cost less
fully paid 50,00,000 depreciation) 1,60,00,000
Debenture Redemption Fund
General Reserve 75,00,000 Investment 40,00,000
Debenture Redemption Fund 50,00,000 Cash & Bank Balances 50,00,000
100000,13.5% convertible debentures
2,00,00,000
of 1,00,00,000 Other current Assets
Rs 100 each
Other Loans 50,00,000
Current liabilities & Provisions 1,25,00,000
4,50,00,000 4,50,00,000

The debentures are due for redemption on 1.04.2077.The terms of issue of debentures provided
that they were redeemable at a premium of 5% and also conferred option to the debenture
holders to convert 20% of their holding into equity shares
Assuming That:
I. Except for 100 debenture holders holding 25000 debentures, the rest opted for
maximum conversion;
II. The investments realized Rs 44,00,000 on sale; and
III. All the transactions are put through, without any lag, on 1.04.2077
Redraft the Balance Sheet of the company as on 1.04.2077, after giving effect to the
redemption. Show your calculations in respect of the number of equity shares to be
allotted and the cash payment necessary.

Question No 16
On 1st July 2015, H.P Ltd issued 2000,6% debentures of R.s 100 each. The interest is payable
on 30th June and 31st December every Year. The company is allowed to purchase its own
debentures which may be cancelled or kept or re-issued at the company’s option. The company
made the following purchases by cheque in the open market.
On 31st May, 2016 200 debentures @ Rs. 98 ex-interest. On 30th September 2017,
100debentures @ R.s 97 cum interest. The debentures, which were purchased on 31st May
2016, were cancelled on 31st December 2017.All payments were made on due dates.
Pass necessary journal entries to record the above transactions (including receipts and
payments) and also show the relevant items in the Balance Sheet as on 31st December, 2017.

© The Institute of Chartered Accountants of Nepal 11


Paper 1 : Advanced Accounting

HP Ltd followed English calendars for accounting purpose and closes its book account on 31st
December every year.

Question No 17
Explain about Financial Leverage Multiplier (FLM).
Question No 18
State the difference between Government Accounting and Business Accounting.
Question No 19
Write short notes on:
• Supplementary capital as per capital adequacy norms of banks and financial
institutions.
• Distribution of Management expense in Non-life Business Insurer.
• Annuity method for calculating Goodwill.
• Steps of Acquisition for Business Combination (NFRS 03)

Question No 20
Explain about Capitalization of Dismantling Cost as per NAS 16 “Property, Plant &
Equipment”.

© The Institute of Chartered Accountants of Nepal 12


Paper 1 : Advanced Accounting

Answers/Hints:
Nepal Financial Reporting Standards (NFRS)
Answer 1.
Total amount to be included in the property, plant and equipment at 31 Asadh 2076:
Particulars Rs Million
Lease 25,000
Buildings 9,000
Fittings 6,000
Interest Capitalized (40,000*10%*9/12) 3,000
43,000

Only nine months’ interest can be capitalized, because NAS 23 states that capitalization of
borrowing costs must cease when substantially all the activities necessary to prepare the asset
for its intended use or sale are complete.
Answer 2
As per NAS 02 “Inventories”
a) Value at Rs.12,000.Rs.10,000 is irrelevant. The rule is lower of cost or NRV, not
lower of cost or replacement cost. Since the special order is known to be profitable,
the NRV will be above the cost.
b) Value at NRV,i.e Rs.15,900,as this is below cost
(NRV=contract price,Rs.18,000-company’s share of modification cost i.e.Rs.2100)

Answer 3
This problem is based on NAS-37.The standard provides that an enterprise should recognize a
provision only when all of the following conditions are met:
1. There is a present obligation as a result of a past event;
2. It is probable that an outflow of resources embodying economic benefits will be
required to settle the obligation; and
3. A reliable estimate can be made of the amount of obligation.
In the present case, V Ltd fulfils all the above conditions the sale of pumps with a warranty
obligation constitutes the present obligation as a result of the past event. It is probable that
some outflow will be involved in setting the warranty obligation, satisfy the second condition.
As per the details based on past precedence reliable estimate can be made as under:
[6000*(5%of 100000)+1000*(10%of 100000)=Rs.400lakhs
Thus, V Ltd as on 31-03-2076 should make a provision for warranty obligation against sale of
vacuum pumps to the extent of rupees 400 lakhs.

© The Institute of Chartered Accountants of Nepal 1


Paper 1 : Advanced Accounting

Answer 4
The Problem is based on NAS 37. The Lease in question is giving rise to an onerous contract.
An ‘onerous contract’ is a contract in which the unavoidable costs of meeting the obligations
under the contract exceed the economic benefits expected to be received under it. An issue that
arises is as to how the recognition and measurement principles of NAS-37 should be applied
to the onerous contracts. In this connection, the standard provides that if an enterprise has a
contract that is onerous the present obligation under the contract should be recognized and
measured as a provision as per NAS-37.
For a contract to qualify as an onerous contract, the unavoidable costs of meeting the obligation
under the contract should exceed the economic benefits expected to be received under it. The
unavoidable costs under a contract reflect the least net cost of exiting from the contract, which
is the lower of the cost of fulfilling it and any compensation or penalties arising from failure to
fulfill it.
The amount of provision in respect of an onerous contract should be measured by applying the
principles laid down in NAS-37.
In view of the above, one can clearly say that lease is an onerous contract and hence, provision
need to be made for rent of balance 33 months upto 30.12.2076. Thus the management view
point is correct.

Answer 5
According to NAS-37 “Provisions, contingent liabilities and contingent assets”, contingent
liability should be disclosed in the financial statements if following conditions are satisfied:
1. There is a present obligation arising out of past events but not recognized as provision.
2. It is not probable that an outflow of resources embodying economic benefits will be
required to settle the obligation.
3. The possibility of an outflow of resources embodying economic benefits is also remote.
4. The amount of the obligation cannot be measured with sufficient reliability to be
recognized as provision.
In this case, the probability of winning of first five cases is 100% and hence, question of
providing for contingent loss does not arise. The probability of winning of next ten cases is
60% and for remaining five cases is 50%. As per NAS-37, we make a provision if the loss is
probable. As the loss does not appear to be probable and the possibility of an outflow of
resources embodying economic benefits is not remote rather there is reasonable possibility of
loss, therefore disclosure by way of note should be made. For the purpose of the disclosure of
contingent liability by the way of note, amount may be calculated as under:
Expected loss in next ten cases = 30% of Rs.120000+10% of Rs.200000
=Rs.36,000+Rs.20,000
=Rs.56,000
Expected loss in remaining five cases= 30% of Rs.100000+20% of Rs.210000

© The Institute of Chartered Accountants of Nepal 2


Paper 1 : Advanced Accounting

=Rs.30,000+Rs.42,000
=Rs.72,000
To disclose contingent liability on the basis of maximum loss will be highly unrealistic.
Therefore, the better approach will be disclosing the overall expected loss of
Rs.920,000(Rs.56000*10+Rs.72000*5) as contingent liability.

Answer 6
Statement of Profit or Loss

2071 2070
PARTICULARS Rs. in Thousands Rs. in Thousands
Sales 52,100 48,300
Cost of Sales:
2071(33500-2500) (31,000)
2072(30200+2500) (32,700)
Gross Profit 21,100 15,600
Tax (4,600) (4,300)
Net Profit 16,500 11,300

Retained Earnings
2071 2070
Opening Retained Earnings Rs. in Thousands Rs. in Thousands
As Previously reported
Rs.(11200+13800) 25,000 11,200
Prior Period Adjustment (2,500)
As restated 22,500 11,200
Net Profit for the Year 16,500 11,300
Closing Retained Earning 39,000 22,500

Hire Purchase Accounting


Answer 7
Here,
Total Cash price = Rs. 86,000
Hire Purchase Price = Rs 100,000
Total Interest = Rs 14,000
Total interest shall be allocated as
Total Price = 100,000
Less: Down payment = 20,000
Outstanding Balance = 80,000
Less: First installment = 40,000
Outstanding Balance = 40,000
Less: Second installment = 20,000

© The Institute of Chartered Accountants of Nepal 3


Paper 1 : Advanced Accounting

Outstanding Balance = 20,000


Less: Last installment = 20,000
Outstanding Balance Nil
Now, total interest shall be allocated on the ratio of o/s balance i.e. 80,000:40,000:20,000
=4:2:1
Therefore, Interest included in first installment =4/7*14,000 =Rs.8,000
Interest included in second installment =2/7*14,000 =Rs.4,000
Interest included in first installment =1/7*14,000 =Rs.2,000
Answer 8

Dr. Hire Purchase Stock Account Cr.


Date Particulars Rs. Date Particulars Rs.
To Goods sold on By Hire Purchase
2071 Hire Purchase A/c 900,000 2071 Debtors 570,000

By Balance c/d 330,000

900,000 900,000
By Hire Purchase
2072 To Balance b/d 330,000 2072 Debtors A/c 1,470,000
To Goods sold on
Hire Purchase 1,800,000

By Balance c/d 660,000

2,130,000 2,130,000

Dr. Hire Purchase Debtors Account Cr.


Date Particulars Rs. Date Particulars Rs.
To Hire Purchase Stock By Cash(Down
2071 A/c 570,000 2071 Payment) 180,000

By Cash(Installment) 375,000

By Balance c/d 15,000

570,000 570,000
By Cash(Down
2072 To Balance b/d 15,000 2072 Payment) 360,000
To Hire Purchase Stock
A/c 1,470,000 By Cash(Installment) 1,070,000
By Goods Repossessed
A/c 36,000
By Balance c/d 19,000

1,485,000 1,485,000

© The Institute of Chartered Accountants of Nepal 4


Paper 1 : Advanced Accounting

Dr. Hire Purchase Adjustment Account(For 2071) Cr.


Particulars Rs. Particulars Rs.
By Goods sold on Hire
To Stock Reserve A/c 110,000 Purchase A/c 300,000
(Loading on Closing Stock ) (Loading on Goods Sold )

To Gross Profit t/f to P/L A/c 190,000

300,000 300,000

Dr. Hire Purchase Adjustment Account(For 2072) Cr.


Particulars Rs. Particulars Rs.
By Goods sold on Hire
To Stock Reserve A/c 220,000 Purchase A/c 600,000
(Loading on Goods Sold on
(Load on Closing Stock ) HP )
To Goods Repossessed
A/c 12,000

By Stock Reserve A/c 110,000

To Gross Profit t/f P/L A/c 478,000 (Loading on Opening Stock )

710,000 710,000

Dr. Goods Repossessed Account Cr.


Particulars Rs. Particulars Rs.
To Hire Purchase
To Hire Purchase Debtors 36,000 Adjustments(Loading) 12,000

By Balance C/d 24,000

36,000 36,000

Working Notes:
I. Calculation of H.P price, Cost Price and Loading on goods sold on H.P.
2071
2072
H.P Price Rs.9,00,000
Rs.18,00,000

© The Institute of Chartered Accountants of Nepal 5


Paper 1 : Advanced Accounting

Cost Price Rs.6,00,000


Rs.12,00,000
Loading Rs.3,00,000
Rs.6,00,000
II. Calculation of total installment due in 2071 & 2072.

Installment due per unit


during Total Installments due during
Units
Month sold 2071 2072 2071 2072
Shrawan 25 12 0 300 0
Bhadra 25 11 1 275 25
Aswin 25 10 2 250 50
Kartik 25 9 3 225 75
Mansir 25 8 4 200 100
Poush 25 7 5 175 125
Magh 25 6 6 150 150
Falgun 25 5 7 125 175
Chaitra 25 4 8 100 200
Baisakh 25 3 9 75 225
Jestha 25 2 10 50 250
Asadh 25 1 11 25 275

1,950 1,650

III. No of installments due in 2072 and in 2073 in respect of sales during 2072
Since the sale during 2072 are 200% of the sales during 2071, the no.of installments due
in 2072 and 2073 will be as under:
Due in 2072-3900, Due in 2073-3300

Accounting for Amalgamation, Absorption and Corporate Structuring


Answer 9
Computation of number of shares issued
Calculation of Rectified Profits and Purchase Considerations

Particulars Rs. Rs.


Small Ltd.
Given Profits 5,000,000
Less: Irrecoverable Trade receivables 100,000
50% Contingent Liability 250,000 (3,50,000)

© The Institute of Chartered Accountants of Nepal 6


Paper 1 : Advanced Accounting

4,650,000

Add: Profit on Omitted sale(15% of Rs.250,000) 37,500

4,687,500

Less: Debenture Interest(400,000* Rs.10*5%) (200,000)

4,487,500

Less: Income Tax @ 25% (1,121,875)

Profits after Tax(PAT) 3,365,625

Less: Preference Dividend(10%of Rs.20,000,000) (2,000,000)

Rectified Profits 1,365,625


Average P/E Ratio=10

Total Consideration for all equity shareholders 13,656,250


(Average P/E ratio*Profit)
Less:10% thereof for shareholders of little ltd.
(As little ltd holds 4lakhs out of 40 lakhs shares

of small ltd) (1,365,625)


Balance available for other Shareholders of small ltd
(A) 12,290,625
Little Ltd.

Given Profits 2,500,000


Less: Increase in FOREX liability(USRs.
10000*50) 500,000

50% Contingent Liability 500,000 1,000,000

1,500,000

Add: Undervaluation of Inventory(4500000*10/90) 500,000

2,000,000

Less: Debenture Interest(400,000* Rs.10*5%) (200,000)

1,800,000

Less: Income Tax @ 25% 450,000

© The Institute of Chartered Accountants of Nepal 7


Paper 1 : Advanced Accounting

Profits after Tax(PAT) 1,350,000

Less: Preference Dividend(10%of Rs.20,000,000) (200,000)

Rectified Profits 1,150,000


Average P/E Ratio=8
Total Consideration for all equity shareholders

(Average P/E ratio*Profit) 9,200,000


Less:10% thereof for shareholders of small ltd.
(As small ltd holds 2lakhs out of 20 lakhs shares

of small ltd) (920,000)


Balance available for other Shareholders of little ltd
(B) 8,280,000

Statement showing disposal of purchase Consideration

Small Ltd
Particulars (Rs) Little Ltd (Rs) Total (Rs)

Purchase Consideration 12,290,625 8,280,000 20,570,625


(A) above (B) above
No. of shares(Purchase
Consideration/Face

value +securities Premium) 534,375 331,200 865,575

Share Capital 5,343,750 3,312,000 8,655,750

Securities Premium 6,946,875 4,968,000 11,914,875

Purchase Consideration 12,290,625 8,280,000 20,570,625

Projected Profit And Loss Account Of Big Ltd.

For the period 1st Baisakh, 2075 to 31st Asadh,2075

Note No. Rs.


I. Revenue from operations -
II. Other Income 5 1,700,000
III. Total Revenue(I+II) 1,700,000
IV. Expenses:
Employee benefit Expense 7 1,400,000

© The Institute of Chartered Accountants of Nepal 8


Paper 1 : Advanced Accounting

Finance Costs 6 90,000


Other Expenses 8 1,600,000
V. Total Expenses 3,090,000
VI. Loss for the Period(V-III) (1,390,000)

Projected Balance Sheet Of Big Ltd.

As on 31st Asadh,2075

Particulars Note No. Rs.


I. Equity and Liabilities
1.Shareholder's Funds
a. Share Capital 1 10,655,750
b. Reserve and Surplus 2 15,951,760
Total 26,607,510
II. Assets
1.Non-Current assets
Non-Current Investments 3 26,570,625
2.Current assets
Cash and cash equivalents 4 36,885
Total 26,607,510

Notes to Accounts:

Particulars Rs. Rs.


1.Share Capital

Authorized 20 lakhs shares of Rs 10 each 20,000,000

Issued & Paid up 1065575 shares of Rs 10Each(out 10,655,750


of the above 865575 shares have been issued for

consideration other than cash) 10,655,750


2.Reserve and Surplus

Securities Premium (Rs.11914875+5600000) 17,514,875

Loss for the Period (1,390,000)

Less: Proposed Dividend(2% of Rs 8655750) 173,115 (1,563,115)

Balance of Profit and Loss account carried Forward 15,951,760


3.Non-Current Investments

© The Institute of Chartered Accountants of Nepal 9


Paper 1 : Advanced Accounting

Shares in subsidiaries(W.N.4) 26,570,625


4.Cash and Cash equivalents

Cash at Bank (W.N.3) 36,885


5.Other income

Dividends Received From Subsidiaries 1,700,000


(Rs.1200000+500000)
6.Financial costs

Interest On Bank O/D 90,000


7.employee benefits expenses

Management expenses 1,400,000


8.Other expenses

Preliminary expenses 1,600,000

Working Notes:

1. Shares issued by Big Ltd.to Virgin capital Ltd.(VCL)

No. of shares issued 200,000


face value of Share Capital@ Rs 10 each 2,000,000
Securities Premium @ Rs.28 each 5,600,000
Total Cash Received From VCL 7,600,000

2.Overdraft of Big Ltd.as on 31.3.2075

Particulars Rs.
Towards Incorporation expenses 1,600,000
(Preliminary Expenses)
Towards Management expenses 1,400,000
Total Bank Overdraft availed 3,000,000
Interest @ 18% p.a for 2 months 90,000

3.Bank balance of Big Ltd.as on 31.3.2075

Bank account of Big Ltd.

© The Institute of Chartered Accountants of Nepal 10


Paper 1 : Advanced Accounting

Rs. Rs.
By Incorporation
01.02.2075 To overdraft 3,000,000 01.02.2075 Expenses 1,600,000
By management
31.03.2075 To VCL 7,600,000 31.03.2075 expenses 1,400,000
To dividend By Interest on
31.03.2075 small 1,200,000 31.03.2075 Overdraft 90,000
To dividend
Little 500,000 31.03.2075 By Overdraft 3,000,000

31.03.2075 By Dividend Paid 173,115


By Shares in small
31.03.2075 Ltd

bought from little ltd. 4,000,000


By Shares in Little
31.03.2075 Ltd bought

from small Ltd. 2,000,000


By Balance c/d (bal
Fig) 36,885

12,300,000 12,300,000

4.Investments of Big Ltd.in Projected Balance Sheet

Particulars Rs.
Purchase consideration paid for acquiring
shares of outside holders of:
Small Ltd 12,290,625
Little Ltd 8,280,000
Consideration paid in cash for acquiring
cross holdings:
From Small Ltd(shares of little ltd) 2,000,000
From Little Ltd(Shares of Small Ltd) 4,000,000
26,570,625

Answer 10
Calculation of Purchase Consideration
(i) Value of Net Assets of AB Ltd. MB Ltd. as on 31st Asadh, 2075

Particulars AB Ltd MB Ltd


Assets Taken Over:

Fixed Assets 700,000 300,000

© The Institute of Chartered Accountants of Nepal 11


Paper 1 : Advanced Accounting

Current Assets 400,000 1,100,000 100,000 400,000


Less :Liabilities taken Over:

Debentures(WN) 240,000 80,000

Trade Payables 60,000 (300,000) 20,000 (100,000)

800,000 300,000

(ii) Value of Shares of AB Ltd. MB Ltd.


AB Ltd. holds 1,500shares in Mb Ltd .i.e.1/4th of the shares of MB Ltd.
The Value Of shares of AB Ltd is Rs. 800,000 plus 1/4th of the value of the shares of MB
Ltd.
MB Ltd holds 4,000shares in AB Ltd i.e. 2/5th of the shares of AB Ltd.
Similarly, the value of shares of MB Ltd is Rs. 300,000 plus 2/5th of the value of shares of
AB Ltd.
Let ‘a’ denote the value of shares of AB Ltd. and ‘m’ denote the value of shares of MB ltd
then,
a=800000 + 1/4m; and
m=300000 + 2/5a.
Substituting the value of m,
a=800000+1/4(300000+2/5a)
a=800000+75000+1/10a
9/10a=875000
a=972222
m=300000+2/5(972222)
m=688889
(iii) Amount of Purchase Consideration

Particulars AB Ltd Rs. MB Ltd Rs.


Total Value of Shares(as determined Above) 972,222 688,889
Less: Internal Investments:
2/5 for shares held by MB LTD (388,889)
1/4 for shares held by AB LTD (172,222)
Amount due to outsiders 583,333 516,667

© The Institute of Chartered Accountants of Nepal 12


Paper 1 : Advanced Accounting

Purchase Consideration will be satisfied by AM Ltd.as follows.

Particulars AB Ltd Rs. MB Ltd Rs.


In Shares(of Rs. 100 each) 583300 516600
In cash 33 67

(iv) Net amount of goodwill Capital Reserve


Particulars Rs. Rs.
Total Purchase Consideration
AB Ltd 583,333
MB Ltd 516,667 1,100,000
Less: Net Assets Taken Over
AB Ltd 800,000
MB Ltd. 300,000 (1,100,000)
Nil

(Alternatively, the calculations may be made separately for both companies)


Balance Sheet of AM Ltd
As on 31st Asadh, 2075
I .Equity and Liabilities
1.Shareholder's Funds
a. Share Capital 1 1,099,900
b. Reserve And Surplus 2 70,000
2.Non-Current Liabilities
Long tem Borrowings 3 320,000
3.Current Liabilities
Trade Payables 80,000
Total 1,569,900
II.Assets
1.Non-Current assets
a.Fixed assets 4 1,000,000
b.Other non current assets 5 70,000
2. Current assets 499,900
Total 1,569,900
Notes to accounts
Particulars (Rs.) (Rs.)
1.Share Capital
10999 shares of Rs. 100 each 1,099,900
(all above shares are alloted as
fully paid up for consideration
other than cash)
2.Reserve and Surplus
Investment allowance Reserve 70,000
3.Long term Borrowings

© The Institute of Chartered Accountants of Nepal 13


Paper 1 : Advanced Accounting

15% debentures(WN) 320,000


4.Other Non-current assets
Amalgamation Adjustment A/c 70,000
5.Current assets(400000+100000) 500,000
Less: Purchase consideration paid
in cash Rs. (33+67) (100) 499,900

Working Note:
Calculation of Debentures to be issued

Particulars AB Ltd Rs. MB Ltd Rs.


12% Debentures 300,000 100,000
Interest on debentures@ 12%(a) 36,000 12,000
AM Ltd. Debentures rate of interest (b) 15% 15%
Debenture value to earn above calculated
interest (a/b) 240,000 80,000

Answer 11

Total No. of Shares of X. Ltd =600000/10 60,000shares


X. Ltd shares held by Y. Ltd 5,000 shares
Total No. of Shares of Y. Ltd =300000/10 30,000shares
Y. Ltd shares held by X. Ltd 6,000shares
Hence, X ltd holds 1/5th(6,000/30000) of Y Ltd total Shares

a) Statement of purchase Consideration payable by X Ltd.


I. For Equity Shareholders
8 equity shares of X Ltd for every 6 equity shares of Y. Ltd.
30000 shares *8/6=40000shares
th
Less:1/5 shares of X Ltd (8000) shares
Balance for Outsiders 32000shares
Less: 5000shares of X Ltd already with Y Ltd. (5000) shares
Shares to be issued 27000shares
Value of 27000 equity shares at Rs. 10 Rs. 270,000

II. For Preference Share Holders

Preference Share Capital of Y Ltd Rs. 100,000


Less: 10% discount Rs. 10,000
X Ltd's preference to be issued Rs. 90,000

© The Institute of Chartered Accountants of Nepal 14


Paper 1 : Advanced Accounting

Total purchase Consideration


Particulars Numbers Amount
Equity Shares @ Rs. 10 each 27000 Rs.270,000

Preference shares@ Rs. 100 each 900 Rs.90,000


Total Purchase Consideration Rs.360,000

b) Balance sheet of X Ltd after its absorption of Y Ltd


Particulars Note No. Rs.
I.Equity and Liabilities
1.Shareholder's Funds
a.Share Capital 1 11,60,000
b.Reserve and Surplus 2 376,000
2.Non-Current Liabilities
Long Term Borrowings 3 380,000
3.Current Liabilities
Trade Payables 4 390,000
Total 23,06,000
II.Assets
1.Non Current Assets
a.Fixed assets(700000*115%+287500) 10,92,500
b.Other Non Current Assets 5 18,000
2.Current Assets
a.Inventories(240000+304000) 544,000
b.Trade Receivables 6 610,500
c.Cash and Cash equivalents 7 41,000
Total 23,06,000

Notes to Accounts
Particulars Rs. Rs.
1.Share Capital
Equity Share Capital
87000(60000+27000)equity
shares of Rs. 10 each, fully paid up 870,000
(Out of the above,27000equity
shares have been issued for
consideration other than cash)
20000 10%preference shares of Rs.10
200,000
each
900 10%preference shares of Rs. 100
90,000 1,160,000
each
2.Reserve and Surplus
Revaluation reserve (15%Of Rs.700000) 105,000
Capital Reserve (WN-1) 25,000
Other Reserves (WN-4) 246,000 376,000

© The Institute of Chartered Accountants of Nepal 15


Paper 1 : Advanced Accounting

3.Long Term Borrowings


Secured(assumed)
12% debentures Existing 200,000
Add:Issued to Y.Ltd(WN-5,calculation
180,000
B)
4.Trade Payables
Creditors(220000+125000-10000) 335,000
Bills Payable(30000+25000) 55,000 390,000
5.Other Non Current Assets
Discount on issue of debentures
(WN-5,Calculation C) 18,000
6.Trade Receivables
Debtors(360000+180500-10000) 530,500
Bills Receivable(60000+20000) 80,000 610,500
7.Cash and Cash Equivalents
Cash at Bank (WN-3) 41,000

Working Notes:
1. Calculation of Capital Reserve

Net Assets Taken over from Y Ltd Rs.


Fixed Assets(Rs.250000*115%) 287,500
Inventory(Rs.320000*95%) 304,000
Debtors(Rs.190000*95%) 180,500
Bills Receivable 20,000
Cash at Bank 15,000
Total Assets 807,000
Liabilities Taken over :
Debentures(WN-5,Calculation A) 162,000
Creditors 125,000
Bills Payable 25,000
Total Liabllities(B) 312,000
Net assets taken over (A-B) 495,000
Less:Investment cancelled(i.e 5000shares (80,000)
held in X ltd)
415,000
Purchase Consideration (360,000)
Capital reserve 55,000
Less:Liquidation expenses reimbursed (30,000)
to Y Ltd)
Capital reserve 25,000

2.Cash taken over from Y. Ltd


Rs.
Cash balance given in Balance sheet of

© The Institute of Chartered Accountants of Nepal 16


Paper 1 : Advanced Accounting

Y.Ltd 40,000
Add:Dividend received from X.Ltd 5,000
(5000shares * Rs. 1)
45,000
Less:Dividend paid (30000shares *Rs.1) (30,000)
15,000

3.Cash balance in Balance Sheet (after absorption)


Rs.
Cash balance given in Balance sheet of
X.Ltd 110,000
Add:Cash taken over from Y.Ltd(WN-2) 15,000
125,000
Less:Dividend paid Rs.60000
Expenses on liqidation Rs.30000 (90,000)
35,000
Add:Dividend from Y Ltd 6,000
41,000

4. Other Reserves in the Balance Sheet (after absorption)


Rs.
Reserve given in Balance sheet of
X.Ltd 300,000
Add:Dividend from Y.Ltd(6000shares*Rs.1) 6,000
306,000
Less:Dividend declared (60000shares*Rs.1) (60,000)
246,000

5. Debenture holders Payment


Rs.
Debentureholders of Y Ltd 150,000
Add:Premium@ 8% 12,000
Value of debentureholder liability taken over
by X Ltd (A) 162,000
Issue Price of X.Ltd debentures@10% discount
(A/90%) (B) 180,000
Discount on issue of debentures © 18,000

6. Intercompany transactions
Creditors of Y Ltd include Rs. 10000 due to X.ltd
Therefore journal entry in the books of X Ltd will be
Creditor’s a/c dr. 10,000
To Debtors a/c 10,000
Branch Accounting
Answer 12

© The Institute of Chartered Accountants of Nepal 17


Paper 1 : Advanced Accounting

Dr. Chitwan Branch Stock Account Cr.


Particulars Rs. Particulars Rs.
By Goods sent to Branch
To Balance b/d 40,000 A/c(Return) 15,000

To Goods Sent to Branch A/c 420,000 By Goods sent to Branch A/c 12,000
To Goods Returned by B.
Debtors 500 (Transfer to Gwalior Branch)

By Branch Cash A/c (Cash Sales) 105,000


By Branch Debtors A/c(Credit
sales) 180,000
By Branch Adjustments(Normal
Loss) 350
(Rs.280*125/100)
By Branch
Adjustments(Pilferage) 600
(Rs.3000*25/125)

By Branch Profit & loss A/c 2,400


(Rs.3000*100/125)

By Branch Adjustment a/c(Load) 800

By Branch Profit & loss A/c(cost) 3,200

By Balance C/d 141,150

460,500 460,500

Dr. Chitwan Branch Stock Account Cr.


Particulars Rs. Particulars Rs.

To Branch Stock A/c(Normal Loss) 350 By Stock Reserve A/c 8,000


To Branch Stock A/c(Pilferage) 600 (Rs 40000*25/125)

To Branch Stock A/c(Lost In Fire) 800 By Goods Sent to Branch A/c 78,600
To Stock Reserve A/c 28,230
(141150*25/125)
To Gross Profit t/f to Branch P/L
A/c 56,620
86,600 86,600

© The Institute of Chartered Accountants of Nepal 18


Paper 1 : Advanced Accounting

Dr. Chitwan Branch Profit and Loss Account Cr.


Particulars Rs. Particulars Rs.

To Branch Expenses A/c: By Branch Adjustment A/c 56,620


Petty Expenses
31,900 (Gross Profit)
Insurance Charges
200 By Insurance Claim 3,000
Bad debts
400 32,500
To Branch Stock A/c(Pilferage) 2400
To Branch Stock A/c(Lost in Fire) 3200
To Net profit t/f to General P/L A/c 21,520

59,620 59,620

Dr. Chitwan Branch Stock Reserve Account Cr.


Particulars Rs. Particulars Rs.

To Branch Adjustment A/c 8,000 By Balance B/d 8,000

To Balance C/d 28,230 By Branch Adjustment A/c 28,230

36,230 36,230

Answer 13.
Trading and Profit And Loss Account
For the year ending on 31st Asadh, 2072
Dr. Cr.
Particulars Head Branch Rs. Particulars Head Office Branch
Office Rs Rs Rs.
To Opening Stock 124,150 32,150 By Sales 410,160 168,200
To Purchases 307,250 - By Goods sent to 81,900 -
Branch
To Goods Received By Closing Stock 45,270 14,160
from
Head Office - 81,900
To Gross profit c/d 105,930 68,310
537,330 182,360 537,330 182,360
To Salaries 35,280 14,680 By Gross Profit b/d 105,930 68,310
To Sundry Expenses 16,170 2,290

© The Institute of Chartered Accountants of Nepal 19


Paper 1 : Advanced Accounting

To Branch Manager’s - 10,040


Commission
outstanding
To Net Profit 54,480 41,300
105,930 68,310 105,930 68,310

Balance Sheet as at Asadh 31,2072


Liabilities Rs. Assets Rs.
Capital: 94,200 Cash on hand & at bank
Add: Profit-H.O 54,480 H.O 24,800
Branch 41,300 189,980 Branch 3,260 28,060
Sundry Creditors: Cash In 29,030
Transit(23230+5800)
H.O 24,600 Sundry Debtors:
Branch 6,150 30,750 H.O 89,700
Branch 7,550 97,250
Branch Manager’s Commission 10,040 Stock At Cost:
Outstanding
H.O 45,270
Branch 14,160 59,430
Furniture:
H.O 15,000
Branch 2,000 17,000
230,770 230,770
Working Note: Calculation of Manager’s Commission
Memorandum Mahendranagar Branch Trading and Profit and Loss Account
(Invoice value)
Dr. Cr.
Particulars Rs. Particulars Rs.
To Opening Stock 36,270 By Sales 168200
By Closing Stock 17660
To Goods Received from HO 102,500
To Gross profit c/d 47,090
185,860 185,860
To salaries 14,680 By Gross Profit b/d 47,090
To sundry expenses 2,290
To Manager's Commission
1/3 of Rs 30,120
(Rs.47,090-Rs.14,680- 10,040
Rs2,290)
To Net Profit 20,080
47,090 47,090

© The Institute of Chartered Accountants of Nepal 20


Paper 1 : Advanced Accounting

Partnership Accounting
Answer 14
Dr. Realisation Account Cr.
Particulars Rs. Particulars Rs.
To Sundry Assets: By Sundry Creditors 12,500
Land & Building 37,500 By Bank A/c:
Plant 11,250 Furniture Sold 3,700
Furniture 5,000 By A&B's Realisation A/c
Furniture 5,000 By A&B's Realisation A/c
Stock 53,250 (Assets taken over)
Debentures 15,000 Land and Building 50,000
To A & B Realisation Plant at 90% 10,125
A/C:
Creditors taken over 12,500 Stock at 80% 42,600
Debtors 10575 113,300
To Bank (Realisation 2,500 By Partners' Capital A/c:
expenses paid)
Loss on Realisation
A 3,500
B 2,500
K(7:5:3) 1,500 7,500
137,000 137,000

Dr. Partner's Capital Accounts Cr.


Particulars A (Rs.) B (Rs) K (Rs) Particulars A(Rs) B (Rs) K (Rs)
By Balance
To Realisation A/c 3,500 2,500 1,500 b/d 45,000 30,000 12,500

To Bank A/c - - 11,000 By Bank A/c 1,050 750 -


By A's & B's
To Share Capital 35,000 25,000 - A/c. 3,908 2,792 -

To Bank-(final Realisation
Payment) 11,458 6,042 - A/c.
7:5
(Profit)

49,958 33,542 12,500 49,958 33,542 12,500

Dr. Bank Account Cr.


Particulars Rs. Particulars Rs.

To Balance b/d 8,000 By Realisation A/c (expenses) 2,500

To Realisation A/c(Furniture) 3,700 By K's capital A/c 11,000

© The Institute of Chartered Accountants of Nepal 21


Paper 1 : Advanced Accounting

To A's Capital A/c 1,050

To B's Capital A/c 750

13,500 13,500

To COJIG & Co. P .Ltd. 60,000 By A & B's realisation A/c 12,500
(Sundry Creditors)

By A's Loan A/c 30,000

By B's Capital A/c 11,458

By B's Capital A/c 6,042

60,000 60,000

Dr. A & B's Realisation Account Cr.


Particulars Rs. Particulars Rs.

To Realisation A/c 113,300 By Realisation A/c 12,500

To Bank A/c 12,500 By COJIG & Co. P Ltd 120,000


To A's Capital A/c 3,908 (Purchase Considerations)
To B's Capital A/c 2,792

132,500 132,500

Dr. COJIG & Company P Ltd Account Cr.


Particulars Rs. Particulars Rs.

To A & B Realisation A/c 120,000 By Bank A/c 60,000

(Purchase Considerations) By Equity Share Capital A/c 60,000


(Shared by A& B 7:5)

120,000 120,000

© The Institute of Chartered Accountants of Nepal 22


Paper 1 : Advanced Accounting

Accounting for Shares & Debentures


Answer 15
Balance sheet of X Ltd as on 01.04.2077

Liabilities Rs. Assets Rs


Fixed Assets(at cost less
Share Capital: depreciation) 1,60,00,000
600000Equity shares of Rs 10Each 60,00,000 Investments Nil
Current assets,Loans and
(500000+100000) Advances
Reserve & Surplus Other current Assets 2,00,00,000
General Reserve(Note 5) 1,24,00,000 Cash & Bank Balances(note 6) 4,75,000
Share Premium
Account(100000*Rs 5.75) 5,75,000 Miscellaneous Expenditure Nil
Secured Loans Nil
Unsecured Loans
Other Loans 50,00,000
Current Liabilities & Provisions 1,25,00,000
3,64,75,000 3,64,75,000

Working notes:
1. Calculation of the number of shares to be allotted for conversion

Nos.
Total number of debentures outstanding 100000
Less: Number of debentures opting for cash 25000
Total Number of debentures opting for conversion 75000

Since 20% of the holding to be converted into equity shares, 15000 debentures (20% of 75000)
are to be converted. Therefore, the total number of equity shares to be allotted:
(15000*105)/Rs 15.75 = 100000equity shares of Rs 10 each at a premium of Rs 5.75 per Share.
2. Calculation of Required Cash for Payment

Rs.
Amount Payable to the debenture holders opting
for conversion:80% of (75000*Rs 105) 63,00,000
Amount Payable to the debenture holders opting
for cash(25000*105) 26,25,000
Required amount of Cash Payment 89,25,000

© The Institute of Chartered Accountants of Nepal 23


Paper 1 : Advanced Accounting

3. Debenture Redemption Fund Investment Account

Dr. Cr.
Date Particulars Rs Date Particulars Rs
To Balance b/d 40,00,000 By Bank A/C 44,00,000
To Debenture Redemption fund
A/C 4,00,000
(Profit)
44,00,000 44,00,000

4. Debenture Redemption Fund Account


Dr. Cr.
Date Particulars Rs Date Particulars Rs
To Premium on Redemption of
debenture 500,000 By Balance c/d 50,00,000
By Debenture
To general Reserve A/c 49,00,000 Redemption
Fund Investment A/c 400,000
54,00,000 54,00,000

5. General Reserve Account


Dr. Cr.
Date Particulars Rs Date Particulars Rs
To Balance C/d 1,24,00,000 By Balance b/d 75,00,000
By Debenture
Redemption
Fund A/c 49,00,000
1,24,00,000 1,24,00,000

6. Cash and Bank A/c


Dr. Cr.
Date Particulars Rs Date Particulars Rs
By Debenture holders
To Balance b/d 50,00,000 A/C 89,25,000
By Debenture Redemption Fund
Investment A/C 44,00,000 By Balance c/d 475,000
94,00,000 94,00,000

© The Institute of Chartered Accountants of Nepal 24


Paper 1 : Advanced Accounting

Answer 16
In the Books of HP Ltd Journal.

Dr. Cr.
Date Particulars Rs. Rs.
2015 July 1 Bank A/C……….Dr. 200,000
To Debentures Application A/C 200,000
(Being money received in respect of 2000,6%
debentures of Rs 100 each)
2015 July 1 Debentures Application A/C……….Dr. 200,000
To 6% Debentures A/C 200,000
(Being the issue of 2000,6% debentures Of Rs 100
each at par)
2015 Dec 31 Debentures Interest A/C……….Dr. 6,000
To Bank A/C 6,000
(Being Payment of the half yearly interest on
2000 debentures)
2015 Dec 31 Profit and Loss A/C……….Dr. 6,000
To Debentures Interest A/C 6,000
(Being the interest on debentures transferred
to Profit and Loss Account)
2016 May 31 Investment on own debentures A/C(98*200)….Dr 19,600
Debentures Interest A/C 500
To Bank A/C 20,100
(Being Purchase of 200 debentures @ Rs 98 ex-
interest)
2016 June 30 Debentures Interest A/C……….Dr. 5,500
To Bank A/C 5,400
To interest on own debentures A/C 100
(Being the interest on 1800 debentures paid in
cash and Interest on own debentures for 1 month
credited to Interest on own Debentures account)
2016 Dec 31 Debentures Interest A/C……….Dr. 6,000
To Bank A/C 5,400
To interest on own debentures A/C 600
(Being the interest on 1800 debentures paid in
cash and Interest on 200 debentures credited to
Interest on own Debentures account)
2016 Dec 31 Profit and Loss A/C……….Dr. 12,000
To Debentures Interest A/C 12,000
(Being the interest on debentures transferred
to Profit and Loss Account)
Interest on own debentures A/C……….Dr 700
To Profit and Loss A/C 700
(Being the interest on own debentures credited
to Profit and Loss Account)

© The Institute of Chartered Accountants of Nepal 25


Paper 1 : Advanced Accounting

2017 June 30 Debentures Interest A/C……….Dr. 6,000


To Bank A/C 5,400
To interest on own debentures A/C 600
(Being the interest on 1800 debentures paid in
cash and Interest on 200 debentures credited to
Interest on own Debentures account)
2017 Sept 30 Investment in own debentures A/C(Note1)……Dr 9,550
Debentures Interest A/C 150
To Bank A/C 9,700
(Being Purchase of 100 debentures @ Rs 97 cum-
interest;interest on 3 months included on the
purchase price)
2017 Dec 31 Debentures Interest A/C……….Dr. 5,850
To Bank A/C 5,100
To interest on own debentures A/C 750
(Being the interest on 1700 debentures paid in
cash and Interest on 200 debentures for 6months
and the interest on 100 debentures for 3 months
credited to Interest on own Debentures account)
2017 Dec 31 6% Debentures A/C……….Dr. 20,000
To Investment in own debentures A/C 19,600
To Profit on Cancellation A/C 400
(Being the cancellation of 200 debentures
Purchased on May 2016)
2017 Dec 31 Profit on Cancellation A/C………Dr. 400
To Capital Reserve A/C 400
(Being the profit on cancellation of debentures
transferred to Capital Reserve Account)
2017 Dec 31 Profit and Loss A/C……….Dr. 12,000
To Debentures Interest A/C 12,000
(Being the interest on debentures transferred
to Profit and Loss Account)
2017 Dec 31 Interest on own debentures A/C……….Dr 1,350
To Profit and Loss A/C 1,350
(Being the interest on debentures transferred
to Profit and Loss Account)
2017 Dec 31 Profit and Loss A/C……….Dr 20,000
To General Reserve A/C 20,000
(Being amount transferred to general reserve)

Balance Sheet of HP Limited as on 31.12.2017(extracts)


Liabilities Rs. Assets Rs
Secured Loan Investments
1800,6% debentures 180,000 Own debentures 9,550
Reserve & Surplus (face value Rs.10000)

© The Institute of Chartered Accountants of Nepal 26


Paper 1 : Advanced Accounting

Capital Reserve 400


General Reserve 20,000

Working Notes:
1. When debentures are purchased cum-interest, Investment in Own Debentures account
is debited with the amount of purchase price less interest accrued up to the date of
purchase. Therefore, investment in Own debentures account is to be debited by Rs
9,700 less Rs 150=Rs 9,550.It should be noted that even though Rs 9700 debentures
have been purchased for Rs 9550,there is no profit. Question of profit or loss will arise
at the time of cancellation of these debentures.
2. An Amount equal to the face value of debentures redeemed is to be transferred to
general reserve.

Answer 17
Financial Leverage Multiplier(FLM):
Financial Leverage Multiplier (FLM) is the connection between return on assets and return on
equity of the firm. The FLM is one of several ways of looking at the relative amounts of debt
and equity the firm is using to finance its assets. An important feature of FLM is the
relationship:
ROA*FLM=ROE
That implies that if ROE is important to investors in a firm, then the relative level of ROE can
be managed by changes in the FLM once ROA results can be anticipated.
FLM=Total Assets/Equity
Since Total Assets=Debt+Equity,
FLM is alternatively:
FLM =(Debt +Equity)/Equity=Debt/Equity+1

Answer 18
The objectives of government accounting and business accounting differ significantly. Hence,
there are various differences between government accounting and business accounting which
are described as follows:
Purpose
Government account is maintained by government offices to know the position of public fund
while business accounting is maintained by the business organizations to show the financial
performance, financial position, change in cash position and change in equity of the
organization during an accounting period.
Cash Basis and Accrual Basis of Accounting

© The Institute of Chartered Accountants of Nepal 27


Paper 1 : Advanced Accounting

Government accounting is prepared on cash basis while accrual basis of accounting is the
fundamental concept of business accounting.
Income Statement
Financial Statement of the commercial enterprises show the performance of the entity during a
period and financial position at a specific date. Income statement of the commercial enterprise
show the result of the operation of the entity, whereas government prepares the income and
expenditure account and it shows the cash surplus or deficit during a specific period.

Rules and Regulations


Government accounting is prepared following the rules and regulations prescribed by the
government while business accounting is prepared following the generally accepted accounting
principles namely Nepal Accounting Standard prescribed by Accounting Standard Board of
Nepal.
Auditing
Internal Audit of Government accounts is conducted by the office of Comptroller General and
final audit is conducted by the office of Auditor General. Internal audit of business account is
conducted by the person, either internal staff or external professional, appointed by the
management of business organization and final audit is conducted by the person holding
Certificate of Practice issued by ICAN.
Consolidated Fund
All revenues received by government of Nepal, all loans raised on the security of revenues and
all the money received in repayment of any loan made under the authority of any act and any
amount received by the government of Nepal is credited to the government fund known as
consolidated fund. Revenue of business can be kept in a single account or different accounts as
per the necessity of the business.
Level of Accounting
Books of account are divided into Central Level Accounting and Operating Level Accounting
in case of Government accounting while the business accounts may not be classified in such
manner.
Budgetary Control
Government accounting is totally based on budget approved by the parliament; any expenditure
not provision in approved budget cannot be incurred. Business organizations also prepare
budget but it is not necessary that all the transaction will be based on the budgets only.
Budgetary Control System is strictly followed in government accounting.

Solution 19
A. Supplementary Capital as per Capital Adequacy norms of banks and financial
institutions:

© The Institute of Chartered Accountants of Nepal 28


Paper 1 : Advanced Accounting

Supplementary Capital means the funds of a bank or financial institution kept under
such headings as may be prescribed by the Nepal Rastra Bank from time to time.
Provisions of directive 1 of Unified Directives on “Provisions relating to Capital
Adequacy’’ states that with a condition of not allowing to include more than core
capital, the amount under the following heads shall be included in the following
supplementary capital:
a. Provisions for General Loan Loss
b. Assets Revaluation Fund
c. Hybrid Capital Instrument
d. Unsecured Subordinated Term Loan
e. Exchange Equalization Fund
f. Investment Equalization Fund

B. Distribution of Management Expense:


Distribution of the management expense among the Profit and Loss account and
Revenue Account shall be done as per the Directives of the Insurance Board. The
separate directives issued for life Insurance Business Insurer and Non-life insurance
business insurer prescribes separate method of distribution of management expense.
The distribution shall be as follows for Non-Life Insurance Business Insurer.
Method of Distribution for Non-Life Insurance Business Insurer
a. Distribute 10% of the total management expense to the profit and loss account
b. Distribute the remaining 90% to the each of the revenue account as per the
following formula:
Management Expense = Weight/Total Weight *Total Management Expenses*0.9
Explanation:
1. Weight = Direct Insurance Premium-Agent commission of concerned category
of insurance business the insurer carried out.
2. Total Weight = Sum of Weight of every category of Insurance Business.

C. Annuity method for calculating Goodwill:


In the super profit method for calculating goodwill time value of money is not
considered. Although it was expected that super profit would be earned in five future
years, still no devaluation was done on the value of money for the time difference. In
fact, when money will be received in different points of time, its values should be
different depending upon the rate of interest. If 15% rate of interest and annual super
profit is Rs 3,000(say), then discounted value of super profit to be earned in different
future years will be as follows:
Super Discount Factor
Year Profit @ 15% Discounted value of super Profit
1 3,000 0.8696 2,608.8
2 3,000 0.7561 2,268.3
3 3,000 0.6575 1,972.5

© The Institute of Chartered Accountants of Nepal 29


Paper 1 : Advanced Accounting

4 3,000 0.5718 1,715.4


5 3,000 0.4972 1,491.6
10,056.6

So under the annuity method, discounted value of super profit becomes Rs 10,056.60
and not Rs 15,000
The word annuity is used to mean identical annual amount of super profit, so for
discounting it is possible to refer to annuity table. As per the annuity table, present value
of re 1 to be received at the end of each year for 5 years @15% interest p.a is 3.3522.So
value of goodwill under annuity method is Rs 3000*3.3522=10,056.60.

D. Steps of Acquisition for Business Combination (NFRS 03)


The following steps are involved in the acquisition for business combination:
Step 1: Identify the acquirer
Step 2: Determining the acquisition date
Step 3: Recognizing and measuring the identifiable assets acquired, the liabilities
assumed and any non-controlling interest in the acquiree; and
Step 4: Recognizing and measuring goodwill or a gain from bargain purchase

Answer 20
NAS 16 property, plant and equipment requires capitalization of expenses for
decommissioning, site restoration and similar liabilities. Paragraph 16 (c) of NAS 16 is
of critical significant. The cost of an item of PPE inter alia comprises of
“The initial estimate of the cost of dismantling and removing the item and restoring the
site on which it is located, the obligation for which an entity incurs either when the item
is acquired or as a consequence of having used the item during a particular period for
purposes other than to produce inventories during that period.”
The present value of these costs should be capitalized, with an equivalent liability set
up. The discount on this liability would then be unwound over the period until the costs
are paid. This means that the liability increases by the interest rate each year, with the
interest taken to finance costs in the statement of profit or loss.

© The Institute of Chartered Accountants of Nepal 30


Paper 2: Audit and Assurance

Paper 2: Audit and Assurance

© The Institute of Chartered Accountants of Nepal 1


Paper 2: Audit and Assurance

Revision Questions

GENERAL CONCEPTS- AUDITING AND ASSURANCE


Question No. 1
The auditor cannot reduce audit risk to zero and cannot obtain absolute assurance that the
financial statements are free from material misstatement due to fraud or error due to inherent
limitations of an audit. Discuss the matters that affect inherent limitations of an audit?

ETHICS
Question No. 2
PQR and Associates, a new firm of chartered accountants has developed their website
‘pqrassociates.com’. The firm mentioned the website address on its letter heads and also sent
the information about its website to prospective clients and other professional members via
email. Give your opinion about the validity of act done.

REGULATORY COMPLIANCE
Question No. 3
The finance manager of Him International Private Limited accused the statutory auditor Mr.
Subedi for divulging company confidential information to its competitors. The board of the
company then immediately removed the auditor without hearing any defence from the
auditor. The company then requested Company Registrar Office to appoint another statutory
auditor for the remaining period. Give your view.

PLANNING AN AUDIT ENGAGEMENT


Question No. 4
How inquiry can be used as risk assessment procedures by the auditor? Discuss.

Question No. 5
The auditor shall establish an overall audit strategy that sets the scope, timing and direction of
audit and guides the development of audit plan. What are the considerations to be taken while
establishing overall audit strategy?

Question No. 6
Mr. B has been appointed as the external auditor of ADE Company Ltd. He wants to apply
audit sampling in performing audit procedures. Guide him in designing audit sample and size.

Question No. 7
The auditor shall determine whether, in the auditor’s judgment, any of those accounting
estimates that have been identified as having high estimation uncertainty give rise to
significant risks. What are the specific substantive procedures to be performed for accounting
estimates that give rise to significant risks?

© The Institute of Chartered Accountants of Nepal 2


Paper 2: Audit and Assurance

Question No. 8
The auditor shall obtain an understanding of internal control relevant to the audit and shall
determine whether, on the basis of the audit work performed, the auditor has identified one or
more deficiencies in internal control. What are the matters that the auditor may consider in
determining whether a deficiency or combination of deficiencies in internal control
constitutes a significant deficiency?

Question No. 9
The auditor is responsible for communicating matters required by NSA 260 to those charged
with governance. How can auditor determine the appropriate person(s) within the entity’s
governance structure with whom to communicate?

Question No. 10
The external auditor of a Company has identified few conditions during the audit that may
cast significant doubt on the entity’s ability to continue as a going concern but he is not sure.
Suggest him the further audit procedures when such events or conditions are identified.

GATHERING AUDIT EVIDENCE DURING AN AUDIT ENGAGEMENT


Question No. 11
Write short note on test of controls.

Question No. 12
Differentiate between
a) Internal evidence and external evidence.
b) Provisions and Contingent liabilities

Question No. 13
How will you vouch/verify the following?
a) Payment of retirement gratuity to employees
b) Wages Paid to seasonal labourers
c) Contingent Liability
d) Capital WIP

Question No. 14
What do you mean by non-response in case of external confirmation? How would you deal
with non-response as an external auditor?

USING THE WORK OF OTHERS


Question No. 15
What do you mean by significant component as per NSA 600? How can the group
engagement partner determining the type of work to be performed on the financial
information of significant components?

© The Institute of Chartered Accountants of Nepal 3


Paper 2: Audit and Assurance

INTERNAL AUDIT AND CORPORATE GOVERNANCE


Question No. 16
a) What are the core principles of internal audit function?
b) Write short note on role of internal audit and corporate governance.

AUDIT CONCLUSIONS AND REPORTING


Question No. 17
a) What do you understand by review engagement? What are the objectives of practitioner
in a review of financial statements?
b) Mr. C, a practitioner has completed the compilation engagement and wants to issue the
report. What are the elements the practitioner’s report issued for the compilation
engagement shall contain?

Question No. 18
As an auditor, comment and give your views with explanations on following cases:
a) Sahara Hospital Pvt. Ltd. imported and installed an inventory software from Japan on
Ashad 15, 2076. The software price was $10,000 and the exchange rate as on that day
was Rs.110.10 per USD. The payment to the supplier could be made within 30 days as
per payment terms and the company had paid on Shrawan 10, 2076 when the rate was
Rs.113.2 per USD. The financial statement for FY 2075/76 shows the payable amount at
Rs. 1,101,000 whereas the exchange on Ashad end 2076 was Rs.114.10 per USD.
b) Jayamata Ltd. had installed the new machinery to manufacture new soap on Ashoj 2075.
To test the functioning of the machine, 100 samples of soaps were produced. Having
found no defects in the functioning, the commercial production started from Kartik 2075.
The samples were sold out with other soaps later produced. The production cost of
producing the samples have been included in Cost of production in profit and loss account
and the sales proceeds from samples have been clubbed with sales revenue in financial
statements for FY 2075/76.

GOVERNMENT AUDIT
Question No. 19
The audit of government expenditure is one of the major components of government audit.
What are the basic considerations for audit of expenditure?

AUDIT OF SPECIAL ORGANIZATIONS


Question No. 20
Mention the specific audit procedures to be followed by auditor in respect of leasing
transactions during the audit of a leasing company.

© The Institute of Chartered Accountants of Nepal 4


Paper 2: Audit and Assurance

Answers/Hints

GENERAL CONCEPTS- AUDITING AND ASSURANCE


Answer No. 1
The auditor cannot reduce audit risk to zero and cannot therefore obtain absolute assurance that
the financial statements are free from material misstatement due to fraud or error. This is because
there are inherent limitations of an audit, which result in most of the audit evidence on which the
auditor draws conclusions and bases the auditor’s opinion being persuasive rather than
conclusive. The inherent limitations of an audit arise from:
a) The nature of financial reporting;
b) The nature of audit procedures; and
c) The need for the audit to be conducted within a reasonable period of time and at a
reasonable cost.

The above factors are briefly discussed below:


a) The nature of financial reporting
The preparation of financial statements involves judgment by management in applying the
requirements of the entity’s applicable financial reporting framework to the facts and
circumstances of the entity. In addition, many financial statement items involve subjective
decisions or assessments or a degree of uncertainty, and there may be a range of acceptable
interpretations or judgments that may be made. Consequently, some financial statement items
are subject to an inherent level of variability which cannot be eliminated by the application of
additional auditing procedures. For example, this is often the case with respect to certain
accounting estimates.

b) The nature of audit procedures


There are practical and legal limitations on the auditor’s ability to obtain audit evidence. For
example: there is the possibility that management or others may not provide, intentionally or
unintentionally, the complete information that is relevant to the preparation of the financial
statements or that has been requested by the auditor. Accordingly, the auditor cannot be
certain of the completeness of information, even though the auditor has performed audit
procedures to obtain assurance that all relevant information has been obtained. Similarly,
fraud may involve sophisticated and carefully organized schemes designed to conceal it.
Therefore, audit procedures used to gather audit evidence may be ineffective for detecting an
intentional misstatement. The auditor is neither trained as nor expected to be an expert in the
authentication of documents. Also, an audit is not an official investigation into alleged
wrongdoing. Accordingly, the auditor is not given specific legal powers, such as the power of
search, which may be necessary for such an investigation.

© The Institute of Chartered Accountants of Nepal 1


Paper 2: Audit and Assurance

c) The need for the audit to be conducted within a reasonable period of time and at a reasonable
cost
The matter of difficulty, time, or cost involved is not in itself a valid basis for the auditor to
omit an audit procedure. Appropriate planning assists in making sufficient time and resources
available for the conduct of the audit. Notwithstanding this, the relevance of information, and
thereby its value, tends to diminish over time, and there is a balance to be struck between the
reliability of information and its cost. This is recognized in certain financial reporting
frameworks. Therefore, there is an expectation by users of financial statements that the
auditor will form an opinion on the financial statements within a reasonable period of time
and at a reasonable cost, recognizing that it is impracticable to address all information that
may exist or to pursue every matter exhaustively on the assumption that information is in
error or fraudulent until proved otherwise. Consequently, it is necessary for the auditor to
plan the audit so that it will be performed in an effective manner; direct audit effort to areas
most expected to contain risks of material misstatement, whether due to fraud or error, with
correspondingly less effort directed at other areas; and use testing and other means of
examining populations for misstatements.

Other matters that affect the inherent limitations of an Audit are:


• Fraud, particularly fraud involving senior management or collusion.
• The existence and completeness of related party relationships and transactions.
• The occurrence of non-compliance with laws and regulations.
• Future events or conditions that may cause an entity to cease to continue as a going concern.

ETHICS
Answer No. 2
According to guidelines on marketing professional services by professional accountants in public
practice issued by ICAN, the firm should ensure that none of the information contained in the
Website be circulated on their own or through email or by any other mode or technique except
on a specific "pull” request. The firm would also not issue any circular or any other
advertisement or any other material of any kind whatsoever by virtue of which they solicit
people to visit their Website. The firm would, however, be permitted to mention their Website
address on their professional stationery.
Here, PQR and Associates sent the information about its website to prospective clients and other
professional members via email. Thus it is not permissible to circulate the information as it
would solicit people to visit their website. However the website address can be mentioned on the
letter heads as per the provisions of the guidelines.

REGULATORY COMPLIANCE
Answer No. 3

© The Institute of Chartered Accountants of Nepal 2


Paper 2: Audit and Assurance

According to Section 111 of Companies act 2063, The auditor of a company shall be appointed
by the general meeting in the case of a public company, and, in accordance with the provision as
contained in the memorandum of association, articles of association or consensus agreement, if
any failing such provision, by the general meeting in the case of a private company. As per
section 119 of Companies act 2063, no auditor shall be removed pending the completion of audit
of accounts of any financial year for which he/she was appointed as the auditor. However, if any
auditor breaches the code of conduct of auditors or does any act against the interest of the
company which has appointed him as the auditor or commits any act contrary to the prevailing
law, such auditor may be removed through the same process whereby he/she was appointed as
auditor, by giving prior information to the Institute of Chartered Accountants of Nepal, and with
the approval of the regulatory authority, if any authorized by the prevailing law for the
regulation of business of the company concerned, and failing such authority, with the approval of
the Company Registrar Office. While removing an auditor, the auditor shall be provided with a
reasonable opportunity to defend him/herself. Further, section 113 of companies act has
provision that where the auditor appointed pursuant to the Act ceases to continue his/her office
for any reason, the Company Registrar Office may, at the request of the board of directors of the
company, appoint another auditor.
In the given case, the statutory auditor Mr. Subedi has been accused of divulging company
confidential information to its competitors which is breach of code of conduct. Thus he may be
removed. But here Him International Private Limited has removed the auditor by the board
without giving opportunity to defend and the company did not give prior information to ICAn
and has not obtained approval of company registrar office. The auditor can be removed only by
the general meeting of the company. At the same time, he should be given opportunity to defend
himself/herself and the company should give prior information to ICAN and obtain approval
from the company registrar office. Hence the removal of auditor is not valid. However, company
should request to Company Registrar Office to appoint another statutory auditor for the
remaining period after following appropriate procedure for removal of existing auditor.

PLANNING AN AUDIT ENGAGEMENT


Answer No. 4
Inquiry performed as risk assessment procedures may assist in assessing the risks of material
misstatement by providing information in order to provide a basis for designing and
implementing responses to the assessed risks. Inquiries of management and of others within the
entity who in the auditor’s judgment may have information, assist in identifying risks of material
misstatement due to fraud or error. Much of the information obtained by the auditor’s inquiries is
obtained from management and those responsible for financial reporting. However, the auditor
may also obtain information, or a different perspective in identifying risks of material
misstatement, through inquiries of others within the entity and other employees with different
levels of authority.

© The Institute of Chartered Accountants of Nepal 3


Paper 2: Audit and Assurance

• Inquiries of management may help auditor understand the entity and its environment
including entity’s internal control, related parties and transactions.
• Inquiries directed towards those charged with governance may help the auditor understand
the environment in which the financial statements are prepared.
• Inquiries directed toward internal audit personnel may provide information about internal
audit procedures performed during the year relating to the design and effectiveness of the
entity’s internal control and whether management has satisfactorily responded to findings
from those procedures.
• Inquiries of employees involved in initiating, processing or recording complex or unusual
transactions may help the auditor to evaluate the appropriateness of the selection and
application of certain accounting policies.
• Inquiries directed toward in-house legal counsel or external legal counsel may provide
information about such matters as litigation, compliance with laws and regulations,
knowledge of fraud or suspected fraud affecting the entity, warranties, post-sales
obligations, arrangements (such as joint ventures) with business partners and the meaning of
contract terms.
• Inquiries directed towards marketing or sales personnel may provide information about
changes in the entity’s marketing strategies, sales trends, or contractual arrangements with
its customers.

Answer No. 5
While establishing the overall audit strategy, the auditor shall take following into considerations:
i. The characteristics of the engagement that define its scope:
It includes the matters such as the financial reporting framework, industry-specific reporting
requirement expected audit coverage, including the number and locations of components to
be included, nature of the business segments to be audited, including the need for specialized
knowledge, availability of the work of internal auditors and the extent of the auditor’s
potential reliance on such work, expected use of audit evidence obtained in previous audits,
effect of information technology on the audit procedures, including the availability of data
and the expected use of computer-assisted audit techniques etc.

ii. The reporting objectives of the engagement to plan the timing of the audit and the nature of
the communications required;:
It includes the matters such as entity’s timetable for reporting, such as at interim and final
stages, the organization of meetings with management and those charged with governance to
discuss the nature, timing and extent of the audit work, the discussion with management and
those charged with governance regarding the expected type and timing of reports to be
issued and other communications, both written and oral, including the auditor’s report,
management letters and communications to those charged with governance, the discussion
with management regarding the expected communications on the status of audit work

© The Institute of Chartered Accountants of Nepal 4


Paper 2: Audit and Assurance

throughout the engagement, communication with auditors of components regarding the


expected types and timing of reports to be issued and other communications in connection
with the audit of components, the expected nature and timing of communications among
engagement team members.

iii. The factors that, in the auditor’s professional judgment, are significant in directing the
engagement team’s efforts;
It includes the matters such as the determination of materiality and, where applicable the
determination of materiality for components and communication thereof to component
auditors and the preliminary identification of significant components and material classes of
transactions, account balances and disclosures; preliminary identification of areas where
there may be a higher risk of material misstatement, the impact of the assessed risk of
material misstatement at the overall financial statement level on direction, supervision and
review, Significant business developments affecting the entity, including changes in
information technology and business processes, changes in key management, and
acquisitions, mergers and divestments, significant industry developments such as changes in
industry regulations and new reporting requirements, significant changes in the financial
reporting framework, such as changes in accounting standards and other significant relevant
developments, such as changes in the legal environment affecting the entity.

iv. The results of preliminary engagement activities and, where applicable, whether knowledge
gained on other engagements performed by the engagement partner for the entity is relevant;
and
It includes the matters such as Results of previous audits that involved evaluating the
operating effectiveness of internal control, including the nature of identified deficiencies and
action taken to address them.

v. The nature, timing and extent of resources necessary to perform the engagement.
It includes the matters such as the selection of the engagement team including, where
necessary, the engagement quality control reviewer and the assignment of audit work to the
team members, including the assignment of appropriately experienced team members to
areas where there may be higher risks of material misstatement and engagement budgeting,
including considering the appropriate amount of time to set aside for areas where there may
be higher risks of material misstatement.

Answer No. 6
Audit sampling is the application of audit procedures to less than 100% of items within a
population of audit relevance such that all sampling units have a chance of selection in order to
provide the auditor with a reasonable basis on which to draw conclusions about the entire
population. NSA 530 ‘Audit Sampling’ applies when the auditor has decided to use audit

© The Institute of Chartered Accountants of Nepal 5


Paper 2: Audit and Assurance

sampling in performing audit procedures. Accordingly, audit sampling can be applied using
either non-statistical or statistical sampling approaches. When designing an audit sample, the
auditor shall consider the purpose of the audit procedure and the characteristics of the population
from which the sample will be drawn. The auditor’s consideration includes the specific purpose
to be achieved and the combination of audit procedures that is likely to best achieve that purpose.
Consideration of the nature of the audit evidence sought and possible deviation or misstatement
conditions or other characteristics relating to that audit evidence will assist the auditor in
defining what constitutes a deviation or misstatement and what population to use for sampling.
For example, in a test of details relating to the existence of accounts receivable, such as
confirmation, payments made by the customer before the confirmation date but received shortly
after that date by the client, are not considered a misstatement. Also, a mis-posting between
customer accounts does not affect the total accounts receivable balance. Therefore, it may not be
appropriate to consider this a misstatement in evaluating the sample results of this particular
audit procedure, even though it may have an important effect on other areas of the audit, such as
the assessment of the risk of fraud or the adequacy of the allowance for doubtful accounts.

In considering the characteristics of a population, for tests of controls, the auditor makes an
assessment of the expected rate of deviation based on the auditor’s understanding of the relevant
controls or on the examination of a small number of items from the population. This assessment
is made in order to design an audit sample and to determine sample size. For example, if the
expected rate of deviation is unacceptably high, the auditor will normally decide not to perform
tests of controls. Similarly, for tests of details, the auditor makes an assessment of the expected
misstatement in the population. If the expected misstatement is high, 100% examination or use of
a large sample size may be appropriate when performing tests of details. In considering the
characteristics of the population from which the sample will be drawn, the auditor may
determine that stratification or value weighted selection is appropriate. The decision whether to
use a statistical or non-statistical sampling approach is a matter for the auditor’s judgment;
however, sample size is not a valid criterion to distinguish between statistical and non-statistical
approaches.

The auditor shall determine a sample size sufficient to reduce sampling risk to an acceptably low
level. The required sample size is affected by the level of sampling risk that the auditor is willing
to accept. The lower the risk the auditor is willing to accept, the greater the sample size will need
to be. The sample size can be determined by the application of a statistically-based formula or
through the exercise of professional judgment.

Answer No. 7
For accounting estimates that give rise to significant risks, the auditor shall obtain sufficient
appropriate audit evidence about whether management’s decision to recognize, or to not
recognize, the accounting estimates in the financial statements; and the selected measurement

© The Institute of Chartered Accountants of Nepal 6


Paper 2: Audit and Assurance

basis for the accounting estimates, are in accordance with the requirements of the applicable
financial reporting framework.

For accounting estimates that give rise to significant risks, the auditor shall evaluate how
management has assessed the effect of estimation uncertainty on the accounting estimate, and the
effect such uncertainty may have on the appropriateness of the recognition of the accounting
estimate in the financial statements; and the adequacy of related disclosures. In addition to other
substantive procedures, the auditor shall evaluate the following:
• How management has considered alternative assumptions or outcomes, and why it has
rejected them, or how management has otherwise addressed estimation uncertainty in
making the accounting estimate.
• Whether the significant assumptions used by management are reasonable
• Where relevant to the reasonableness of the significant assumptions used by management
or the appropriate application of the applicable financial reporting framework,
management’s intent to carry out specific courses of action and its ability to do so.

If, in the auditor’s judgment, management has not adequately addressed the effects of estimation
uncertainty on the accounting estimates that give rise to significant risks, the auditor shall, if
considered necessary, develop a range with which to evaluate the reasonableness of the
accounting estimate.

Answer No. 8
If the auditor has identified one or more deficiencies in internal control, the auditor shall
determine, on the basis of the audit work performed, whether, individually or in combination,
they constitute significant deficiencies. The significance of a deficiency or a combination of
deficiencies in internal control depends not only on whether a misstatement has actually
occurred, but also on the likelihood that a misstatement could occur and the potential magnitude
of the misstatement. Significant deficiencies may therefore exist even though the auditor has not
identified misstatements during the audit.
Examples of matters that the auditor may consider in determining whether a deficiency or
combination of deficiencies in internal control constitutes a significant deficiency include:
• The likelihood of the deficiencies leading to material misstatements in the financial
statements in the future.
• The susceptibility to loss or fraud of the related asset or liability.
• The subjectivity and complexity of determining estimated amounts, such as fair value
accounting estimates.
• The financial statement amounts exposed to the deficiencies.
• The volume of activity that has occurred or could occur in the account balance or class of
transactions exposed to the deficiency or deficiencies.
• The importance of the controls to the financial reporting process; such as:

© The Institute of Chartered Accountants of Nepal 7


Paper 2: Audit and Assurance

o General monitoring controls (such as oversight of management).


o Controls over the prevention and detection of fraud.
o Controls over the selection and application of significant accounting policies.
o Controls over significant transactions with related parties.
o Controls over significant transactions outside the entity’s normal course of business.
o Controls over the period-end financial reporting process (such as controls over non-
recurring journal entries).
• The cause and frequency of the exceptions detected as a result of the deficiencies in the
controls.
• The interaction of the deficiency with other deficiencies in internal control.

Answer No. 9
The auditor shall determine the appropriate person(s) within the entity’s governance structure
with whom to communicate. Those charged with governance refers to the person(s) or
organization(s) (e.g., a corporate trustee) with responsibility for overseeing the strategic direction
of the entity and obligations related to the accountability of the entity. This includes overseeing
the financial reporting process. For some entities in some jurisdictions, those charged with
governance may include management personnel, for example, executive members of a
governance board of a private or public sector entity, or an owner-manager.
Governance structures vary by jurisdiction and by entity, reflecting influences such as different
cultural and legal backgrounds, and size and ownership characteristics. For example:
• In some jurisdictions, a supervisory (wholly or mainly non-executive) board exists that is
legally separate from an executive (management) board (a “two-tier board” structure). In
other jurisdictions, both the supervisory and executive functions are the legal
responsibility of a single, or unitary, board (a “one-tier board” structure).
• In some entities, those charged with governance hold positions that are an integral part of
the entity’s legal structure, for example, company directors. In others, for example, some
government entities, a body that is not part of the entity is charged with governance.
• In some cases, some or all of those charged with governance are involved in managing
the entity. In others, those charged with governance and management comprise different
persons.
• In some cases, those charged with governance are responsible for approving the entity’s
financial statements (in other cases management has this responsibility).

In most entities, governance is the collective responsibility of a governing body, such as a board
of directors, a supervisory board, partners, proprietors, a committee of management, a council of
governors, trustees, or equivalent persons. In some smaller entities, however, one person may be
charged with governance, for example, the owner-manager where there are no other owners, or a
sole trustee. When governance is a collective responsibility, a subgroup such as an audit
committee or even an individual, may be charged with specific tasks to assist the governing body

© The Institute of Chartered Accountants of Nepal 8


Paper 2: Audit and Assurance

in meeting its responsibilities. Alternatively, a subgroup or individual may have specific, legally
identified responsibilities that differ from those of the governing body.
Such diversity means that it is not possible to specify for all audits the person(s) with whom the
auditor is to communicate particular matters. Also, in some cases, the appropriate person(s) with
whom to communicate may not be clearly identifiable from the applicable legal framework or
other engagement circumstances, for example, entities where the governance structure is not
formally defined, such as some family-owned entities, some not-for-profit organizations, and
some government entities. In such cases, the auditor may need to discuss and agree with the
engaging party the relevant person(s) with whom to communicate. In deciding with whom to
communicate, the auditor’s understanding of an entity’s governance structure and processes. The
appropriate person(s) with whom to communicate may vary depending on the matter to be
communicated.
When the entity is a component of a group, the appropriate person(s) with whom the component
auditor communicates depends on the engagement circumstances and the matter to be
communicated. In some cases, a number of components may be conducting the same businesses
within the same system of internal control and using the same accounting practices. Where those
charged with governance of those components are the same (e.g., common board of directors),
duplication may be avoided by dealing with these components concurrently for the purpose of
communication.

Answer No. 10
As per NSA 570 ‘Going Concern’, if events or conditions have been identified that may cast
significant doubt on the entity’s ability to continue as a going concern, the auditor shall obtain
sufficient appropriate audit evidence to determine whether or not a material uncertainty exists
related to events or conditions that may cast significant doubt on the entity’s ability to continue
as a going concern through performing additional audit procedures, including consideration of
mitigating factors. These procedures shall include:
(a) Where management has not yet performed an assessment of the entity’s ability to continue as
a going concern, requesting management to make its assessment.
(b) Evaluating management’s plans for future actions including, for example, its plans to
liquidate assets, borrow money or restructure debt, reduce or delay expenditures, or increase
capital in relation to its going concern assessment, whether the outcome of these plans is
likely to improve the situation and whether management’s plans are feasible in the
circumstances.
(c) Where the entity has prepared a cash flow forecast, and analysis of the forecast is a
significant factor in considering the future outcome of events or conditions in the evaluation
of management’s plans for future actions:
(i) Evaluating the reliability of the underlying data generated to prepare the forecast; and
(ii) Determining whether there is adequate support for the assumptions underlying the
forecast.

© The Institute of Chartered Accountants of Nepal 9


Paper 2: Audit and Assurance

(d) In addition to the procedures required (c), the auditor may compare:
• The prospective financial information for recent prior periods with historical results; and
• The prospective financial information for the current period with results achieved to date.
(e) Considering whether any additional facts or information have become available since the
date on which management made its assessment.
(f) Requesting written representations from management and, where appropriate, those charged
with governance, regarding their plans for future actions and the feasibility of these plans.

The examples of such audit procedures may include the following:


• Analysing and discussing cash flow, profit and other relevant forecasts with management.
• Analysing and discussing the entity’s latest available interim financial statements.
• Reading the terms of debentures and loan agreements and determining whether any have
been breached.
• Reading minutes of the meetings of shareholders, those charged with governance and
relevant committees for reference to financing difficulties.
• Inquiring of the entity’s legal counsel regarding the existence of litigation and claims and the
reasonableness of management’s assessments of their outcome and the estimate of their
financial implications.
• Confirming the existence, legality and enforceability of arrangements to provide or maintain
financial support with related and third parties and assessing the financial ability of such
parties to provide additional funds.
• Evaluating the entity’s plans to deal with unfilled customer orders.
• Performing audit procedures regarding subsequent events to identify those that either
mitigate or otherwise affect the entity’s ability to continue as a going concern.
• Confirming the existence, terms and adequacy of borrowing facilities.
• Obtaining and reviewing reports of regulatory actions.
• Determining the adequacy of support for any planned disposals of assets.

GATHERING AUDIT EVIDENCE DURING AN AUDIT ENGAGEMENT


Answer No. 11
Test of controls may be defined as an audit procedures designed to evaluate the operating
effectiveness of controls in preventing, or detecting and correcting material misstatements at the
assertion level. The auditor shall design and perform test of controls to obtain sufficient
appropriate audit evidence as to the operating effectiveness of relevant controls when:
a) The auditor’s assessment of risks of material misstatement at the assertion level includes an
expectation that the controls are operating effectively (i.e. the auditor intends to rely on the
operating effectiveness of controls in determining the nature, timing and extend o
substantive procedures); or
b) Substantive procedures alone cannot provide sufficient appropriate audit evidence at the
assertion level.

© The Institute of Chartered Accountants of Nepal 10


Paper 2: Audit and Assurance

Although the purpose of a test of controls is different from the purpose of a test of details, both
may be accomplished concurrently by performing a test of controls and a test of details on the
same transaction, also known as a dual-purpose test. For example, the auditor may design, and
evaluate the results of, a test to examine an invoice to determine whether it has been approved
and to provide substantive audit evidence of a transaction. A dual-purpose test is designed and
evaluated by considering each purpose of the test separately.

Tests of control may include:


• Inspection of documents supporting transactions and other events to gain audit evidence
that internal controls have operated properly, for example verifying that a transaction has
been authorised.
• Inquiries about, and observation of, internal controls which leave no audit trail, for
example, determining who actually performs each function and not merely who is
supposed to perform it.
• Re-performance of internal controls, for example, reconciliation of bank accounts, to
ensure they were correctly performed by the entity.
• Testing of internal control operating on specific computerised application or over the
overall information technology function, for example, access or program change controls.

Answer No. 12
a) Internal evidence and external evidence
Basis Internal Evidence External Evidence
Source Evidence which originates within the Evidence which originates outside
organization being audited is internal the organization being audited is
evidence. external evidence.
Reliability Less reliable. More reliable.
Quantity In audit situation, the bulk of evidenceIn audit situation, few of evidence
that an auditor gets in internal in that an auditor gets in external in
nature. nature.
Control The client entity and its staffs have the
The client entity and its staffs have
control over the internal evidence. no control over the external
evidence
Examples For example, sales invoice, debit and For example, purchase invoice,
credit notes, goods received note, supplier’s challan, quotations,
inspection report, minutes, etc. confirmations, bank statement.

b) Provisions and Contingent liabilities


Basis Provisions Contingent liabilities
Meaning A provision is present liability of Contingent liabilities are possible
uncertain timing or amount. obligations whose existence will be

© The Institute of Chartered Accountants of Nepal 11


Paper 2: Audit and Assurance

confirmed by uncertain future events


that are not wholly within the control
of the entity.
Recognition An entity recognises a provision if it If outflow of cash or other economic
Criteria is probable that an outflow of cash or resources is not probable, the item is
other economic resources will be treated as contingent liabilities.
required to settle the provision.
Measurement A provision is measured at the Their amount cannot be measured
amount that the entity would reliably because settlement is not
rationally pay to settle the obligation probable.
at the end of the reporting period or
to transfer it to a third party at that
time. A provision is discounted to its
present value.
Presentation Provisions are present in statement of A contingent liability is not
and financial position under current recognised in the statement of
disclosure liabilities. financial position. However, unless
the possibility of an outflow of
economic resources is remote, a
contingent liability is disclosed in the
notes.
Examples For examples: warranty obligations; For example: litigation against the
legal or constructive obligations to entity when it is uncertain whether
clean up contaminated land or restore the entity has committed an act of
facilities; and obligations caused by a wrongdoing and when it is not
retailer’s policy to make refunds to probable that settlement will be
customers. needed.

Answer No. 13
a) The payment of retirement gratuity to employees can be vouched as follows:
i. Examine the basis on which gratuity payable is worked out- actuarial or agreement or
on the assumptions that all employees retire on the balance sheet date.
ii. Ensure that the basis of computing gratuity is valid and comply with the specific by-
rules of the entity or as per other laws applicable to the entity.
iii. Verify computation of liability of gratuity on aggregate basis.
iv. Check the amount of gratuity paid to employees who retired during the year with
reference to the number of years of service rendered by the retiring employees and
their salary basis.
v. Ensure the concern has adhered to the accounting treatment in accordance with NAS
on ‘Employee Benefits’ and Retirement Payments.

© The Institute of Chartered Accountants of Nepal 12


Paper 2: Audit and Assurance

vi. Ensure the compliance of local tax laws on the payment of retirement fund.
vii. Examine the mode of payment of retirement gratuity whether it has been made
through account payee cheque, or bank transfer.

b) Wages paid to seasonal labourers can be vouched as follows:


i. Ascertain and evaluate the internal control system for recruitment and usage of
seasonal labourers.
ii. Examine that these labourers are hired on proper authority and the rates of pay are
authorized at appropriate levels. Also ensure that pay is not below minimum wages
established by government.
iii. Ensure the attendance is properly checked by the Time keeping Department.
iv. Check that the certificate regarding work done by the labourers has been given by the
proper person, in case the labourers have been appointed on a per piece basis.
v. Check the computation of wages payable to the labourers, after taking into account
the deductions.
vi. Confirm that all the payments to the labourers have been acknowledged. Check the
payment mode of wages and ensure the compliance of local tax laws.
vii. See the time and job records, to ensure that the labourers have been paid for time
worked. See the treatment of abnormal idle time.
viii. Reconcile the number of seasonal labourers on payroll as per the Personnel
Department’s records vis-à-vis the number of labourers to whom the wages have been
paid, to ensure that there no ghost/dummy workers. This assumes greater importance,
if the seasonal labourers are hired on temporary basis, and not on payment payroll.

c) A contingent liability will be known or determined only on the occurrence or non-occurrence


of one or more uncertain future events. The uncertainty as to whether there will be any legal
obligation distinguishes a contingent liability from an actual liability. NAS 37 requires that in
case there is a probability that a loss may be incurred and a reasonable estimate of the
amount can be made, then such contingent liability must be adjusted in the financial
statements. Otherwise, disclosure will have to be made describing nature of the event,
uncertainties affecting the event and estimate of the financial effect or a statement that such
an estimate cannot be made.
The auditor may take following steps to verify the contingent liabilities:
i. Inspect the minute books of the company to ascertain all contingent liabilities known to
the company.
ii. Examine the contracts entered into by the company and the likelihood of contingent
liabilities emanating there from.
iii. Scrutinize the lawyer’s bills to track unreported contingent liabilities.
iv. Examine bank letters in respect of bills discounted and not matured.

© The Institute of Chartered Accountants of Nepal 13


Paper 2: Audit and Assurance

v. Examine bank letters to ascertain guarantees on behalf of other companies or


individuals.
vi. Discuss with various functional officers of the company about the possibility of
contingent liability existing in their respective field.
vii. Obtain a certificate from the management that all known contingent liabilities have
been included in the accounts and they have been properly disclosed.
viii. Ensure that proper disclosure has been made as per NAS 37 Provisions, Contingent
Liabilities and Contingent Assets.

d) Capital Work in Progress (WIP) can be verified as follows:


i. Ensure that the capital project is authorised by appropriate authority i.e. management or
board. See the relevant minutes for the purpose.
ii. Obtain the break up in details of the amount shown in the Balance Sheet under this head.
iii. Check purchase cost of plant machinery or other assets with reference to the contract
with, and amount paid to the suppliers.
iv. Examine the allocation of common costs to the capital WIP in case such items have been
constructed internally.
v. Ensure the assets already put to commercial use are not included under Capital WIP.
vi. Verify that only expenses incurred up to pre commissioning stage are capitalised under
this head.
vii. Obtain the certificate of the engineer to ascertain the quantum of the Capital WIP and
whether the value is correctly represented in the Balance Sheet, and its transfer to Fixed
Assets on completion of the project or installation of the plant.
viii. See the Capital WIP is properly disclosed in the Balance Sheet under the head Fixed
Assets as per the presentation and disclosure requirement.
ix. Ensure the compliance of local tax laws.

Answer No. 14
Non response is defined as a failure of the confirming party to respond, or fully respond, to a
positive confirmation request, or a confirmation request returned undelivered.
In the case of each non-response, the auditor shall perform alternative audit procedures to obtain
relevant and reliable audit evidence. Examples of alternative audit procedures the auditor may
perform include:
• For accounts receivable balances – examining specific subsequent cash receipts, shipping
documentation, and sales near the period end.
• For accounts payable balances – examining subsequent cash disbursements or
correspondence from third parties, and other records, such as goods received notes.
Further, the nature and extent of alternative audit procedures are affected by the account and
assertion in question. A non-response to a confirmation request may indicate a previously
unidentified risk of material misstatement. In such situations, the auditor may need to revise the

© The Institute of Chartered Accountants of Nepal 14


Paper 2: Audit and Assurance

assessed risk of material misstatement at the assertion level, and modify planned audit
procedures. For example, fewer responses to confirmation requests than anticipated, or a greater
number of responses than anticipated, may indicate a previously unidentified fraud risk factor
that requires evaluation according to NSA 240 ‘the auditor’s responsibilities relating to fraud in
an audit of financial statements’. Accordingly, the auditor shall evaluate whether the information
obtained from the other risk assessment procedures and related activities performed indicates that
one or more fraud risk factors are present. While fraud risk factors may not necessarily indicate
the existence of fraud, they have often been present in circumstances where frauds have occurred
and therefore may indicate risks of material misstatement due to fraud.

USING THE WORK OF OTHERS


Answer No. 15
According to definition provided by NSA 600 ‘special considerations—audits of group financial
statements (including the work of component auditors)’, significant component refers to a
component identified by the group engagement team (i) that is of individual financial
significance to the group, or (ii) that, due to its specific nature or circumstances, is likely to
include significant risks of material misstatement of the group financial statements. As the
individual financial significance of a component increases, the risks of material misstatement of
the group financial statements ordinarily increase. The group engagement team may apply a
percentage to a chosen benchmark as an aid to identify components that are of individual
financial significance. Depending on the nature and circumstances of the group, appropriate
benchmarks might include group assets, liabilities, cash flows, profit or turnover. For example,
the group engagement team may consider that components exceeding 15% of the chosen
benchmark are significant components. A higher or lower percentage may, however, be deemed
appropriate in the circumstances. The group engagement team may also identify a component as
likely to include significant risks of material misstatement of the group financial statements due
to its specific nature or circumstances that is, risks that require special audit consideration. For
example, a component could be responsible for foreign exchange trading and thus expose the
group to a significant risk of material misstatement, even though the component is not otherwise
of individual financial significance to the group.

For a component that is significant due to its individual financial significance to the group, the
group engagement team, or a component auditor on its behalf, shall perform an audit of the
financial information of the component using component materiality.

For a component that is significant because it is likely to include significant risks of material
misstatement of the group financial statements due to its specific nature or circumstances, the
group engagement team, or a component auditor on its behalf, shall perform one or more of the
following:
(a) An audit of the financial information of the component using component materiality.

© The Institute of Chartered Accountants of Nepal 15


Paper 2: Audit and Assurance

(b) An audit of one or more account balances, classes of transactions or disclosures relating to
the likely significant risks of material misstatement of the group financial statements.
(c) Specified audit procedures relating to the likely significant risks of material misstatement of
the group financial statements. For example, in the case of a likely significant risk of inventory
obsolescence, the group engagement team may perform, or request a component auditor to
perform, specified audit procedures on the valuation of inventory at a component that holds a
large volume of potentially obsolete inventory, but that is not otherwise significant.

INTERNAL AUDIT AND CORPORATE GOVERNANCE


Answer No. 16
a) Internal audit function is a function of an entity that performs assurance and consulting
activities designed to evaluate and improve the effectiveness of the entity’s governance, risk
management and internal control processes. The core principles, taken as whole, articulate
internal audit effectiveness. For an internal audit function to be considered effective, all
principles should be present and operating effectively. How an internal auditor, as well as an
internal audit function, demonstrates achievement of the core principles may be quite different
from organization to organization, but failure to achieve any of the principles would imply
that an internal audit activity was not as effective as it could be in achieving internal audit’s
objectives. The core or basic principles recommended to be followed during internal audit
function in general are given below:
i. Independence: The Internal Auditor shall be free from any undue influences which force
him to deviate from the truth. This independence shall be not only in mind, but also in
appearance. Also, the internal auditor shall resist any undue pressure or interference in
establishing the scope of the assignments or the manner in which these are conducted
and reported, in case these deviate from set objectives. The overall organisation
structure of key personnel, the position and reporting of the Chief Internal Auditor
within this structure, along with the powers and authority which is derived from
superiors further establishes the independence of the Internal Auditor.
ii. Integrity and Objectivity: The Internal Auditor shall be honest, truthful and be a person
of high integrity. He shall operate in a highly professional manner and seen to be fair in
all his dealings. He shall avoid all conflicts of interest and not seek to derive any undue
personal benefit or advantage from his position. The Internal Auditor shall conduct his
work in a highly objective manner, especially in gathering and evaluation of facts and
evidence. He shall not allow prejudice or bias to override his objectivity, especially in
arriving at conclusions or reporting his opinion.
iii. Due professional care: The Internal Auditor shall exercise due professional care and
diligence while carrying out the internal audit. The Internal Auditor shall exercise
reasonable care in carrying out the work to ensure the achievement of planned
objectives.

© The Institute of Chartered Accountants of Nepal 16


Paper 2: Audit and Assurance

iv. Confidentiality: The Internal Auditor shall at all times, maintain utmost confidentiality
of all information acquired during the course of the audit work. Under no circumstance
any confidential information shall be shared with third parties outside the company,
without the specific approval of the Management or Client or unless there is a legal or a
professional responsibility to do so (e.g., to share information with Statutory Auditors).
v. Skills and competence: The Internal Auditor shall have sound knowledge, strong inter-
personal skills, practical experience and professional expertise in certain areas and other
competence required conducting a quality audit. He shall undertake only those
assignments for which he has the requisite competence.
vi. Risk Based Audit: The Internal Auditor shall identify the important audit areas through
a risk assessment exercise and tailor the audit activities such that the detailed audit
procedures are prioritised and conducted over high risk areas and issues, while less time
is devoted to low risk areas through curtailed audit procedures. Additionally, this
approach shall ensure that risks under consideration are more aligned to the overall
strategic and company objectives rather than narrowly focused on process objectives.
vii. System and Process Focus: An Internal Auditor shall adopt a system and process
focused methodology in conducting audit procedures. This methodology is more
sustainable than the one adopted to test transactions and balances as it goes beyond
“error detection” to include “error prevention”. It requires a root cause analysis to be
conducted on deviations to identify opportunities for system improvement or
automation, to strengthen the process and prevent a repetition of such errors.
viii. Participation in Decision Making: In conducting internal audit assignments, the Internal
Auditor shall avoid passing any judgement or render an opinion on past management
decisions. As part of his advisory role, the Internal Auditor shall avoid participation in
operational decision making which may be subject of a subsequent audit. The focus of
the Internal Auditor shall remain with the quality and operating effectiveness of the
decision making process and how best to strengthen it, such that the chance of flawed or
erroneous decisions is minimised.
ix. Sensitive to Multiple Stakeholders Interests: The Internal Auditor shall evaluate the
implications of his observations and recommendations on multiple stakeholders,
especially where diverse interests may be conflicting in nature. In such situations, the
Internal Auditor shall remain objective and present a balanced view. This would permit
senior management to make a decision using all the information and balance the
strategy and objectives of the company with the expectations and interests of its
multiple stakeholders.
x. Quality and Continuous improvement: The quality of the internal audit work shall be
paramount for the Internal Auditor since the credibility of the audit reports depends on
the reliability of reported findings. The Internal Auditor shall ensure that a self-
assessment mechanism is in place to monitor his own performance and also that of his

© The Institute of Chartered Accountants of Nepal 17


Paper 2: Audit and Assurance

subordinates and external experts on whom he is relying to complete some part of the
audit work.

b) Internal Auditing means the appraisal activity established within an entity as a service to the
entity. Its functions include, amongst other things, monitoring internal control. To elaborate,
internal auditing is an independent, objective assurance and consulting activity designed to
add value and improve an organization's operations. It helps an organization accomplish its
objectives by bringing a systematic, disciplined approach to evaluate and improve the
effectiveness of risk management, control, and governance processes. Internal auditing is a
catalyst for improving an organization's governance, risk management and management
controls by providing insight and recommendations based on analyses and assessments of
data and business processes. With commitment to integrity and accountability, internal
auditing provides value to governing bodies and senior management as an objective source of
independent advice. The internal audit function may assess the governance process in its
accomplishment of objectives on ethics and values, performance management and
accountability, communicating risk and control information to appropriate areas of the
organization and effectiveness of communication among those charged with governance,
external and internal auditors, and management.
Internal auditors provide an independent and objective assessment of the effectiveness and
efficiency of a company’s operations, specifically its internal control structure. The internal
audit function helps an organization accomplish its objectives by bringing a systematic,
disciplined approach to evaluate and improve the effectiveness of risk management, control,
and governance processes. The scope of internal auditing is broad and may involve the
efficiency of operations, IT controls, the reliability of financial reporting, deterring and
detecting fraud, and compliance with laws and regulations. Internal Auditors may also
conduct compliance and operational audits, offering solutions for weaknesses in internal
controls and verifying that all laws and regulations are upheld.
The conceptualization of the role of internal audit is the increasing of centralization of risk
management and internal control in corporate governance. Hence, internal audit can be
conceptualized as one mechanism that can help management and the board of directors to
manage risk by not only identifying and evaluating operating, reporting and compliance, but
also providing advice and recommendations related to appropriate actions for management to
manage risk and develop the company in order to achieve established corporate goals.

AUDIT CONCLUSIONS AND REPORTING


Answer No. 17
a) A review engagement is conducted to provide limited assurance that there are no material
modifications that should be made to the financial statements for them to be in conformity
with the financial reporting framework. Limited assurance is the level of assurance obtained
where engagement risk is reduced to a level that is acceptable in the circumstances of the

© The Institute of Chartered Accountants of Nepal 18


Paper 2: Audit and Assurance

engagement, but where that risk is greater than for a reasonable assurance engagement, as the
basis for expressing a this conclusion. The combination of the nature, timing and extent of
evidence gathering procedures is at least sufficient for the practitioner to obtain a meaningful
level of assurance. To be meaningful, the level of assurance obtained by the practitioner is
likely to enhance the intended users’ confidence about the financial statements. The review
engagement or limited assurance is sometimes calling negative engagement. This type of
engagement is different from a reasonable assurance engagement. Generally, this
engagement, practitioner performs fewer procedures and review to support a conclusion on
the financial statements in terms of whether anything has come to the auditor’s attention to
indicated that the financial statements are not prepared in accordance with the applicable
financial reporting framework.

The practitioner’s objectives in a review of financial statements under this NSRE 2400 are to:
i. Obtain limited assurance, primarily by performing inquiry and analytical procedures,
about whether the financial statements as a whole are free from material misstatement,
thereby enabling the practitioner to express a conclusion on whether anything has come
to the practitioner’s attention that causes the practitioner to believe the financial
statements are not prepared, in all material respects, in accordance with an applicable
financial reporting framework; and
ii. Report on the financial statements as a whole and communicate, as required by this
NSRE 2400.

b) The practitioner’s report issued for the compilation engagement shall be in writing, and shall
include the following elements:
1) The report title;
2) The addressee(s), as required by the terms of the engagement;
3) A statement that the practitioner has compiled the financial information based on
information provided by management;
4) A description of the responsibilities of management, or those charged with governance as
appropriate, in relation to the compilation engagement, and in relation to the financial
information;
5) Identification of the applicable financial reporting framework and, if a special purpose
financial reporting framework is used, a description or reference to the description of that
special purpose financial reporting framework in the financial information;
6) Identification of the financial information, including the title of each element of the
financial information if it comprises more than one element, and the date of the financial
information or the period to which it relates;
7) A description of the practitioner’s responsibilities in compiling the financial information,
including that the engagement was performed in accordance with this NSRS (Nepal

© The Institute of Chartered Accountants of Nepal 19


Paper 2: Audit and Assurance

Standard in Related Services), and that the practitioner has complied with relevant
ethical requirements;
8) A description of what a compilation engagement entails in accordance with this NSRS;
9) Explanations that:
(i) Since a compilation engagement is not an assurance engagement, the practitioner is
not required to verify the accuracy or completeness of the information provided by
management for the compilation, and
(ii) Accordingly, the practitioner does not express an audit opinion or a review conclusion
on whether the financial information is prepared in accordance with the applicable
financial reporting framework.
10) If the financial information is prepared using a special purpose financial reporting
framework, an explanatory paragraph that:
(i) Describes the purpose for which the financial information is prepared and, if
necessary, the intended users, or contains a reference to a note in the financial
information that discloses this information; and
(ii) Draws the attention of readers of the report to the fact that the financial information is
prepared in accordance with a special purpose framework and that, as a result, the
information may not be suitable for other purposes;
11) The date of the practitioner’s report;
12) The practitioner’s signature; and
13) The practitioner’s address.

Answer No. 18
a) According to NAS 22 ‘The effects of changes in Foreign Exchange Rates’ at the end of each
reporting period the foreign currency monetary items shall be translated using the closing
rate and exchange differences arising on translating monetary items at rates different from
those at which they were translated on initial recognition during the period or in previous
financial statements shall be recognised in profit or loss in the period in which they arise.
When monetary items arise from a foreign currency transaction and there is a change in the
exchange rate between the transaction date and the date of settlement, an exchange difference
results. When the transaction is settled within the same accounting period as that in which it
occurred, all the exchange difference is recognised in that period. However, when the
transaction is settled in a subsequent accounting period, the exchange difference recognised
in each period up to the date of settlement is determined by the change in exchange rates
during each period.
In the given case, the payable for inventory software was valued using the exchange rate of
transaction date. Here the payable amount is foreign currency monetary items and thus shall
be translated using the closing rate. The exchange difference arising there from should be
recognised in profit and loss for FY 2075/76. Hence the payable amount should be shown at

© The Institute of Chartered Accountants of Nepal 20


Paper 2: Audit and Assurance

Rs. 1,141,000 and the loss of Rs. 40,000 should be recognised. The auditor may qualify the
opinion if the management did not revise the financial statements accordingly.

b) According to NAS 16 ‘Property Plant and Equipment’, any costs directly attributable to
bringing the asset to the location and condition necessary for it to be capable of operating in
the manner intended by management shall be included in the cost of an item of property,
plant and equipment. The costs of testing whether the asset is functioning properly, after
deducting the net proceeds from selling any items produced while bringing the asset to that
location and condition such as samples produced when testing equipment belongs to directly
attributable costs.
In the given case, the company has included the cost of samples produced while testing the
machinery under cost of production and the sales proceeds of the samples under sales
revenue in profit and loss account. Thus this treatment is against the stated provision of NAS
16. The cost of testing the machinery less the proceeds from sample sales should be included
in the cost of machinery and shown in Balance Sheet. The auditor may qualify the opinion if
the management did not revise the financial statements accordingly.

GOVERNMENT AUDIT
Answer No. 19
The basic standards set for audit of government expenditure are to ensure that there is provision
of funds authorized by the competent authority fixing the limits within which expenditure can be
incurred. The auditor should consider the following things with regards to audit of expenditure:
i. Verify that the expenditure incurred conforms to the relevant provisions of the
statutory enactment and in accordance with the financial rules and regulations framed
by the competent authority. Such an audit can be termed as compliance audit. It is the
function of audit to carry out the examination of the Constitution, various laws, rules,
regulations and orders issued by the executive authorities to see that whether the
expenditure comply to these laws.
ii. Verify/examine that there is sanction, either special or general, accorded by
competent authority authorising the expenditure. The auditor should ensure whether
the expenditure is properly covered by a sanction and also satisfy that the authority
sanctioning it is competent for the purpose by virtue of the powers vested in it by the
provisions of the Constitution and of the laws, rules or orders made thereunder.
iii. Verify/ensure that there is provision of funds out of which expenditure can be
incurred and the same has been authorised by competent authority and the amount of
expenditure does not exceed the appropriation made.
iv. Verify/ensure that the expenditure is incurred with due regard to the broad and
general principles of financial propriety. Such an audit can be termed as propriety
audit. Accordingly, the auditor should bring out the cases of improper, avoidable or

© The Institute of Chartered Accountants of Nepal 21


Paper 2: Audit and Assurance

infructuous expenditure even though the expenditure has been incurred in conformity
with the existing rules and regulations.
v. Examine/ensure that the various programmes, schemes and projects where large
financial expenditure has been incurred are being run economically and are yielding
results expected of them. Such an audit can be termed as performance audit.

AUDIT OF SPECIAL ORGANIZATIONS


Answer No. 20
In respect of leasing transactions entered into by a leasing company involved in the leasing
capital goods, the auditor should check/verify the following:
1. The objective clause of the leasing company to see that the goods like capital goods,
consumer durables etc. in respect of which the company can undertake such activities.
Further, whether company can undertake financing activities or not.
2. Whether there exists a procedure to ascertain the credit analysis of lessee like lessee’s
ability to meet the commitment under lease, past credit record, capital strength,
availability of collateral security etc.
3. The lease agreement should be examined and the following points may be noted-
a. The description of the lessor, the lessee, the equipment and the location where the
equipment is to be installed. (The stipulated time that the equipment shall not be
removed from the described location except for repairs. For the sake of
identification, the lessor may also require plates or markings to be attached to the
equipment).
b. The tenure of lease dates of payment lease charges, deposits or advance etc. should
be noted.
c. Whether the equipment shall be returned to the lessor on termination of the
agreement and the cost shall be borne by the lessee.
d. Whether the agreement prohibits the lessee from assigning or subletting the
equipment and authorizes the lessor to do so.
4. Examine the lease proposal form submitted by the lessee requesting the lessor to provide
him the equipment under lease.
5. Examine the lease proposal form submitted by the lessee requesting the lessor to provide
him on lease the equipment.
6. Ensure that the invoice is retained safely as the lease is a long term contract.
7. Examine the acceptance letter obtained from the lessee indicating that the equipment is
received in order and is acceptable to the lessee,
8. Check the Board resolution authorizing a particular director to execute the lease
agreement has been passed by the lessee.
9. See that the copies of the insurance policies have been obtained by the lessor for his
records.

© The Institute of Chartered Accountants of Nepal 22


Paper 3 : Corporate and Other Laws

Paper 3: Corporate and Other Laws

© The Institute of Chartered Accountants of Nepal 1


Paper 3 : Corporate and Other Laws

Revision Questions

NEPAL CHARTERED ACCOUNTANT ACT, 2053 AND RULES 2061


Question No. 1
How the council of the Institute of Chartered Accountants of Nepal (ICAN) takes a decision for the report
received from Disciplinary Committee?

COMPANIES ACT, 2063


Question No. 2
Nep Green Ltd. was dealing in export of coffee to specified foreign countries. The company was willing
to purchase coffee plants in Gandaki State. The prospectus issued by the company contained some
important extracts of the director report and number of plants in Gandaki State. The report was found
untrue. Mr. Anand purchased the shares of Nep Green Ltd. on the basis of the director’s report published
in the prospectus. However, he did not suffer any loss due to purchase of such shares. Will Mr. Anand
have any remedy against the company? State also the circumstances where an director is not liable under
the Companies Act, 2063.

Question No. 3
The Board of Directors of Vishwakarma Electronics Limited consists of Mr. Ghanshyam (Managing
Director), Mr. Hritik (Director) and Mr. Indira (Chairman). The financial statements of the company were
signed by Mr. Ghanshyam and Mr. Hritik. Examine whether the authentication of financial statements of
the company was in accordance with the provisions of the Companies Act, 2063? Further, explain the
time period for preparation of financial statement as per companies’ act 2063.

Question No. 4
EFG Ltd. was incorporated on 01.04.2074 and obtained the certificate of commencement of business on
15.07.2074. No General Meeting of the company has been held till 30.04.2076. Discuss the provisions of
the Companies Act, 2063 regarding the time limit for holding the first annual general meeting of the
Company. Is there any remedy for shareholders to call annual general meeting of the company.

SECURITIES ACT, 2063


Question No. 5
What are the sources of fund of securities board ?

Question No. 6
Who shall maintain the records of securities business person? What are the matters to be contained in the
register of securities business person?

BANKS AND FINANCIAL INSTITUTIONS ACT, 2073


Question No. 7
Mr. Ram Prasad Luitel was nominated as a director of promoter group shareholder in 12th annual general
meeting of Eastern Bank Ltd. During his tenure, he faced differences with other directors of the same
group. He was removed by shareholders holding 60% shares of promoter group by passing a resolution in
the general meeting. With reference to the provision of Banks and Financial Institution Act 2073, state the
circumstances for discontinuance to hold the office of director. Advise.

© The Institute of Chartered Accountants of Nepal 2


Paper 3 : Corporate and Other Laws

Question No. 8
What are the function, duties and powers of audit committee as per Banks and Financial Institution Act,
2073?

NEPAL RASTRA BANK ACT, 2058


Question No. 9
What is the quantum and composition of security that NRB is required to maintain to issue currency notes
and coins?

INSURANCE ACT, 2049


Question No. 10
Mr. Janak Singh had insured his commercial building to indemnify it from earthquake from Future
Insurance Ltd. In the year 2076 B.S. his building was damaged due to earthquake and he submits an
application of claim from insurance company to recover the loss sustained. With reference to the
provision of Insurance Act 2049, explain the process of payment against the non-life insurance claim.

INDUSTRIAL ENTERPRISES ACT, 2073


Question No. 11
What the fixed assets to be included in the definition of fixed assets as per Industrial Enterprises Act,
2073?

Question No. 12
District Industry Development Office had conducted inspection of various industries in the month of
Falgun, 2076. Wave Textiles Pvt. Ltd., an industry has started its operation without registration under
Industrial Enterprises Act, 2073. Being the consultant of the company enlighten the management of the
company about the way out and punishment under Industrial Enterprises Act, 2073 for non-registration as
an Industry.

LABOUR ACT, 2074


Question No. 13
Mr. Lakhan one of the employee of Jupiter Cement Ltd. wants to know the provision regarding working
hours and overtime. Advise him in accordance with provision of Labour Act, 2074.
a) He is being employed 10 hours a day and 60 hours a week.
b) The company provides interval for refreshment for only 15 minutes.
c) The company is employing him on overtime without his consent.
d) Overtime remuneration is calculated and paid on the basis of his usual remuneration.

Question No. 14
What are the circumstances for warning for misconducts under Labour Act, 2074?

BONUS ACT, 2030


Question No. 15
Sindhu Energy Ltd. an profit making enterprise owned by government of Nepal, have obtained loan from
Government of Nepal amounting to NRs.3 arab in the year 2072-73. As per loan agreement, the company

© The Institute of Chartered Accountants of Nepal 3


Paper 3 : Corporate and Other Laws

is required to pay installment amounting to NRs.30 crores every year starting from the year 2075-76 as
per repayment schedule. During the year, the company could not repay the installment due to shortage of
cash. The company has earned profit during the year and has allocated bonus to staff amounting to
NRs.50 lakh. Managing Director of the company wants your know about the conditions to be fulfilled by
government owned enterprises to distribute the bonus. Advise

NEGOTIABLE INSTRUMENT ACT, 2034


Question No. 16
Discuss with reasons, whether the following persons can be called as a holder under the Negotiable
Instruments Act, 2034:
a) X who obtains a cheque drawn by Y by way of gift.
b) A, the payee of the cheque, who is prohibited by a court order from receiving the amount of the
cheque.
c) M, who finds a cheque payable to bearer, on the road and retains it.
d) B, the agent of C, is entrusted with an instrument without endorsement by C, who is the payee.
e) B, who steals a blank cheque of A and forges A’s signature.

Question No. 17
M, the holder of a bill, endorses it “without recourse” to N. N endorses it to P, P to Q, Q to R and R
endorses it again to M. Can M recover the amount of the bill from N, P, Q and R, or any of them? Discuss
with reference to the provisions of the Negotiable Instruments Act, 2034.

SOCIAL WELFARE ACT, 2049


Question No. 18
How does the meeting and decision of council take place?

WTO AND NEPAL LAWS


Question No. 19
What are the functions of world trade organization?

MULUKI DEWANI SAMHITA AIN, 2074


Question No. 20
Write short notes on Contingent Contract?

© The Institute of Chartered Accountants of Nepal 4


Paper 3 : Corporate and Other Laws

Answers/Hints

NEPAL CHARTERED ACCOUNTANT ACT, 2053 AND RULES 2061


Answer No. 1
Rule 84 of the Nepal Chartered Accountants Rules, 2061 states that, the disciplinary committee shall have
to submit a report in the council on the proceedings of the investigation carried on the complaint filed
against a member or an accounting firm in accordance with rule 81 with its recommendations.
After being reported according to sub-rule (1), the council shall make a decision on the complaint
following the procedures written below:
i) The council shall inform the complainer and order to replete the complaint if recommended to so
by the disciplinary committee.
ii) The council sending the alleged member or accounting firm a copy of the report shall, once again
grant an opportunity according to sub section (7) of section 14 of the Act to defend of his own or
by appointing a legal advisor if the disciplinary committee in its report has recommended an action.
After receiving a reply from the alleged member or accounting firm in this connection the council
can direct the disciplinary committee for re-reporting performing more investigation on the subject
if it feels essential.
iii) After the completion of the procedure according to sub rule (ii) if a member or an accounting firm
is found guilty the council shall order punishment in accordance with sub-section (5) of the section
14 of the Act.

COMPANIES ACT, 2063


Answer No. 2
Under section 24 of the Companies Act 2063, where a person has subscribed securities of a company
acting on any statement included in the prospectus which is misleading/false and has sustained any loss or
damage as a consequence thereof, the company and every director shall, be liable to pay compensation to
the person who has sustained such loss or damage.
In the present case, Mr. Anand purchased the shares of Nep Green Ltd. on the basis of the director report
published in the prospectus. Mr. Anand can claim compensation for any loss or damage that he might
have sustained from the purchase of shares. However, he did not suffer any loss due to purchase of such
shares.
Hence, Mr. Andrew will have no remedy against the company.
Circumstances when a director is not liable: Director shall not be liable for any false statement in the
prospectus under the following situations:
a) He resigns before the decision made by the company to publish the prospectus prior to the sale or
allotment of securities; or
b) He on becoming aware of any false statement in the prospectus, publishes a notice for general public on
that matter prior to the sale or allotment of securities; or
c) He proves that he did not know that the prospectus contained any false statements.

© The Institute of Chartered Accountants of Nepal 1


Paper 3 : Corporate and Other Laws

Answer No. 3
According to section 109(7) of the Companies Act, 2063, the annual financial statements and the report of
board of directors prepared pursuant to Section 109, shall be approved by the board of directors and
signed by the Chairperson of the board of directors and at least one director.

In the instant case, the financial statements have been signed by Mr. Ghanshyam and Mr. Hritik, the
directors. In view of Section 109(7) of the Companies Act, 2063, Mrs. Indira, the Chairman should be one
of the two signing directors.
The following annual financial statements shall be prepared by the board of directors of a public company
every year at least thirty days prior to the holding of its annual general meeting, and in the case of a
private company, within six months of the expiry of its financial year:
(a) Balance sheet as at the last date of the financial year.
(b) Profit and loss account of the financial year.
(c) Description of cash flow of the financial year.

Answer No. 4
According to Section 76 of the Companies Act, 2063, every public company shall be required to hold its
first annual general meeting within one year of commencement of business and thereafter it shall hold the
annual general meeting every year within six months after the expiry of its financial year.
The first financial year of EFG Ltd is for the period starting from 01.04.2074 to 32.03.2075. EFG Ltd had
obtained the certificate of commencement of business on 15.07.2074 which reflects that the first annual
general meeting (AGM) of the company should be held on or before 14.07.2075.
If any public company fails to call the annual general meeting even within three months after the expiry
of the time-limit as referred to in section 76(1), the office may give direction to call the annual general
meeting of such company.
If the company fails to call the annual general meeting even within three months after the receipt of the
direction from the office, any shareholder may make a petition, setting out the matter, to the court. Where
such petition is made, the Court may either cause to hold the annual general meeting or issue any other
appropriate order.

SECURITIES ACT, 2063


Answer No. 5
The following amounts shall be credited to the fund:-
(a) Amounts received from the Government of Nepal,
(b) Amounts obtained as grants, assistance or loans from any native, foreign or international
organizations, institutions or bodies,
Provided that prior approval of the Government of Nepal shall be obtained in receiving such grants,
assistance or loans.
(c) Amounts received by way of the license fees,
(d) Amounts received by way of the fees for registration of securities,
(e) Amounts received by way of the fees, charges, dues for transactions in securities,
(f) Amounts received by way of fines imposed by the Board,
(g) Amounts received from any other sources.

© The Institute of Chartered Accountants of Nepal 2


Paper 3 : Corporate and Other Laws

Answer No. 6
The Board shall maintain a register of all licensed securities business persons and maintain updated
records in the register.
Following are the matters to be specified in the register of securities business persons:-
a) Names and addresses of securities business persons,
b) Date of issue of license to securities business persons,
c) Types of securities business,
d) Terms and conditions specified in the license,
e) Names and addresses of the formal agents,
f) Names and addresses of managers and officers,
g) Place where documents and records pertaining to securities business are kept,
h) Names of the directors of company or body, names of company secretaries and name of each
shareholder and number of shares held by such a shareholder,
i) Such other details as the Board considers necessary and appropriate.

BANKS AND FINANCIAL INSTITUTIONS ACT, 2073


Answer No. 7
As per section 19 of Banks and Financial Institutions Act, 2073, no person shall continue to hold the
office of a director in a bank or financial institution, in any of the following circumstances:-
a) Not having qualification pursuant to section 16 or 17;
b) Not being qualified under section 18,
c) If shareholders holding at least 51% share of a group, passes a resolution in the general meeting to
remove the director who was nominated as a director from the same group.
Explanation: For the purpose of this section "Group" means group of promoter and public
shareholders.
d) If the resignation tendered by the director is accepted,
e) If he does any act prohibited by this act or by Rastra Bank from being done by as a director,
f) If the Rastra Bank directs to remove the director for being disqualified to remain in the post of
director because of his activity against the interest of depositor or bank or financial institution.
If bank or financial institution believes that any of its directors is disqualified to be in position of director
in line with this Act or Rastra Bank Act or if the director is not serving in the position, than a fifteen days'
notice must be given to Rastra Bank. Rastra Bank can give instruction to conduct necessary investigation
against director in relation being disqualified to be in the position of director.
In the given case, Ram Prasad Luitel was nominated from promoter group shareholders. While removing
the director 60% shareholders of the promoter group shareholders have consented which is in consonance
with Banks and Financial Institution Act, 2073.
Hence, the removal of Ram Prasal Luitel from the post of director of Eastern Bank Ltd. is in accordance
with Banks and Financial Institution Act, 2073.

Answer No. 8
The functions, duties and powers of audit committee as per section 61 of Banks and Financial Institutions
Act, 2073, shall be as follows:-
a) To monitor and supervise whether or not the account, budget and internal audit procedure, internal
control system is appropriate and if it is appropriate than whether or not it is implemented.

© The Institute of Chartered Accountants of Nepal 3


Paper 3 : Corporate and Other Laws

b) To conduct internal audit of account and books of bank and financial institution and to confirm
whether or not such documents are prepared accurately as per prevailing laws, regulation and
directives of the Rastra Bank.
c) To examine or caused to be examine the management and functioning of regular management and
work performance of bank or financial institution so as get assured that the prevailing laws has been
fully implemented in bank or financial institution.
d) To monitor whether or not the functions and activities carried on by bank or financial institution is as
per this act or rules formed under this act, bye-laws, rules or given directives and submit a report to
the Board of Directors on this matter.
e) To recommend the names of three auditors for appointment of external auditor.
f) To suggest on matters as asked by the Board of Directors.

NEPAL RASTRA BANK ACT, 2058


Answer No. 9
The Bank shall issue notes, only against the security, and the liability of such issued notes shall be equal
to the value of property kept as security. At least fifty percent of the property to be kept as security shall
be one or more of gold, silver, foreign currency, foreign securities, and foreign bills of exchange and the
remaining percentage shall be one or more of the coins (Mohar Double or coins of higher denomination)
the Debt Bond issued by Government of Nepal, the promissory note or bills of exchange payable in Nepal
within a maximum of eighteen months from the date of repayment by bank.
Provided that with the permission of the Government of Nepal the ratio of property kept as security may
be at least forty percent one or more of gold, silver, foreign currency, foreign securities, and foreign bills
of exchange and the remaining percentage shall be one or more of the coins (Mohar Double or coins of
higher denomination) the Debt Bond issued by Government of Nepal, the promissory note or bills of
exchange payable in Nepal within a maximum of eighteen months from the date of repayment by bank.
The valuation of property to be kept as security shall be made as follows:-
(a) The price of gold at the rate fixed by Government of Nepal on the recommendation of the Board;
(b) The price of silver at the rate deemed appropriate by the Board;
(c) The foreign currencies at the exchange rate fixed by the Bank;
(d) The Debt Bond issued by Government of Nepal, the foreign securities and Bills of Exchange at the
rate deemed appropriate by the Board on the basis of market rates;
(e) Coins at the rate of face value.

INSURANCE ACT, 2049


Answer No. 10
As per Rule 32 of Insurance Rules, 2050, if any claim has to be made under the Insurance Policy by an
Insured who has taken up a Non-Life Insurance Policy, the Insured shall submit an application to the
Insurer stating all the details relating to it. On receipt of an application of the Insured for the payment
against the Insurance claim of the Non-Life Insurance, the Insurer shall immediately designate a Surveyor
to make inquiry, if necessary.
The Surveyor deputed shall make necessary inquiry and determine the liability of the Insurer within 15
days. Surveyor shall submit a report to the Insurer including the comprehensive details and inform the

© The Institute of Chartered Accountants of Nepal 4


Paper 3 : Corporate and Other Laws

insured relating to it, by mentioning the amount to be received by the Insured subject to the terms and
conditions and facilities of the Insurance Policy.
The Insurer shall determine the liability and shall provide the payment against the claim of the Non-Life
Insurance to the Insured generally within 35 days from the submission of the report by the surveyor.
In the given case, Janak Singh should file a application as per Insurance Act, 2049 with the insurer statinf
all the details relating to it and the insurer shall determine the liability and provide the claim for loss of
building from earthquake.

INDUSTRIAL ENTERPRISES ACT, 2073


Answer No. 11
The fixed asset of an industry shall consist of the following assets:-
(a) Land, physical infrastructure constructed in space, under-ground, water or under water,
(b) Physical infrastructures above land (such as: sewerage, internal road, water distribution system,
drinking water infrastructure),
(c) Factory Office, factory building, building or godown,
(d) Residential buildings constructed for employee or workers,
(e) Electricity distribution and equipment’s related with such distribution,
(f) Machinery, equipment, tools and reserved fittings,
(g) Means of transportation,
(h) Electrical equipment and office equipment,
(i) Furniture, fixture,
(j) Communication system and related system & equipments.
In addition to the assets referred above, following pre-operative expenses and expenses incurred during
different stages of construction shall be capitalized as fixed assets:
a) Capitalized technical consultancy and supervision expenses;
b) Pre-investment and pre-operation costs;
c) Capitalized interest cost during the construction period.

Answer No. 12
As per section 41 of Industrial Enterprises Act, 2073, if it is found that any person operates any industry
without registration under this act or previous act on inspection, inquiry and investigation done by district
industry development office. Then the office shall prepare a detailed report about such violation and send
it to the ministry through concerned department for punishments and such information shall also be given
to district administration office. Ministry may provide the maximum time of 3 months to get such
industry registered and produce the evidence of registration if it has not been registered.
Following are the punishments for non- registration as an Industry:
a) Order to close down such industry;
b) Monetary Punishment:
Micro Industries: 10,000 to 25,000
Cottage Industries: 25,000 to 1,00,000
Small Industries: 25,000 to 1,00,000
Medium Industries: 1,00,000 to 3,00,000
Large Industries: 1,00,000 to 3,00,000

© The Institute of Chartered Accountants of Nepal 5


Paper 3 : Corporate and Other Laws

In the given case, Wave Textiles Ltd. had started its operation without making registration under
industrial enterprises act, 2073. Hence, the company will be punished under section 41(3) of the Industrial
Enterprises Act, 2073 if the evidence of registration have not been submitted within the time limit
prescribed by the ministry.

LABOUR ACT, 2074


Answer No. 13
Following are the provision regarding working hours and overtime to be known by Lakhan as per Labour
Act, 2074:-
a) No workers shall be employed to work more than 8 hours a day and 48 hours a week by an
employer.
b) Workers shall be provided with half an hour rest after five hours of continuous work.
c) No worker shall be compelled to work more than 8 hours a day and 48 hours a week by the
employer. However, the workers may be made to work overtime if the non-completion of the work
may have an adverse effect on the life, health and safety of any person or serious harm or loss may
be caused to the employer or any other person or workers.
d) When requiring any worker to work overtime by the employer, the worker shall be paid
remuneration at a rate of one and half times of the basic salary that the worker receives during
regular hours of work. However, nothing written in this section shall act as a constraint to provide
benefits as determined by a collective agreement or benefits as mentioned in the employment
contract in case of managerial workers in lieu of payment for overtime work performed.

Answer No. 14
Warning may be given for the following misconducts by any worker:
a) act of being absent in the work without approving the leave;
b) act of leaving the workplace without taking permission of the management;
c) act of reaching the workplace frequently late without taking permission of the management;
d) act of disobeying the order given by the employer or any employee;
e) act of doing any other misconduct as specified in the by-law.

BONUS ACT, 2030


Answer No. 15
As per Rule 6(1) of Bonus Rules, 2039, every establishment owned by Government of Nepal with the
objective of making profit, shall be required to allocate 5% of the net profit made in every fiscal year as
bonus.
As per Section 5(3B) of Bonus Act, 2030, entity owned by government of Nepal, can distribute bonus
only in following circumstances:
a) Expenses booked on accrual basis in earlier financial year should be paid in next financial year;
b) The amount of contingent liabilities should be determined for each financial year and separate fund
must be created for such amount;
c) Loan taken from Nepal government or on the guarantee of Nepal government as mentioned in loan
deed should be paid according to repayment schedule.
In the given case Sindhu Enery Ltd., an profit making enterprise owned by government of Nepal shall
have to fulfill the conditions laid down in section 5(3B) of bonus act. During the financial year, the

© The Institute of Chartered Accountants of Nepal 6


Paper 3 : Corporate and Other Laws

company has earned the profit and had allocated 50 lakh as bonus amount. Further, the company has not
paid the loan installment amount during the year as per repayment schedule, to be paid as agreed between
the company and government of Nepal.
Hence, I will advise the managing director to pay the loan installment as per repayment schedule and
distribute bonus to its employees as prescribed by bonus act.

NEGOTIABLE INSTRUMENT ACT, 2034


Answer No. 16
As per section 2(k) of the Negotiable Instruments Act, 2034, "Holder" means a person entitled to the
possession of negotiable instrument in his own name and to receive the amount due on it.The holder may
transfer the negotiable instrument. On applying the above provision in the given cases-
a) Yes, X can be termed as a holder because he has a right to possession and to receive the amount
due in his own name.
b) No, A is not a holder, because to be called as a holder he must be entitled not only to the
possession of the instrument but also to receive the amount mentioned therein.
c) No, M is not a holder of the Instrument though he is in possession of the cheque, so is not entitled
to the possession of it in his own name.
d) No, B is not a holder. While the agent may receive payment of the amount mentioned in the
cheque, yet he cannot be called the holder thereof because he has no right to sue on the
instrument in his own name.
e) No, B is not a holder because he is in wrongful possession of the instrument.

Answer No. 17
As per Section 35 of the Negotiable Instruments Act, 2034, the endorser can endorse by clearly stating the
matter that his liability shall not persist on the Negotiable Instrument. In such a case, the endorser relieves
himself from the liability to all subsequent endorsees. By adding the words without recourse, the endorser
declines to accept any liability on the instrument of any subsequent parties.
If a negotiable instrument is negotiated by the holder, but the endorser again becomes the holder of such
negotiable instrument then the holder cannot enforce payment against an intermediate party to whom he
was previously liable. However, the holder can sue all the prior parties (including all intermediate parties
to whom he was previously liable), if he had made sans recourse or without recourse endorsement.
In the given case, M had endorsed the negotiable instrument to N by using the words “without recourse”
and later he becomes the holder of the negotiable instrument. Since, M has made sans recourse
endorsement while endorsing the instrument to N, M can therefore enforce payment against all
intermediate parties, i.e., N, P, Q and R because he was not liable to them as a prior party.

SOCIAL WELFARE ACT, 2049


Answer No. 18
The meeting of the Council shall take place in the date, time and place specified by the chairperson at
least twice a year. The quorum for the meeting of the Council shall be at least fifty percent of total
number of the member. The chairperson shall preside the Council meeting in case of absence of the
chairperson, vice-chairperson, and if both absent, the member selected among themselves shall preside
over the meeting. The opinion of the majority shall be credible in the meeting of the Council and in case
of tie chairperson shall give the casting vote. The decision of the Council shall be authenticated by the

© The Institute of Chartered Accountants of Nepal 7


Paper 3 : Corporate and Other Laws

member secretary. Other procedures relating to the meeting of the Council shall be determined by the
Council.

WTO AND NEPAL LAWS


Answer No. 19
The major functions of the WTO as envisaged in its Charter are as follows:
a) Administer and implement the trade agreements;
b) Act as a forum for multilateral trade negotiations;
c) Seek to resolve trade disputes;
d) Oversee national trade policies;
e) Cooperate with other international institutions involved in global economic policy making;
f) Maintain trade related database. Members are required to notify in detail various trade measures and
statistics;
g) Act as watchdog of international trade, constantly examining the trade regimes of individual
members;
h) Act as a management consultant for world trade;
i) Provide technical assistance and training for developing countries.

MULUKI DEWANI SAMHITA AIN, 2074


Answer No. 20
In case a contract has been concluded to performing or not to perform any work if any event happens in
the future, the contract shall not create any liability until such event takes place.
In case a contract has been concluded subject to the condition that it shall be deemed to have been
concluded in case person performs any specified work in the future, no liability shall be deemed to have
created from that contract if such person does anything in such a manner as not to perform that work or
acts in such a manner that the work cannot be performed.
In case a contract has been concluded to perform or not to perform any work if any uncertain event does
not happen in the future, liability under that contract shall emerge only after the happening of that event
becomes impossible.
In case a contract has been concluded with a provision to perform or not to perform any work if any event
happens within a specified period in the future, the contract shall be deemed to have become invalid after
the happening of that event becomes impossible within the specified period or after the expiry of that
period.
In case a contract has been concluded with a provision to perform or not to perform any work if any event
does not happen within a specified period in the future, liability under such contract shall emerge if that
event does not happen within that period or if it becomes certain that the event will not happen within that
period.

© The Institute of Chartered Accountants of Nepal 8


Paper 4: Financial Management

Paper 4: Financial Management

© The Institute of Chartered Accountants of Nepal 1


Paper 4: Financial Management

Revision Questions
STRATEGIC FINANCE AND POLICY

Capital Structure

Question No. 1

High Trek Resorts Limited is considering various levels of debt in order to finance its bungee-
jumping project. At present it has no debt, and a total market capitalization of Rs. 15 million. By
undertaking financial leverage, it believes that it can achieve a tax advantage equal to 20 percent
of the market value of the debt. However, the company is concerned with bankruptcy and agency
costs as well as with lenders increasing their required interest rate if the company borrows too
much. The company believes that it can borrow up to Rs. 5 million without incurring any of these
additional costs. However, each additional Rs. 5 million increment in borrowing is expected to
result in these three costs being incurred. Moreover, these costs are expected to increase at an
increasing rate with financial leverage. The present value cost is expected to be the following for
various levels of debt:
Debt (Rs.in millions) 5 10 15 20 25 30
Present value cost of bankruptcy, agency, and
0 0.6 1.2 2 3.2 5
increased interest rate (Rs.in millions)

Is there an optimal amount of debt for the company? If so, what is it?

Operating and Financial Leverage (Gearing)

Question No. 2

Shiva Cement Company Limited has Rs.7.4 million in long-term debenture having the following
payment schedule:

Rs.
15% debenture, Rs.100,000 payable annually in principal 2,400,000
13% debenture, Rs.150,000 payable annually in principal 3,000,000
18% debenture, interest only until maturity in 10 years 2,000,000
7,400,000

The company’s ordinary equity has a book value of Rs. 7.3 million and a market value of Rs.8
million. The corporate income tax rate is 50 percent. It has expected EBIT of Rs. 2 million with a
standard deviation of Rs. 1.5 million. The average debt-to-equity ratio of other companies in the
industry is 0.47.

© The Institute of Chartered Accountants of Nepal 2


Paper 4: Financial Management

a. Determine the interest coverage and the debt-service coverage ratios for the company.
b. What are the probabilities that these two ratios will go below 1:1?
c. Does the company have too much debt?

Cost of Capital

Question No. 3

Sanima Company Limited has the following book value capital structure:

Equity capital (3 million shares, Rs.100) Rs.300 million


Preference capital, 15 percent (1,000,000 shares, Rs.100) Rs. 100 million
Retained earnings Rs. 100 million
Debentures 11 percent (250,000 debentures, Rs.1000) Rs .250 million
Term loans, 13 percent Rs. 300 million
Rs.1050 million

The next expected dividend per share is Rs.40. The dividend per share is expected to grow at the
rate of 15 percent. The market price per share is Rs.800. Preference stock, redeemable after 6
years, is currently selling for Rs.110 per share. Debentures, redeemable after 6 years, are selling
for Rs.1020 per debenture. The tax rate for the company is 25 percent.

(a) Calculate the average cost of capital using


(i) Book value proportions, and
(ii) Market value proportions

(b) What is the marginal cost of capital schedule for the company if it raises Rs.450
million next year, given the following information:
(i) the amount will be raised from equity and term loans in the proportion 2:1.
(ii) the company expects to retain Rs.80 million earnings next year;
(iii)the debt capital raised by way of term loans will cost 11 percent for the first
Rs.100 million and 12 percent for amounts thereafter.

Analysis of financial statements

Question No. 4

Trisara Private Limited has had the following financial position statements for the past four years:
Rs.in thousands

© The Institute of Chartered Accountants of Nepal 3


Paper 4: Financial Management

Assets 2073 2074 2075 2076


Cash 428 186 84 76
Sundry Debtors 2,426 3,138 3,692 5,124
Inventories (Trading) 4,204 5,786 7,356 8,522
Net Fixed Assets 4,438 4,692 4,776 5,384
Total Assets 11,496 13,802 15,908 19,106
Liabilities and Equity
Sundry Creditors 2,262 3,156 3,696 5,936
Other payables 1,000 1,300 1,500 1,500
Accruals 1,312 1,722 2,578 3,486
Long term debt 1,000 1,600 1,600 1,600
Paid up capital 400 400 400 400
Retained earnings 5,522 5,624 6,134 6,184
Total liabilities and equity 11,496 13,802 15,908 19,106

Using index analysis, explain the major problems in the company’s financial condition?

Time Value of Money

Question No. 5

In 2000, Mr.Deepak bought a 10 Ropanis of land on the Lalitpur District for Rs. 58 lakh. Deepak,
who is considered a shrewd investor, made many such purchases. How much would his
descendants will have in 2069, if instead of buying the land, he had invested the Rs.58 lakh at 5
percent compound annual interest?

Risk and return

Question No. 6

Srijana Maharjan invests the following sums of money in equity of following microfinances having
expected returns as follows:
Investment
Stock( NEPSE Index Symbol) Expected Return
(Rs.’000)
Omni Microfinance (OMF) 6,000 14%
Ambe Microfinance (AMF) 11,000 16%
Ekta Microfinance (EMF) 9,000 17%
Sisne Microfinance (SMF) 7,000 13%
Indra Microfinance (IMF) 5,000 20%
Union Microfinance (UMF) 13,000 15%

© The Institute of Chartered Accountants of Nepal 4


Paper 4: Financial Management

Nepal Microfinance (NMF) 9,000 18%

a. What is the expected return on her portfolio?


b. What would be her expected return if she quadrupled her investment in Indra Microfinance,
while leaving everything else the same? Does her expected return increases or decreases and
comment why?

VALUATION OF SECURITIES

Bond Valuation

Question No. 7

A Rs. 1,000-face-value bond has a current market price of Rs. 935, an 8 percent coupon rate, and
10 years remaining until maturity. Interest payments are made semiannually.
a. State whether the yield to maturity is above or below the coupon rate. Why?
b. What is the implied market-determined semiannual discount rate on this bond?
c. Using your answer to Part (b), what is the bond’s (i) (nominal annual) yield to maturity? (ii)
(effective- annual) yield to maturity?

Equity Share Valuation

Question No. 8

You are a stock analyst and have been appointed to analyze the stock of two different firms, namely
firm A and firm B. You found out that the stocks of firms A and B are considered to be equally
risky. Investors expect the share of firm A – the firm which does not plan to pay dividend -- to be
worth Rs 100 next year. From the share of firm B, too, investors expect a pay-off of Rs.100 – Rs.10
by way of dividend and Rs.90 by way of share price a year from now. Currently, dividends are
taxed at 25 percent and capital gains at 12 percent.

Using your findings, you are now required to determine the current price of the shares of A and B,
if each of them offers an expected post tax rate of 18 percent?

WORKING CAPITAL MANAGEMENT AND FINANCIAL FORECASTING

Question No. 9

The present sales of Shikhar House are Rs.100 million. The firm classifies its customers into 3
credit categories: Gold, Silver, and Standard. The firm extends unlimited credit to customers in

© The Institute of Chartered Accountants of Nepal 5


Paper 4: Financial Management

category Gold, limited credit to customers in category Silver, and no credit to customers in
category Standard. As a result of this credit policy, the firm is foregoing sales to the extent of
Rs.10 million to customers in category Silver and Rs.20 million to customers in category
Standard. The firm is considering the adoption of a more relaxed credit policy under which
customers in category Silver would be extended unlimited credit policy and customers in
category Standard would be provided limited credit. Such relaxation would increase the sales by
Rs.30 million on which bad debt losses would be 10 percent. The contribution margin ratio for the
firm is 20 percent, the average collection period is 45 days, and the cost of capital is 16 percent.
The tax rate for the firm is 35 percent.

The firm is not sure on what will be the effect of relaxing the credit policy on its residual income.
You, as a financial consultant of the firm, is required to show the effect of relaxing the credit policy
on the residual income of the firm.

Question No. 10

Namaste Private Limited issues cheques of Rs.10, 000 daily to various parties and it receives
cheques of Rs.30, 000 from various parties daily. As per past history, it takes 6 days for issued
cheques to be cleared and it takes 4 days for received cheques to be realized. Assume that there is
a balance of Rs.80, 000 to begin with; show the balance in the book of the firm and the books of
the bank. What will be the balance in the steady state situation?

Question No. 11

Milan Company Ltd. currently has total assets of Rs. 3.2 million, of which current assets comprise
Rs.0.2 million. Sales are Rs.10 million annually, and the before-tax net profit margin is 12 percent.
The company currently has no interest-bearing debt. Given renewed fears of potential cash
insolvency, an overly strict credit policy, and imminent stock outs, the company is considering
higher levels of current assets as a buffer against adversity. Specifically, levels of Rs.0.5 million
and Rs.0.8 million are being considered instead of the Rs.0.2 million presently held. Any additions
to current assets would be financed with new equity capital.

Required:
a. Determine the total asset turnover, before-tax return on investment, and before-tax net profit
margin under the three alternative levels of current assets. Also, explain the conclusion.
b. If the new additions to current assets were financed with long-term debt at 15 percent interest,
what would be the before-tax interest “cost” of the two new policies? Also, explain in brief could
this cost be reduced?

© The Institute of Chartered Accountants of Nepal 6


Paper 4: Financial Management

Question No. 12

Birgunj Industries manufactures a range of electronic products. Component RX is one of the major
component required to produce some of the electronic products. The supplier of component RX
has informed Birgunj Industries that it will offer a quantity discount of 10% if Birgunj Industries
places an order of 1,000 components or more at any one time.
Details of component RX are as follows:
Cost per component before discount Rs.200
Annual purchases 150,000 components
Ordering costs Rs.36, 000 per order
Holding costs Rs.30,000 per component per annum

Required:
(i) Calculate the total annual cost of holding and ordering inventory of component RX
using the economic order quantity and ignoring the quantity discount.
(ii) Calculate whether there is a financial benefit to Birgunj Industries from increasing the
order size to 1,000 components in order to qualify for the 10% quantity discount.

Question No. 13

You have been working as finance officer in Financial Planning Department of Mega Limited.
Recently, a BBA fresher has joined to the department and has been assigned to prepare projected
financial statement (along with fund estimation) assignment by CFO with following information:
Balance sheet of Mega Limited
As of July mid, 2019
Equity and liabilities Rs.( in million) Assets Rs.( in million)
Share capital 4,200 Fixed assets 8,870
Retained Earnings 2,480 Inventories 3,480
Term Loans 3,920 Receivables 2,580
Short-term Borrowings 2,490 Cash 180
Accounts Payable 1,240
Provisions 780
15,110 15,110

The sales for the year ending on July mid, 2019 were Rs.31, 410 million. Its profit margin on sales
was 7 percent and its dividend payout ratio was 50 percent. The tax rate was 34 percent. Mega
Limited expects its sales to increase by 30 percent in the next year. The ratio of assets to sales and
spontaneous current liabilities to sales would remain unchanged. Likewise the profit margin ratio,
the tax rate, and the dividend payout ratio would remain unchanged. Assume that the external

© The Institute of Chartered Accountants of Nepal 7


Paper 4: Financial Management

funds requirement would be raised from term loans and short-term bank borrowings in the
ratio 1:2
With the above information, he is confused on how to prepare the projected financials for next
year and come with you for your some help .Now you are required to help him on followings:
a. Estimate the external funds requirement for the next year.
b. Prepare the statements: (i) projected balance sheet and (ii) projected profit and loss account.

Distribution policy

Question No. 14

Capital International Company has an all-ordinary-equity capital structure. It has 200,000 shares
of Rs.2 par value ordinary shares outstanding. When company’s founder, who was also its research
director and most successful inventor, retired unexpectedly last year, the company was left
suddenly and permanently with materially lower growth expectations and relatively few attractive
new investment opportunities. Unfortunately, there was no way to replace the founder’s
contributions to the company. Previously, the company found it necessary to plow back most of
its earnings to finance growth, which averaged 12% per year. Future growth at a 6% rate is
considered realistic, but that level would call for an increase in the dividend payout. Further, it
now appears that new investment projects with at least the 14% rate of return required by
company’s stockholders would amount to only Rs. 800,000 for 2019 compared to a projected Rs.
2,000,000 of net income. If the existing 20% dividend payout was continued, retained earnings
would be Rs. 1.6 million in 2019; but as noted, investments that yield the 14% cost of capital
would amount to only Rs.800,000. The one encouraging point is that the high earnings from
existing assets are expected to continue, and net income of Rs.2 million is still expected for 2019.
Given the dramatically changed circumstances, management is reviewing the dividend policy as
follows:

a. Assuming that the acceptable 2019 investment projects would be financed entirely by earnings
retained during the year and the company uses the residual dividend model, calculate DPS in 2019.
b. What payout ratio does your answer to Part (a) imply for 2019?
c. If a 60% payout ratio is maintained for the foreseeable future, what is your estimate of the
present market price of the share? How does this compare with the market price that should have
prevailed under the assumptions existing just before the news about the founder’s retirement? If
these two values are different, comment on why?
d. What would happen to the share price if the old 20% payout was continued? Assume that if this
payout is maintained, the average rate of return on the retained earnings will fall to 7.5%.

© The Institute of Chartered Accountants of Nepal 8


Paper 4: Financial Management

Mutual Fund

Question No. 15
Civil Mutual Fund has introduced a scheme called Dividend Reinvestment Plan. The face value of
a unit is Rs.10 per unit. On Shrawan 1, 2071, Mr. Raj invested Rs. 200,000 in this plan when the
Net Asset Value (NAV) was Rs.38.20 per unit. The plan matured on Magh 1, 2076. The following
are the particulars of the dividend declared over the period:
Date Dividend (%) NAV (Rs. per unit)
Poush 30, 2071 10 39.10
Poush 30 , 2073 15 44.20
Poush 30, 2074 13 45.05
Poush 30, 2075 16 44.80
Magh 1, 2076 40.40
Capital Gain tax on maturity is 5%

Ignoring other transaction charges, what is the effective yield per annum on the above plan?

Capital Market

Question No. 16

Pokhara Bank Limited must have to issue debentures in order to meet recent regulatory
requirement. It explores two options, either a public issue of debenture or the private placement of
this debenture with an insurance company. In both cases, the funds needed are Rs.60 million for 6
years with no principal repayment until the final maturity of the debentures. With a public issue,
the interest rate will be 15 percent, the underwriting spread will be Rs.10 per debenture, and the
debenture will be priced to the public at Rs.1, 000 per debenture. To realize Rs.60 million in
proceeds, the company will need to issue some additional debentures to offset the spread. Legal,
printing, and other initial costs come to Rs.1, 950,000 with the public issue. For the private issue,
the interest rate will be 15.5 percent, and initial costs will come to only Rs.200, 000.

Ignoring other regulatory requirements, you are required to answer following questions:
a. Ignoring the time value of money, which method has the higher total costs over the 6 years? If
we considered the time value of money, which method would be better?
b. What if the maturity were 12 years and all other things remaining the same?

© The Institute of Chartered Accountants of Nepal 9


Paper 4: Financial Management

CAPITAL INVESTMENT DECISION

Internal Rate of Return

Question No. 17

Quest Drugs Limited is the leader in drug industry of Nepal. It manufactures a range of drugs and
pharmaceutical ingredients. It is considering to develop a new bulk drug called CIVID 2020.
This project will require an outlay of Rs.40 million in plant and machinery right in the beginning.
For this, it is planning to raise loan from its regular banking partner, Commercial Bank Limited
Nepal. However, the credit arrangement between the Bank and the company stipulates that any
new project will be financed by company’s equity and bank’s term loan in equal proportions. The
term loan will carry an interest of 14 percent per annum and will be repayable in 4 equal annual
instalments, the first instalment falling due at the end of year 1.The minimum IRR requirement of
the Bank for any new project is very high which can be fixed up to 20 percent at the Bank’s
discretion.

You, after qualifying as Chartered Accountant, have recently joined Quest Drugs Limited as a
finance officer and you directly report to CFO of the company. Your CFO asked you to make
preliminary analysis of project CIVID 2020 to ascertain the feasibility of credit arrangement made
with the Bank. After discussing with various departments namely marketing, research and human
resources, you have gathered the following information:

The project has an economic life of 5 years. It would generate a revenue of Rs.50 million in year1
which will rise by Rs.10 million per year for the following two years. Thereafter, revenues will
decline by Rs.10 million per year for the remaining two years. Operating costs (costs before
depreciation, interest, and taxes) will be 60 percent of revenues. The project is expected to erode
the revenues of an existing bulk drug. Due to this erosion there will be a loss of Rs.4 million per
year by way of contribution margin for 5 years. While there may be some other impacts as
well, they may be ignored in the present analysis.

For tax purposes, the depreciation rate will be 15 percent as per the written down value method.
The net salvage value of plant and machinery after 5 years is expected to be Rs.20 million. The
net working capital requirement will be 20 percent of revenues.

You can assume that the investment in net working capital will be made right in the beginning of
each year and the same will be fully financed by one of its sister company in the way of interest
free working capital advance. At the end of 5 years the working capital is expected to be liquidated
at par. The tax rate is 30 percent.

© The Institute of Chartered Accountants of Nepal 10


Paper 4: Financial Management

Required
1. Based on the above information, you are required to advise on the feasibility of the project on
the basis of IRR.

Replacement Decision

Question No. 18

Synergy Ltd. has just installed Machine- M at a cost of Rs.210, 000. The machine has a five year
life with no residual value. The annual volume of production is estimated at 150,000 units, which
can be sold at Rs.6 per unit in the first two years and at Rs.7,8 and 9 in the third, fourth and fifth
years. The first year’s operating costs are estimated at Rs.200, 000 (excluding depreciation) at this
output level. Fixed costs are estimated at Rs.3 per unit for the same level of production. The second
year’s cost will be the same as in the first year. Thereafter, costs (operating and fixed) will increase
over the first year’s cost by 10%, 20% and 25% respectively in the third, fourth and fifth years.
Synergy Ltd. has just come across another model called Machine-N capable of fiving the same
output at the same fixed and operating costs as in the first year of Machine- M. There
will be no change over the first year’s costs in the next four years also. Capital cost of this machine
is Rs.250, 000 and the estimated life is five years with nil residual value. The company has an
offer for sale of Machine- M at Rs.110,000. But the cost of dismantling and removal will
amount to Rs.40,000. As the company has not yet commenced operations, it wants to sell
Machine- M and purchase Machine- N.
Synergy Ltd. will be a zero-tax company for seven years in view of several incentives and
allowances available. The cost of capital is 15%.
(i) Advise whether the company should opt for the replacement.
(ii) Will there be any change in your view, if machine-M has not been installed, but the
company is in the process of selecting one or the other machine?

Question No. 19

Define briefly financial management. Explain MM theory on dividend policy along with its
assumptions and criticisms.

Question No. 20

Distinguish between:
a) Foreign Direct Investment and Portfolio Investment
b) Money Interest Rate and Real Interest Rate
c) Absolute Interest Rate Risk and Yield Curve Risk
d) Operational restructuring and financial restructuring

© The Institute of Chartered Accountants of Nepal 11


Paper 4: Financial Management

e) Bank rate and repo rate


f) Capital structure and financial structure
g) Annuity and Perpetuity

Question No. 21

Write short notes on:


a) Gold deposit accounts in the bank
b) Book building
c) Marketability
d) Financial Modeling
e) Leverage effect
f) Limitations of IRR analysis
g) DuPont analysis

© The Institute of Chartered Accountants of Nepal 12


Paper 4: Financial Management

Answers/Hints

STRATEGIC FINANCE AND POLICY

Capital Structure

Answer No. 1

Rs. in millions
(1) (2) (3) (4) Value of
Level Firm PV PV of firm
Of Value of bankruptcy , (2)+(3)-(4)
Debt (unlevered) Tax benefit agency &
(1)*20% increased costs
0 15 0 0.0 15.0
5 15 1 0.0 14.0
10 15 2 0.6 16.4
15 15 3 1.2 16.8
20 15 4 2.0 17.0
25 15 5 3.2 16.8
30 15 6 5.0 16.0

The market value of the company is maximized when the debt level is Rs. 20 million.

Operating and Financial Leverage (Gearing)

Answer No. 2

a.
Total annual interest is as follows:
15% of Rs. 2.4 million = 360,000
13% of Rs. 3.0 million = 390,000
18% of Rs. 2.0 million = 360,000
1,110,000

Interest coverage ratio =Rs.2, 000,000/ Rs. 1,110,000 =1.80 times

Total annual principal payments =Rs.100, 000 +Rs.150, 000=Rs.250, 000

© The Institute of Chartered Accountants of Nepal 1


Paper 4: Financial Management

Rs. 2,000,000
Debt service coverage ratio =
Rs. 1,110,000 + [Rs. 250,000/(1 − 0.50)]
= 1.24 times

b.
Required deviation of EBIT from its mean value before ratio in question becomes 1:1:

Interest coverage: Rs. 1,110,000 − Rs.2, 000,000 = −Rs. 890,000

Debt-service coverage: Rs.1, 610,000 − Rs.2, 000,000 = −Rs.390, 000

Standardizing each deviation from the mean produces the following Z-scores:
−890,000
Interest coverage = = −0.593 (left of the mean)
Rs. 1,500,000

−390,000
Debt service covergae ratio = = −0.260 (left of the mean)
Rs. 1,500,000

From normal distribution table, it can be found that these proportions (Z scores) correspond to
probability of approximately 28 percent and 40 percent respectively. Hence, for interest coverage
and debt-service coverage ratios less than 1:1, these probabilities are approximately 28 percent
and 40 percent respectively. These probabilities assume that the distribution of possible EBITs is
normal.

c.
There is a substantial probability, 40 percent that the company will fail to cover its interest and
principal payments. Its debt ratio (using either book or market values) is much higher than the
industry norm of 0.47. Based on given information, it would appear that the company has too much
debt. However, other factors, such as liquidity, may mitigate this conclusion.

Cost of Capital

Answer No. 3

(a)
(i) The cost of equity and retained earnings is:
D1 40
re = +g= + 0.15 = 0.20 = 20%
Po 800

The cost of preference capital is:

© The Institute of Chartered Accountants of Nepal 2


Paper 4: Financial Management

15 + (100 − 110)/6
rp = = 12.70%
(100 + 110)/2

The pre-tax cost of debentures is:

110 + (1000 − 1020)/6


rd = = 10.56%
(1000 + 1020)/2

The post-tax cost of debentures is:

= 10.56% (1-tax rate) = 10.56 % (1 – 0.25) = 7.92 %

The post-tax cost of term loans is

= 13% (1-tax rate) = 13% (1 – 0.25) = 9.75 %

(i) Average cost of capital using book value proportions:

Source of Capital Component Book Value Rs. Book Value


Cost in Million Proportion (1) * (3)
(1) (2) (3)
Equity capital 20% 300 0.286 5.71%
Preference capital 12.70% 100 0.095 1.21%
Retained earnings 20% 100 0.095 1.90%
Debentures 7.92% 250 0.238 1.89%
Term loan 9.75% 300 0.286 2.79%
1,050 1.000 13.50%

(ii) Average cost of capital using market value proportions:

Source of Capital Component Market Value Market Value


Cost Rs. in Million Proportion (1) * (3)
(1) (2) (3)
Equity capital &
20% 2,400 0.783 15.66%
Retained earnings
Preference capital 12.70% 110 0.036 0.46%
Debentures 7.92% 255 0.083 0.66%
Term loan 9.75% 300 0.098 0.95%
3,065 1.000 17.73%

© The Institute of Chartered Accountants of Nepal 3


Paper 4: Financial Management

(b)
The Rs.450 million will be raised in the following way:
Retained earnings Rs.80 million
Additional equity Rs.220 million (450*2/3 -80)
Debt Rs.150 million (450*1/3)

The first batch will consist of Rs.80 million of retained earnings and Rs.40 million of debt costing
20 percent and 11(1-0.25) =8.25 percent respectively.
The second batch will consist of Rs.120 million of additional equity and Rs.60 million of debt at
20 percent 8.25 percent respectively.
The third chunk will consist of Rs.100 million of additional equity and Rs.50 million of debt
costing 20 percent and 12(1-0.25) = 9 percent respectively.

The marginal cost of capital in the chunks will be as:


First batch: (2/3) x 20% + (1/3) x 8.25% = 16.08 %
Second batch: (2/3) x 20% + (1/3) x 8.25% = 16.08 %
Third batch: (2/3) x 20% + (1/3) x 9.00% = 16.33%

The marginal cost of capital schedule for the firm will be as follows:
Range of total financing Weighted marginal cost of capital
(Rs.in million) (%)
0 - 300 16.08%
301-450 16.33%

Analysis of financial statements

Answer No. 4

Index analysis has been made taking the base year as 2073:
Index 2073 = Data 2073/ Data 2073
Index 2074 = Data 2074/ Data 2073
Index 2075 = Data 2075/ Data 2073
Index 2076 = Data 2076/ Data 2073

Assets 2073 2074 2075 2076


Cash 100.00 43.46 19.63 17.76
Sundry Debtors 100.00 129.35 152.18 211.21
Inventories(Trading) 100.00 137.63 174.98 202.71
Net Fixed Assets 100.00 105.72 107.62 121.32
Total Assets 100.00 120.06 138.38 166.20

© The Institute of Chartered Accountants of Nepal 4


Paper 4: Financial Management

Liabilities and equity


Sundry Creditors 100.00 139.52 163.40 262.42
Other payables 100.00 130.00 150.00 150.00
Accruals 100.00 131.25 196.49 265.70
Long term debt 100.00 160.00 160.00 160.00
Paid up capital 100.00 100.00 100.00 100.00
Retained earnings 100.00 101.85 111.08 111.99
Total liabilities and equity 100.00 120.06 138.38 166.20

Conclusions:
In the last three years, the company has increased its sundry debtors and inventories rather
dramatically. There arises doubt as to recoverability of sundry debtors as well as sale-ability of
inventory due to increased chunk of sundry debtors and inventories over the years. While net fixed
assets jumped in 2076, changes were only modest in 2074 and 2075. The basic problem is that
retained earnings have grown at only a very slow rate, almost all of which occurred in 2075. This
is due to inadequate profitability, excessive dividends, or both. While the company increased its
long-term debt in 2074, it has not done so since. The burden of financing has fallen on short term
financing, that is, sundry creditors, other payables and accruals, together with drawing down the
cash position. The question would be whether creditors and payables are past due and also whether
employees are being paid on time. It is clear that the company cannot continue to expand its assets
without increasing its equity base in a significant way.

Time Value of Money

Answer No. 5

In FVIF Table, we won’t find the FVIF of Re.1 in 69 years at 5 percent. But, we can find the FVIF
of Re. 1 in 50 years – Rs. 11.467 –and the FVIF of Re. 1 in 19 years – Rs. 2.527. So we can express
the problem as follows:
Future Value (FV) 69 = Present Value (PV) × {1 +interest rate (i)} 69
=PV× (1 +i ) 50+19
=PV× (1 +i)50×(1 +i)19
=Rs. 58 lakh×11.467 ×2.527
=Rs.58 lakh ×28.977
=Rs.1, 680 .67 lakh

Risk and return

Answer No. 6
a.

© The Institute of Chartered Accountants of Nepal 5


Paper 4: Financial Management

Expected Weighted
Stock Investment Weight
Return Return
( NEPSE Index Symbol) (Rs.’000) (1)
(2) (1 *2)
Omni Microfinance (OMF) 6,000 0.10 14% 1.40%
Ambe Microfinance (AMF) 11,000 0.18 16% 2.93%
Ekta Microfinance (EMF) 9,000 0.15 17% 2.55%
Sisne Microfinance (SMF) 7,000 0.12 13% 1.52%
Indra Microfinance (IMF) 5,000 0.08 20% 1.67%
Union Microfinance (UMF) 13,000 0.22 15% 3.25%
Nepal Microfinance (NMF) 9,000 0.15 18% 2.70%
60,000 1.00 16.02%

b.
Expected Weighted
Stock Investment Weight
Return Return
( NEPSE Index Symbol) (Rs.’000) (1)
(2) (1 *2)
Omni Microfinance (OMF) 6,000 0.08 14% 1.12%
Ambe Microfinance (AMF) 11,000 0.15 16% 2.35%
Ekta Microfinance (EMF) 9,000 0.12 17% 2.04%
Sisne Microfinance (SMF) 7,000 0.09 13% 1.21%
Indra Microfinance (IMF) 20,000 0.27 20% 5.33%
Union Microfinance (UMF) 13,000 0.17 15% 2.60%
Nepal Microfinance (NMF) 9,000 0.12 18% 2.16%
75,000 1.00 16.81%

The expected return on Srijana's portfolio increases to 16.81 percent, because the additional funds
are invested in the highest expected return (20 percent) stock amongst stocks in the portfolio.

VALUATION OF SECURITIES

Bond Valuation

Answer No. 7

a. The yield to maturity is higher than the coupon rate of 8 percent because the bond sells at a
discount from its face value.

b. Discount Rate (Yield to maturity) can be calculated using the following approximation formula

© The Institute of Chartered Accountants of Nepal 6


Paper 4: Financial Management

Interest + (Face Value − Present Value)/Number of period


YTM =
(Face Value + Present Value )/2

1,000 ∗ 8% ∗ 1/2 + (1000 − 935)/2 ∗ 10


YTM =
(1,000 + 935 )/2
1,000 ∗ 8% ∗ 1/2 + (1000 − 935)/2 ∗ 10
YTM =
(1,000 + 935 )/2
40 + 3.25
YTM =
967.50

YTM = 4.5% (rounded up)

Alternatively, Discount Rate (Yield to maturity) can also be calculated using the interpolation
method.

c.
(i) Nominal annual YTM = (2 ×semiannual YTM) = 2 ×4.5 % =9 %
(ii) Effective annual YTM = (1 +semiannual YTM) 2 -1 = (1 +0.045)2−1 =9.203 %

Equity Share Valuation

Answer No. 8

Particulars Firm A Firm B


Next year’s price 100 90
Dividend Income 0 10
Current Price (Assume) A B
Capital Appreciation (100-A) (90-B)
Post tax Capital Appreciation 0.88(100-A) 0.88 (90-B)
Post tax dividend income 0 7.5 (=0.75 * 10)
Total return on share (i) {0.88(100-A)}/A=18% {0.88(90-B)+7.5}/B=18%
Current price (See WN ) Rs.83.02 per share Rs.81.79 per share

Working Note (WN):


Current price can be obtained by solving the equation (i) for each firm A and firm B
Firm A Firm B
{0.88(100-A)}/A=18% {0.88(90-B)+7.5}/B=18%
88-0.88A=0.18A 86.7-0.88B=0.18B
1.06A=88 1.06B = 86.7
A= 83.02 B= 81.79

© The Institute of Chartered Accountants of Nepal 7


Paper 4: Financial Management

WORKING CAPITAL MANAGEMENT AND FINANCIAL FORECASTING

Answer No. 9

Increase in sales = Rs.30 million


Increase in variable cost =Rs.30 million * 0.8= Rs.24 million
Increase in contribution margin =Rs.30 million * 0.2= Rs.6 million
Increase in bad debt =Rs.30 million * 0.1= Rs.3 million
Increase in net income after tax (A) =Rs.{(6-3) * (1-.35)} million = Rs.1.95 million
Opportunity cost on increased receivable (B) = 24 million*45/360 days *16%
=Rs.0.48 million
Net Increase in net income after tax (A-B) = 1.47 million
Assumed 360 days in a year.
Conclusion:
There will be increase in residual income by Rs.1.47 million upon the relaxation of the credit
policy of the firm.

Answer No. 10

The balances in the books of Namaste Private Limited and the books of the bank are shown below:
Rs.
Day 1 2 3 4 5 6 7 8
As per Book of Namaste Private Limited
Opening Balance 80,000 100,000 120,000 140,000 160,000 180,000 200,000 220,000
Add: Cheque received 30,000 30,000 30,000 30,000 30,000 30,000 30,000 30,000
Less: Cheque issued 10,000 10,000 10,000 10,000 10,000 10,000 10,000 10,000
Closing Balance (A) 100,000 120,000 140,000 160,000 180,000 200,000 220,000 240,000
As per Bank's Statement
Opening Balance 80,000 80,000 80,000 80,000 80,000 110,000 140,000 160,000
Add: Cheque received 30,000 30,000 30,000 30,000
Less: Cheque issued 10,000 10,000
Closing Balance (B) 80,000 80,000 80,000 80,000 110,000 140,000 160,000 180,000

Difference (A-B) 20,000 40,000 60,000 80,000 70,000 60,000 60,000 60,000

Conclusion : From day 7 we can find that the balance as per the bank’s statement is less
than the balance as per Namaste Private Limited ’s books by a constant sum of Rs.60,000.
Hence in the steady situation, Namaste Private Limited has a negative net float of Rs.60,000.

© The Institute of Chartered Accountants of Nepal 8


Paper 4: Financial Management

Answer No. 11

a.
POLICY
EXISTING (1) 2 3
Sales (Rs.in millions) 10 10 10
EBIT (Rs.in millions) 1.2 1.2 1.2
Total assets (Rs.in millions) 3.2 3.5 3.8

Total asset turnover 3.125 2.857 2.632


Before-tax return on assets 37.50% 34.29% 31.58%
Before-tax net profit margin 12.0% 12.0% 12.0%

The before-tax net profit margin is unchanged, as sales and earnings before interest and taxes
(EBIT) are the same regardless of the liquidity policy employed.

b.

POLICY
2 3
Increased Assets (Rs.in millions) 3.5 3.8
Existing assets (Rs.in millions) 3.2 3.2
Additional debt (Rs.) 300,000 600,000
Additional interest @ 15% (Rs.) 45,000 90,000

The cost of financing additional current assets could be reduced by the amount that could be earned
on any additional investment of cash in marketable securities. Also, more lenient credit terms may
lead to increased sales and profits. A hidden cost is that part of the debt capacity of the company
is used up by virtue of financing increased levels of current assets with debt.

Answer No. 12

(i)
Economic Order Quantity (EOQ)
2 ∗ 150,000 ∗ 36,000
=� = 600 units
30,000

Total cost of inventory management using EOQ is:


Cost of ordering inventory + cost of holding inventory

© The Institute of Chartered Accountants of Nepal 9


Paper 4: Financial Management

150,000 ∗ 36,000 30,000 ∗ 600


= +
600 2
= 9,000,000 + 9,000,000
= Rs. 18,000,000

(ii)
Total cost of inventory management using 1,000 units is:

150,000 ∗ 36,000 30,000 ∗ 1,000


= +
1,000 2
= 5,400,000 + 15,000,000
= Rs. 20,400,000

Additional inventory management cost if 1,000 components are purchased:


= Rs. 20,400,000 − Rs. 18,000,000 = Rs. 2,400,000

Value of the discount is (150,000 x Rs.200) x 10% = Rs.3, 000,000

Since value of discount is greater than additional cost involved, it is therefore worthwhile to
purchase 1,000 components and take the quantity discount.

Answer No. 13

a.
Determination of External Fund Requirement (EFR):
Rs in million
Total Assets (T) 15,110
Total sales (S0) 31,410
Additional Sales (ΔS) 9,423
New sales level (S1) 40,833
Profit margin ( p) 0.07
Liabilities-spontaneous (L) 2,020
Dividend payout (d) 0.5

T−L
EFR = ∗ ΔS − p ∗ S1 ∗ (1 − d)
S0

15,110 − 2,020
EFR = ∗ 9,423 − 0.07 ∗ 40,833 ∗ (1 − 0.50)
31,410

© The Institute of Chartered Accountants of Nepal 10


Paper 4: Financial Management

EFR = 3,927 − 1,429

EFR = Rs. 2,498 million

b.
(i)
Projected Income Statement
For the year ending mid-July, 2020
Particulars Rs. in million
Sales 40,833
Profits before tax 4,330
Taxes ( 34% ) 1,472
Profit after tax ( 7% on sales) 2,858
Dividend 1,429
Transfer to retained earning 1,429

(ii)
Projected Balance Sheet
As at Mid July 2020
Equity and liabilities Rs.( in million) Assets Rs.( in million)
Share capital 4,200 Fixed assets 11,531
Retained Earnings (WN1) 3,909 Inventories 4,524
Term Loans (WN2) 4,753 Receivables 3,354
Short-term Borrowings (WN3) 4,155 Cash 234
Accounts Payable 1,612
Provisions 1,014
19,643 19,643

Working Notes (WNs):


1.
Retained Earnings = 2,480 +1429
= 3909
2.
Term Loan =3,920 + 2,498 *1/3
=4,753
3.
Short-term Borrowings = 2,490+ 2498 *2/3
= 4,155

© The Institute of Chartered Accountants of Nepal 11


Paper 4: Financial Management

4. Accounts payable, provisions, fixed assets, inventories, receivables and cash are increased in
proportion to increase in sales as per question.

Distribution policy

Answer No. 14

a. DPS in 2019 under the residual dividend model

Particulars Rs.
Projected net income 2,000,000
Less projected capital investments 800,000
Available residual amount 1,200,000
No of Shares outstanding 200,000
DPS 6 per share

b. Payout ratio using the residual dividend model

EPS =Rs.2, 000,000 /200,000 shares = Rs.10 per share


Payout ratio =DPS/EPS =Rs.6 per share / Rs.10 per share = 60%
Or,
Payout ratio =Total dividend /Net Income = Rs.1,200,000/Rs.2,000,000 = 60%

c.
Present market price when payout ratio=60%
D1 6 6
P0 = = = = Rs. 75
r−g 14% − 6% 8%

Under the former circumstance, D 1 would be based on a 20% payout on Rs.10 EPS, or Rs.2. With
r= 14% and g = 12%, P 0 will be as follows:
D1 2 2
P0 = = = = Rs. 100
r−g 14% − 12% 2%

Although the company has suffered a severe setback, its existing assets will continue to provide a
good income stream. More of these earnings should be passed on to the shareholders, as the slowed
internal growth has reduced the need for funds. However, the net result is a 25% decrease in the
value of the share.

© The Institute of Chartered Accountants of Nepal 12


Paper 4: Financial Management

d.
If the payout ratio was continued at 20%, even after internal investment opportunities had declined,
the price of the share would drop to Rs.25 {Rs.2/ (14% - 6%)} rather than to Rs.75. Thus, an
increase in the dividend payout is consistent with maximizing shareholder wealth.

Because of the diminishing nature of profitable investment opportunities, the greater the level of
investment, the lower the average ROE. Thus, the more money the company retains and invests,
the lower its average ROE will be.
We can determine the average ROE under different conditions as follows:
Old situation (with founder active and a 20% payout):
g = (100% - Payout ratio) * Average ROE
12 % = (100% - 20%) * Average ROE
Average ROE = 12%/80% = 15% > (r = 14%)
Note that the average ROE is 15%, whereas the marginal ROE is presumably equal to 14%.

New situation (with founder retired and a 60% payout as explained in Part c):
New growth rate will be as follows:
g = (100- Payout ratio)* ROE = (100% - 20%) * 7:5% = 6.0%
Then,
g = 6% = (100% - 60%) * ROE
ROE = 6% /40% = 15% > (r = 14%)

This suggests that a new payout of 60% is appropriate and that the company is taking on
investments down to the point at which marginal returns are equal to the cost of capital.
Note that if the 20% payout was maintained, the average ROE would be only 7.5%, which would
imply a marginal ROE far below the 14% cost of capital.

Mutual Fund
Answer No. 15
Number of Units purchased = 200,000/38.2 = 5,235.60

Opening
Opening Dividend Dividend Reinvestment New Balance
Date Face
Units % Rs. Rate units Units
Value
Shrawan 1,
5,235.60
2071
Poush 30,
5,235.60 52,356 10 5,235.60 39.10 133.90 5,369.50
2071

© The Institute of Chartered Accountants of Nepal 13


Paper 4: Financial Management

Poush 30 ,
5,369.50 53,695 15 8,054.25 44.20 182.22 5,551.73
2073
Poush 30,
5,551.73 55,517 13 7,217.24 45.05 160.21 5,711.93
2074
Poush 30,
5,711.93 57,119 16 9,139.09 44.80 204.00 5,915.93
2075

On Magh 1, 2076,
Maturity Value = 5,915.93 × 40.40 = Rs.239, 003.51
Less: Cost = Rs.200, 000
Total Gain = Rs.39, 003.51
Gain net of tax = Rs.37, 053.33
Effective yield = (37,053.33 / 200,000) *(1/5.5)
= 3.37%
Capital Market

Answer No. 16

a.
Public issue:
Net proceed per debenture= Rs.1, 000-Rs.10=Rs.990
Number of Rs.1, 000 face-value debenture to be issued to raise Rs.60 million (to nearest note) =
Rs.60,000,000/Rs.990 =60,607 or Rs.60, 607,000 in debentures.
Total interest cost =Rs. 60, 607,000 ×15% ×6 years =Rs.54, 546,300
Total costs =Rs. 54,546,300 +Rs.1, 950,000 =Rs.56, 496,300

Private placement:
Total interest cost =Rs.60, 000,000 ×15.5% ×6 years =Rs.55, 800,000
Total costs =Rs.55, 800,000 +Rs.200, 000 =Rs.56, 000,000

The public issue has the higher total costs. As the interest payments are spread out over the 6 years,
the time value of money effect would act to enhance the private placement. The differential in out-
of-pocket expense occurs at the beginning.

b.
Public issue:
Total interest cost =Rs. 60, 607,000 ×15% ×12 years =Rs.109, 092,600
Total costs =Rs. 109,092,600+Rs.1, 950,000 =Rs.111, 042,600
Private placement:
Total interest cost =Rs.60, 000,000 ×15.5% ×12 years =Rs.111, 600,000

© The Institute of Chartered Accountants of Nepal 14


Paper 4: Financial Management

Total costs =Rs.111, 600,000 +Rs.200, 000 =Rs.111, 800,000

The private placement has the higher total costs. With a longer-term debenture, the differential in
interest rate becomes more important.

CAPITAL INVESTMENT DECISION

Internal Rate of Return

Answer No. 17

Calculation of cash flows of the project


Rs.in million
Year 0 1 2 3 4 5
1.Revenues 50.00 60.00 70.00 60.00 50.00
2.Operating costs (60%*1) 30.00 36.00 42.00 36.00 30.00
3.Loss of contribution margin 4.00 4.00 4.00 4.00 4.00
4.Depreciation (WN1) 6.00 5.10 4.34 3.68 3.13
5.Profit before tax (1-2-3-4) 10.00 14.90 19.66 16.32 12.87
6.Tax ( 30% * 5 ) 3.00 4.47 5.90 4.90 3.86
7.Profit after tax (5-6) 7.00 10.43 13.76 11.42 9.01
8.Net working capital (WN2) (10.00) (2.00) (2.00) 2.00 2.00
9.Recovery of working capital 10.00
10.Initial Outlays on Fixed assets (40.00)
11.Net salvage value of fixed assets 20.00
12.Operating cash flow (7 + 4) 13.00 15.53 18.10 15.10 12.14
13.Terminal cash inflow (9+11) 30.00
14.Net cash flow (8+10+12+13) (50.00) 11.00 13.53 20.10 17.10 42.14

Net Present Value (NPV) of cash flows at discount rate of 20% and 25% is as follows:
Rs.in million
Year Net Cash flows PV factor @ 20% PV @ 20% PV factor @ 25% PV @ 25%
0 (50.00) 1 (50.00) 1 (50.00)
1 11.00 0.8333 9.17 0.8000 8.80
2 13.53 0.6944 9.40 0.6400 8.66
3 20.10 0.5787 11.63 0.5120 10.29
4 17.10 0.4823 8.25 0.4096 7.00

© The Institute of Chartered Accountants of Nepal 15


Paper 4: Financial Management

5 42.14 0.4019 16.94 0.3277 13.81


NPV 5.38 (1.44)

By interpolation method,

NPV at LDR
IRR = LDR + ∗ (HDR − LDR)
NPV at LDR − NPV at HDR

Where, LDR= Lower Discount Rate


HDR= Higher Discount Rate

5.38
IRR = 20% + ∗ (25% − 20%) = 23.94%
5.38 − (−1.44)

Advise: As the IRR of the project is 23.94 % which is greater than the Bank’s minimum IRR
requirement of 20%, the project seems eligible for bank’s 14 % term loan in accordance with credit
stipulation.

Replacement Decision

Answer No. 18

i)
Particulars Rs.
Cash outflow on Machine – N 250,000
Less : Sale value of Machine- M 110,000
Less : Cost of Dismantling and Removal (40,000) 70,000
Net outflow 180,000

P.V. of incremental cash inflows = (From (ii) workings below)


1,332,020 (N) – 1,134,147 (M) = 197,873
NPV of Machine – N = 197,873 – 180,000 = 17,873

Decision: Since NPV of Machine – N is positive, replacement is advised.

Note: Rs.210, 000 spent on Machine – M is a sunk cost and hence not relevant for deciding the
replacement decision.

(ii) Independent evaluation:


Machine- M

© The Institute of Chartered Accountants of Nepal 16


Paper 4: Financial Management

PV of cash
Year Total cost Revenue Net cash inflow PV factor @ 15%
inflows
1 650,000 900,000 250,000 0.8696 217,400
2 650,000 900,000 250,000 0.7562 189,050
3 715,000 1,050,000 335,000 0.6576 220,296
4 780,000 1,200,000 420,000 0.5718 240,156
5 812,500 1,350,000 537,500 0.4972 267,245
1,134,147
Cash Outflow (210,000)
NPV 924,147

Machine- N
PV of cash
Year Total cost Revenue Net cash inflow PV factor @ 15%
inflows
1 650,000 900,000 250,000 0.8696 217,400
2 650,000 900,000 250,000 0.7562 189,050
3 650,000 1,050,000 400,000 0.6576 263,040
4 650,000 1,200,000 550,000 0.5718 314,490
5 650,000 1,350,000 700,000 0.4972 348,040
1,332,020
Cash Outflow (250,000)
NPV 1,082,020

Decision: As the NPV of Machine – N is higher than that of Machine – M, the choice should fall
on Machine – N.
Note: As the company is a zero tax company depreciation and the tax effect on the same are not
relevant for consideration.

Answer No. 19

The word ‘Financial Management’ is a combination of two crucial words in business environment
viz. ‘Finance’ and ‘Management’. ‘Finance’ means funds and therefore, financial management
refers to the management of funds. It is a technique of planning, sourcing and investing funds in
most effective manner. Financial management is concerned with the acquisition, financing, and
management of assets with an overall organization goal in mind. Financial decisions involve
procurement of funds and utilization of funds. The expected outcome of activity of ‘Procurement
of Funds’ is not only limited to the acquisition of required funds for business but it should be
ensured that it is acquired at the lowest possible costs, risks and dilution in control. On the other
hand, ‘Utilization of Funds’ entails employment of funds at the right place to ensure highest returns

© The Institute of Chartered Accountants of Nepal 17


Paper 4: Financial Management

possible which are at least more than the cost of funds. It also needs to ensure that there are no idle
funds to incur cost unnecessarily.

According to MM theory on dividend policy, dividend policy of a firm is irrelevant as it does not
affect the wealth of shareholders. That is, dividend policy has no effect on the price of the shares
of the firm and believes that it is the investment policy that increases the firm’s share value. Thus,
when investment decision of the firm is given, dividend decision on the split of earnings between
dividends and retained earnings is of no significance in determining the value of the firm.
MM hypothesis is based on the following assumptions.
1. The firm operates in perfect capital market.
2. Taxes do not exist.
3. The firm has a fixed investment policy
4. Risk of uncertainty does not exist. That is, investors are able to forecast future prices and
dividends with certainty and one discount rate (r) is appropriate for all securities and all time
periods. Thus, r = K = K t for all t.
5. There are no flotation costs or transaction costs.

Because of the unrealistic nature of the assumption, MM hypothesis lacks practical relevance in
the real world situation. Thus, it is being criticized on the following grounds.
1. The assumption that taxes do not exist is far from reality.
2. MM argue that the internal and external financing are equivalent. This cannot be true if the costs
of floating new issues exist.
3. According to MM hypothesis, the wealth of a shareholder will be same whether the firm pays
dividends or not. But, because of the transactions costs and inconvenience associated with the sale
of shares to realize capital gains, shareholders prefer dividends to capital gains.
4. Even under the condition of certainty it is not correct to assume that the discount rate (k) should
be same whether firm uses the external or internal financing.

Answer No. 20

a)
Foreign Direct Investment (FDI) means an investment made with an intent of obtaining an
ownership stake in an enterprises domiciled in a country by an enterprise situated in some other
country. In other words, the investment made by the international investors to obtain a substantial
interest in the enterprise located in a different country is a Foreign Direct Investment. The
investment may result in the transfers of funds, resources, technical know-how, strategies, etc.
There are several ways, of making FDI i.e. creating a joint venture or through merger and
acquisition or by establishing a subsidiary company. The investor company has a substantial
amount of influence and control over the investee company. FDI investors play an active role in
the management of the investee company as the FDI investors gain both ownership and

© The Institute of Chartered Accountants of Nepal 18


Paper 4: Financial Management

management right through investment. FDI investors invest in financial and non-financial assets
like resources, technical know-how along with securities. Entry and exit of FDI are very difficult,
while this is not so with PI.

Portfolio Investment (PI) refers to the investment made in the financial assets (only) of an
enterprise based in one country by the foreign investors. The investment made in passive holdings
like stocks, bonds, etc. of the enterprise of a foreign country by overseas investors is known as a
Portfolio Investment. The investment is made in the securities of the company, i.e. stock, bonds,
etc. Such an investment is made with the purpose of getting short term financial gain and not for
obtaining significant control over managerial operations of the enterprise. Usually, PI investors go
for securities that are highly liquid.

b)
Money (Nominal) interest rate is the rate of return which an investor or borrower will get or have
to pay in the market without any adjustment for inflation. The money interest rate describes the
interest rate without any correction for the effects of inflation. Thus, the quoted or stated interest
rate on loans/borrowings or bank deposit accounts is usually a nominal one. This rate shows the
actual price investor or borrower are paid (or have to pay) for lending (or borrowing) money.

The real interest rate refers to the interest rate adjusted to remove the effects of inflation. This rate
shows by how much the actual purchasing power of the money has increases over time. In other
words, it describes the real yield of lending money or the real cost of borrowing money. Real
interest rate can be derived with following Fisher equation as follows:

Real Interest Rate = Nominal Interest Rate –Inflation Rate

For example, a saver who deposits Rs.1, 000 in a bank account for one year may get a nominal
rate of interest of 2.5%, and thus receive Rs.1, 025 in a year’s time. However, if prices increase by
3%, s/he will need Rs.1, 030 to purchase the same goods or services that, one year earlier, would
have cost Rs.1, 000. This means that the real return will actually have been -0.5%. This is the real
interest rate, and it is calculated by subtracting the rate of inflation (3%) from the nominal interest
rate (2.5%).

c)
Absolute interest rate risk results from the possibility of a directional, or up or down, change in
interest rates. Most organizations monitor absolute interest rate risk in their risk assessments, due
to both its visibility and its potential for affecting profitability. From a borrower’s perspective,
rising interest rates might result in higher project costs and changes to financing or strategic plans.
From an investor or lender perspective, a decline in interest rates results in lower interest income
given the same investment, or alternatively, inadequate return on investments held. All else being

© The Institute of Chartered Accountants of Nepal 19


Paper 4: Financial Management

equal, the greater the duration, the greater the impact of an interest rate change. The most common
method of hedging absolute interest rate risk is to match the duration of assets and liabilities, or
replace floating interest rate borrowing or investments with fixed interest rate debt or investments.
Another alternative is to hedge the interest rate risk with tools such as forward rate agreements,
swaps, and interest rate caps, floors, and collars.

Yield curve risk refers to the probability that the yield curve will shift in a manner that affects the
values of securities tied to interest rates-particularly, bonds. The yield curve is a graphical
illustration of the relationship between interest rates and bond yields of various maturities. A yield
curve can be positive (meaning the line goes up and to the right as investors require a higher rate
of return for lending money for a longer period), inverted (meaning that the line slopes down and
to the right), or flat. These shapes indicate not just whether short-term bond yields are higher or
lower than long-term bond yields, but how investors feel about future interest rates and even
economic growth. When the yield curve shifts, the price of the bond, which was initially priced
based on the initial yield curve, will change in price. Any investor holding interest-rate-based
securities is exposed to yield curve risk. Yield curve swaps and strategies using products such as
interest rate futures and forward rate agreements along the yield curve can take advantage of
changes in the shape of the yield curve.

d)
Operational restructuring is the process of increasing the economic viability of the underlying
business model. Examples include mergers, the sale of divisions or abandonment of product lines,
or cost-cutting measures such as closing down unprofitable facilities. In most turnarounds and
bankruptcy situations, both financial and operational restructuring must occur simultaneously to
save the business.

Financial restructuring relates to improvements in the capital structure of the firm. Financial
restructuring involves restructuring the assets and liabilities of corporations, including their debt-
to-equity structures, in line with their cash flow needs to promote efficiency, support growth, and
maximize the value to shareholders, creditors and other stakeholders. Financial restructuring mean
refinancing at every level of capital structure including securing asset-based loans (accounts
receivable, inventory, and equipment), securing mezzanine and subordinated debt financing,
securing institutional private placements of equity, achieving strategic partnering , etc.

e)
The term bank rate means an interest rate charged by the central bank on the loans and advances
extended to the commercial banks, development banks and financial institutions. It is regulated by
central bank’s monetary policies. Bank Rate is one of the essential tools of the central bank which
regulates the economic activity level, i.e. to control liquidity, the central bank will increase bank
rate and vice versa. As a result of which the borrowing cost of banks goes up, and the credit supply

© The Institute of Chartered Accountants of Nepal 20


Paper 4: Financial Management

in the economy goes down. An upward revision in the bank rate signals that the banks should also
raise their lending rate and the deposit rates.

Repo rate is the rate at which banks borrow money from the central bank on the event of a
deficiency of funds. The term ‘repo’, is an acronym for repurchase option that acts as a source of
short-term borrowing in which the banks sell securities to the central bank in return for fund.
Central bank uses this tool to control money supply in the country’s monetary system, i.e. the
decrease in the repo rate indicates that the bank’s cost of borrowing is reduced while if the repo
rate goes up, it represents that bank’s borrowing rate is expensive, that ultimately results in the fall
in money supply in the economy. In this way, it also helps the government authorities to inject or
curb the liquidity in the banking system.

The bank rate is normally higher than the repo rate. In a bank rate, there is nothing like repurchase
agreement; only the money is lent to banks and financial intermediaries at a fixed rate. Conversely,
in Repo Rate, the sale of securities to the central bank, on a repurchase agreement, i.e. to buy back
the securities at a predetermined rate and date in the future. In general, bank rate caters long term
fund requirement of the commercial banks whereas the repo rate focuses on providing short-term
finance to banks.

f)

Capital Structure covers only the long term sources of funds, whereas financial structure implies
the way assets of the company are financed, i.e. it represents the whole liabilities side of the balance
sheet, which includes both long term and long term debt and current liabilities. In other words,
financial structure is a broader concept than capital structure, or that capital structure is a
subdivision of financial structure. Capital structure includes equity capital, preference capital,
retained earnings, debentures, long-term borrowings, etc. On the other hand, financial structure
includes shareholder’s fund, current and non-current liabilities of the company. Capital structure
appears under the head shareholders fund and non-current liabilities. Conversely, the entire equity
and liabilities side shows the financial structure of the company.
g)
Annuity is the series of cash flows, usually of fixed amount, paid/received at regular intervals. The
interval can be annually, semi-annually or monthly, etc. For example, life insurance premium paid
annually, etc. On the other hand, perpetuity is a type of annuity that continues for infinite number
of years. For example, dividend of irredeemable preference shares, etc. In other words, annuity
has a definite end but perpetuity is never ending, it is indefinite. Perpetuity is an annuity, but an
annuity is not perpetuity. Future value of annuity can be easily calculated which is not possible in
case of perpetuity.
The formula for annuity is as follows:

© The Institute of Chartered Accountants of Nepal 21


Paper 4: Financial Management

(1 + R)n − 1
Future Value = Annuity ∗
R
Where, n= number of years, R= Rate of return
The formula, in case of perpetuity, is as follows:
Cash flow
Present Value of perpetuity =
R−G
Where, R= Rate of return, G= Growth rate, if any

Answer No. 21

a)
In general, gold deposit account is a fixed deposit in terms of gold opened in the bank. Customers
deposit the gold that is lying idle with them under this account and in return get safety and earn
interest. In this scheme of account, a depositor deposits gold (such as jewelry, coin, etc.) with a
bank which is then lent by the bank to its borrowers (such as jewelry makers), after melting into
gold bars. This is akin to a normal banking operation (like a savings account in bank), but carried
out in terms of gold instead of in rupees. This account allows the depositors of gold to earn market
determined interest income (denominated in gold but recoverable either in gold or in rupee from
the pure gold they deposit with banks and permits the jewelers to obtain their raw material -gold
bars created from the melting of the gold deposited with the banks- as loans. In addition, Banks
would also be able to monetize their gold. Gold can be submitted in any form (bullion, jewelry
etc.) but the amount deposited with the bank is calculated on the basis of the pure gold content of
that deposit (after removing the weights of precious stones in jewelry etc.), which is verified
through the gold tester. Both principal and interest to be paid to the depositors of gold will be
valued in gold. For example, if a customer deposits 100 grams of gold and gets 1 percent interest,
then, on maturity he has a credit of 101 grams. The customer will have the option of redemption
either in cash or in gold, which will have to be exercised in the beginning itself (at the time of
making the deposit).The operational mechanism aspects of this deposit scheme is governed by
regulations of central bank of the country.
The basic idea behind gold deposit accounts in the bank is to achieve following objectives:

• To mobilize the idle gold held by households and put it into productive use.
• To provide a fillip to the gems and jewelry sector in the country by making gold
available as raw material on loan from the banks.
• To be able to reduce reliance on import of gold over time to meet the domestic demand

b)

© The Institute of Chartered Accountants of Nepal 22


Paper 4: Financial Management

Book building is a process of price discovery. Book building is a process by which the issuer
company before filing of the prospectus, builds-up and ascertains the demand for the securities
being issued and assesses the price at which such securities may be issued and ultimately
determines the quantum of securities to be issued. The book building process comprises of
following process:
i. The issuing company hires an investment bank to act as underwriter who is tasked with
determining the price range the security can be sold for and drafting a prospectus to send
out to the investors.
ii. Invite investors to submit bids on the number of shares that they are interested in buying
and the prices that they would be willing to pay.
iii. The book is 'built' by listing and evaluating the aggregated demand for the issue from the
submitted bids. The underwriter analyzes the information then uses a weighted average to
arrive at the final price for the security, which is termed the 'cut off' price.
iv. The underwriter has to, for the sake of transparency, publicize the details of all the bids
that were submitted.
v. Allocate the shares to the accepted bidders.

Some of the advantages of book building are as follows:


i. Book building helps in evaluating the intrinsic worth of the instrument being offered and
the company’s credibility in the eyes of public.
ii. The prime objective of book building process is to determine the highest market price for
shares and securities and demand level from highest quality investors in order to adjust
pricing and allocation decision.
iii. Price of instrument is determined in a more realistic way on the commitments made by the
prospective investors to the issue.
iv. Book building is a process of fixing price for an issue on feedback from potential investors
on how they are willing to bid to pick up issues and instruments.

Some limitations of book building are as follows:


i. The book building system works very efficiently in matured market conditions. In such
circumstances, the investors are aware of various parameters affecting the market price of
the securities. But, such conditions are not commonly found in practice.
ii. The issuer company should be fundamentally strong and well known to the investors.

c)
The marketability (or liquidity) of a security relates to the owner’s ability to convert it into cash.
Securities are said to be marketable securities when they are liquid financial instruments that can
be quickly converted into cash at a reasonable price. The liquidity of marketable securities comes
from the fact that the maturities tend to be generally less than one year, and that the rates at which
they can be bought or sold have little effect on prices. Marketable securities include marketable

© The Institute of Chartered Accountants of Nepal 23


Paper 4: Financial Management

equity securities and marketable debt securities. Example of marketable equity securities include
shares of common stock which are actively traded on a stock exchange and for which there are
quoted market prices. Example of marketable debt securities include treasury bills and other
money market instruments.

There are two dimensions to marketability: the price realized and the amount of time required to
sell the asset. The two are interrelated in that it is often possible to sell an asset in a short period if
enough price concession is given. For financial instruments, marketability is judged in relation to
the ability to sell a significant volume of securities in a short period of time without significant
price concession. The more marketable the security, the greater the ability to execute a large
transaction near the quoted price. In general, the lower the marketability of a security, the greater
the yield necessary to attract investors. Thus, the yield differential between different securities of
the same maturity is caused not by differences in default risk alone, but also by differences in
marketability. Marketability (liquidity) risk arises from the inability to convert an investment
quickly into cash (marketability characteristics).

d)
Financial modeling is the task of building an abstract representation (a model) of a financial
decision-making situation. This is a mathematical model designed to represent (a simplified
version of) the performance of a financial asset or portfolio of a business, project, or any other
investment. Financial modeling is a general term that means different things to different users; the
reference usually relates either to accounting and corporate finance applications, or to quantitative
finance applications. In other words, financial modeling is about translating a set of hypotheses
about the behavior of markets or agents into numerical predictions; for example, a firm’s decisions
about investments (the firm will invest 30% of assets), or investment returns (returns on “stock K”
will, on average, be 15% higher than the market’s returns).

Different types of financial models are as follows:


i. Risk analysis models - used to analyze different types of risk
ii. Trading models - used in portfolio management and sales/trading functions
iii. Portfolio allocation models - determine asset type and other allocations within a portfolio.
iv. Financial Statements Projection Model (But the most commonly used type of a financial
model is the financial statements projection model. Financial statements projection model
forecasts the company’s future financial results and consists of: Income Statement, Balance
Sheet and Cash Flow Statement.)

Financial modelling process is as follows:


i. Planning and Specification the Project
ii. Designing and Building the Financial Model
iii. Testing the Financial Model
iv. Documentation of the Financial Model

© The Institute of Chartered Accountants of Nepal 24


Paper 4: Financial Management

Following are some uses of financial modeling:


i. Business valuation, especially discounted cash flow,
ii. Scenario planning and management decision making (“what is”; “what if”; “what has to be
done”)
iii. Capital budgeting
iv. Financial statement analysis
v. Option pricing
vi. Other derivatives, especially Interest rate derivatives and Exotic derivatives
vii. Modeling the term structure of interest rates (short rate modeling) and credit spreads
viii. Credit scoring and provisioning
ix. Corporate financing activity prediction problems
x. Portfolio problems
xi. Real options
xii. Risk modeling and Value at risk.

e)
The leverage effect explains a company’s return on equity in terms of its return on capital
employed and cost of debt. It is the difference between return on equity and return on capital
employed. It explains how it is possible for a company to deliver a return on equity exceeding the
rate of return on all the capital invested in the business – i.e., its return on capital employed. When
a company raises debt and invests the funds it has borrowed in its industrial and commercial
activities, it generates operating profit that normally exceeds the interest expense due on its
borrowings. The company generates a surplus consisting of the difference between the return on
capital employed and the cost of debt related to the borrowing. This surplus is attributable to
shareholders and is added to shareholders’ equity. The leverage effect of debt thus increases the
return on equity. If the return on capital employed falls below the cost of debt, then the leverage
effect of debt shifts into reverse and reduces the return on equity, which in turn falls below the
return on capital employed. The leverage effect is expressed in the following formula:

ROE = ROCE + (ROCE−i) ×D/E

Where, ROE is the return on equity, ROCE is the after-tax return on capital employed, i is the
after-tax cost of debt, D is net debt, E is equity. The leverage effect itself is (ROCE− i) × D/E.

f)
Some of limitations of Internal Rate of Return (IRR) analysis are as follows:
i. IRR method assumes that the net cash inflows generated through the project life will be
reinvested to earn the same return as the IRR, but this may not be possible in real life.

© The Institute of Chartered Accountants of Nepal 25


Paper 4: Financial Management

ii. IRR does not consider cost of capital; it should not be used to compare projects of different
duration.
iii. In the case of positive cash flows followed by negative ones and then by positive ones, the
IRR may have multiple values. This makes it impossible to select a project simply based
on IRR
iv. IRR, as an investment decision tool, should not be used to rate mutually exclusive projects,
but only to decide whether a single project is worth investing in.

g)
DuPont analysis examines the return on equity (ROE) analyzing profit margin, total asset turnover,
and financial leverage. It was created by the DuPont Corporation in the 1920s. The DuPont analysis
model breaks down the original equation for ROE into three components: operating efficiency,
asset efficiency, and leverage. Operating efficiency is measured by net profit margin and indicates
net income to sales. Asset efficiency is measured by the total asset turnover and represents the
sales to total assets. Finally, financial leverage is determined by the equity multiplier. Thus,
DuPont Analysis method breaks down and clarifies the different components of the Return on
Equity (ROE) formula, which can help companies with finding ways to improve their return on
equity.

In a DuPont analysis, the formula for ROE is:

ROE = Net Profit Margin ∗ Asset Turnover ∗ Equity Multiplier

The formula breaks down further to:

Net Income Sales Total assets


ROE = ∗ ∗
Sales Total assets Total shareholders′ equity

© The Institute of Chartered Accountants of Nepal 26


Paper 5 : Cost and Management Accounting

Paper 5: Cost and Management Accounting

© The Institute of Chartered Accountants of Nepal 1


Paper 5 : Cost and Management Accounting

Revision Questions
Costs concepts and costing methods
Question No. 1

a) What is cost plus contract? State its advantages.


b) How apportionment of joint costs up to the point of separation amongst the joint
products using market value at point of separation and net realizable value method is
done? Discuss.
c) Discuss the accounting treatment of Idle time and overtime wages.
d) Discuss cost classification based on variability and controllability.

Material Control
Question No. 2

a) A Ltd. manufactures a product X which requires two raw materials A and B in ratio
of 1:4. The sales department has estimated a demand of 5,00,000 units for the
product for the year. To produce one unit of finished product, 4 unit of material A is
required.
Stock position at the beginning of the year is as below:
Product – X 12,000 units
Material A 24,000 units
Material B 52,000 units
To place an order the company has to spend Rs. 15,000. The company is financing
its working capital using a bank cash credit @13% p.a.
Product X is sold at Rs. 1,040 per unit. Material A and B purchased at Rs. 150 and
Rs. 200 respectively.
Required:
Compute economic order quantity (EOQ):
i) If purchase order for the both materials is placed separately.
ii) If purchase order for the both materials is not placed separately.

Labour Control
Question No. 3

a) The following information of a work is given:


Weekly working hours 45
Wages Rate per hour (Rs.) 8.00
Piece Rate per Unit (Rs.) 4.00
Normal Time taken per piece 20 minutes

© The Institute of Chartered Accountants of Nepal 2


Paper 5 : Cost and Management Accounting

Normal Output Per Week 100 Pieces


Actual Output for the week 120 Pieces
Differential Piece Rate 80% of Piece Rate when actual output is below
normal output that is 100 pieces and 120% of
Piece Rate when actual output is above normal
output.

You are required to calculate the earnings of a worker for a week under following
plans:
i) Differential Piece Rate and
ii) Halsey Premium Scheme ( 50% sharing)

b) A company planned to produce 2,000 units of a product in a week of 40 hours by


employing 65 skilled workers. Other relevant information are as follows:
• Standard wages rate : Rs. 45 per hour
• Actual Production : 1800 units
• Actual number of worker employed : 50 workers in a week of 40 hours
• Actual wages rate : Rs. 50 per hour
• Abnormal time loss due to machinery breakdown : 100 hours
You are required to calculate:
i) Labour cost, rate, idle time and efficiency variances
ii) Reconcile the variance

Overhead
Question No. 4

a) RSJ produces a single product and absorbs production overheads at a pre- determined
rate. Information relating to a period is as under:
Production overhead actually incurred Rs. 4,84,250
Overhead recovery rate at production Rs. 1.45 per hour
Actual hours worked 2,65,000 hours
Production:
Finished goods 17,500 units

© The Institute of Chartered Accountants of Nepal 3


Paper 5 : Cost and Management Accounting

Work-in - progress ( 50% complete in all respect) 5,000 units


Sales of finished goods 12,500 units

At the end of the period, it was discovered that the actual production overheads
incurred included Rs. 40,000 on account of 'written off obsolete stores' and wages
paid for the strike period under an award.
It was also found that 30% of the under absorption of production overheads was due
to factory inefficiency and the rest was attributable to normal increase in costs.
Required to calculate:
i) The amount of under absorption of production overheads during the period.
ii) Show the accounting treatment of under absorption of production overheads
and pass journal entry.

b) APP Limited is a manufacturing concern and recovers overheads at a pre-determined


rate of Rs. 30 per man -day.
The following additional information of a period are also available for you:
Total factory overheads incurred Rs. 51,00,000
Man - days actually worked 1,50,000
Sales ( in units) 50,000
Stock at the end of the period:
Completed units 5,000
Incompleted units ( 50% completed) 10,000

There was no opening stock of finished goods and works in progress.


On analyzing the situation, it was discovered that 60% of the unabsorbed overheads
were due to defective planning and balance were attributable to increase in overhead
costs.
How would you treat unabsorbed overheads in cost accounts?

© The Institute of Chartered Accountants of Nepal 4


Paper 5 : Cost and Management Accounting

Costs Accounts System, Cost Control (Integrated and Non-integrated Accounting System)

Question No. 5

The following is the summarized Trading and Profit and Loss Account of XYZ Ltd.
for the year ended 31st Ashadh, 2076:
Particulars Amount Particulars Amount
(Rs.) (Rs.)
Direct Material 14,16,000 Sales ( 30,000 units) 30,00,000
Direct wages 7,42,000 Finished stock (2,000 units) 1,67,500
Works overheads 4,26,000 Work-in-progress:
Administration overheads 1,50,000 - Materials - 34,000
Selling and distribution overheads 1,65,000 - Wages -16,000
Net profit for the year 3,22,500 - Work Overhead- 4,000 54,000
32,21,500 32,21,500
The company's cost records show that in course of manufacturing a standard unit :
i) works overheads have been charged @ 20% on prime cost,
ii) administration overheads are related with production activities and are recovered at
Rs. 5 per finished unit, and
iii) selling and distribution overheads are recovered at Rs. 6 per unit sold.
You are required to prepare:
i) Costing profit and Loss Account indicating the net profits,
ii) A Statement showing reconciliation between profit as disclosed by the Cost Accounts
and Financial Accounts.

Methods of Costing
Question No. 6

a) MKS Ltd. is engaged in construction sector. It took a contract to build a house for Rs.
45 lakhs. The contract commenced on 1st Shrawan , 2075. Following information,
relating to contract, for the year endind on 31st Ashadh, 2076 are as under:
Particulars Rs.
Materials purchased 8,52,000
Wages 10,48,000
Indirect expenses 92,000
Administrative charge 1,18,000
Material at site at the end of the year 38,000

© The Institute of Chartered Accountants of Nepal 5


Paper 5 : Cost and Management Accounting

A plant was purchased for the contract on 1st Shrawan, 2075 which, after charging
depreciation @ 15% p.a. on the cost, appeared at Rs. 6,12,000 at the end of the
year.
A supervisor who is paid Rs. 10,000 per month has devoted two-third of his time
to this contract.
Two-third of the contract was completed. The architect issued certificate covering
50% of the contract price and contract has been paid 90% of the work certified on
account. The books of accounts are closed on 31st Ashadh every year.
Prepare contract account showing following:
i) Works cost of the contract
ii) Value of works uncertified
iii) Notional profit and
iv) Amount to be carried to profit and loss account.

b) From the following information for the month of January-2020, prepare Process-
III cost accounts.
Opening WIP in process-III 1,600 units at Rs. 24,000
Transfer from Process-II 55,400 units at Rs. 6,23,250
Transferred to warehouse 52,200 units
Closing WIP of Process-III 4,200 units
Units Scrapped 600 units
Direct material added in Process- III Rs. 2,12,400
Direct wages Rs. 96,420
Production overheads Rs.56,400

Degree of completion:
Opening Stock Closing Stock Scrap
Material 80% 70% 100%
Labour 60% 50% 70%
Overheads 60% 50% 70%
The normal loss in the process was 5% of the production and scrap was sold @
Rs. 5 per unit.

c) A Ltd produces 'M' as a main product and gets two by products - 'P' and 'Q' in the
course of processing.
Following information are available for the month of Magh, 2076:
M P Q
Cost after separation - Rs. 60,000 Rs. 30,000
No. of units produced 4500 2500 1500
Selling price ( Per Unit) Rs. 170 Rs. 80 Rs. 50
Estimated Net profit to sales - 30% 25%

© The Institute of Chartered Accountants of Nepal 6


Paper 5 : Cost and Management Accounting

The joint cost of manufacture upto separation point amounts to Rs. 2,50,000.
Selling expenses amounting to Rs. 85,000 are to be apportioned to the three products
in the ratio of sales units.
There is no opening and closing stock.
Prepare the statement showing:
i) Allocation of joint cost
ii) Product wise overall profitability and
iii) Advise the company regarding results if the by product 'P' is not further
processed and is sold at the point of separation at Rs. 60 per unit without
incurring selling expenses.

d) A company wants to outsource the operation of its canteen to a contractor. the


company will provide space for cooking, free electricity and furniture in the canteen.
The contractor will have to provide lunch to 300 workers of which 180 are vegetarian
(Veg) and the rest are non - vegetarian ( Non- Veg). In the case of non -veg meals,
there will be a non -veg item in addition to the veg items. A contractor who is
interested in the contract has analysed the costs likely to be incurred. His analysis is
given below:
Cereals Rs. 8 per plate
Veg items Rs. 5 per plate
Non- veg items Rs. 15 per plate
Spices Rs. 1 per plate
Cooking oil Rs. 4 per plate
One cook Salary Rs. 13,000 per month
Three helpers Salary Rs. 7,000 per month per head
Fuel Two commercial cylinders per month,
price Rs. 1000 each

On an average the canteen will remain open for 25 days in a month. The contractor
wants to charge the non- veg meals at 1.50 times of the veg meals.
You are required to calculate:
i) The price per meal (veg and non - veg separately) that contractor should quote
if he wants a profit of 20% on his takings.
ii) The price per meal ( separately for veg and non -veg ) that a worker will be
required to pay if the company provides 60% subsidy for meals out of welfare
fund.

© The Institute of Chartered Accountants of Nepal 7


Paper 5 : Cost and Management Accounting

Cost Concepts for Decision Making


Question No. 7

A company is producing an identical product in two factories. The following are the
details in respect of both factories:
Particulars Factory X Factory Y
Selling price per unit ( Rs.) 50 50
Variable cost per unit ( Rs.) 40 35
Fixed cost ( Rs.) 2,00,000 3,00,000
Depreciation included in above fixed cost (Rs.) 40,000 30,000
Sales in units 30,000 20,000
Production capacity ( units) 40,000 30,000

You are required to determine:


i) Break Even Point ( BEP) each factory individually.
ii) Cash Break Even point for each factory individually.
iii) BEP for company as a whole, assuming the present product mix is in sales
ratio.
iv) Consequence on profit and BEP if product mix is changed to 2:3 and total
demand remain same.

Costing for planning and Control –Budgets


Question No.8

V Ltd. produces and markets a very popular product called 'X'. The company is
interested in presenting its budget for the second quarter of Fiscal Year 2075/76.
The following information are made available for this purpose:
i) It expects to sell 50,000 bags of 'X' during the second quarter of Fiscal Year
2075/76 at the selling price of Rs. 900 per bag.
ii) Each bag of 'X' requires 2.5 kgs of a raw - material called 'Y' and 7.5 kgs. of
raw - material called 'Z'.
iii) Stock levels are planned as follows :
Particulars Beginning of Quarter End of Quarter
Finished Bags of 'X' (Nos.) 15,000 11,000
Raw - Material 'Y' (Kgs.) 32,000 26,000
Raw - Material 'Z' (Kgs.) 57,000 47,000
Empty Bag (Nos.) 37,000 28,000

iv) 'Y' cost Rs. 120 per Kg., 'Z' costs Rs. 20 per Kg. and 'Empty Bag' costs Rs. 80
each.
v) It requires 9 minutes of direct labour to produce and fill one bag of 'X' .

© The Institute of Chartered Accountants of Nepal 8


Paper 5 : Cost and Management Accounting

Labour cost is Rs. 50 per hour.


vi) Variable manufacturing costs are Rs. 45 per bag. Fixed manufacturing costs
Rs. 30,00,000 per quarter.
vii) Variable selling and administration expenses are 5% of sales and fixed
administration and selling expenses are Rs. 2,50,000 per quarter.

Required
i) Prepare a production budget for the said quarter.
ii) Prepare a raw- material purchase budget for 'Y', 'Z' and 'Empty Bags@ for the
said quarter in quantity as well as in rupees.
iii) Compute the budgeted variable cost to produce one bag of 'X'.

Standard Costing
Question No. 9

a) Explain the reason for Preference of Standard Costing.

b) Aaradhya Ltd. manufactures a commercial product for which the standard cost per
unit is as follows:
Particulars Rs.
Material:
5 kg. @ Rs. 4 per kg. 20.00
Labour:
3 hours @ 10 per hour 30.00
Overhead:
Variable: 3 hours @ Rs. 1 3.00
Fixed : 3 hours @ Rs. 0.50 1.50
Total 54.50
During Jan. 2020, 600 units of the product were manufactured at the cost shown
below:
Particulars Rs.
Material purchased:
5,000 kg. @ Rs. 4.10 per kg. 20,500
Material Used:
3,500 kg.
Direct Labour :

© The Institute of Chartered Accountants of Nepal 9


Paper 5 : Cost and Management Accounting

1,700 hours @ 9 per hour 15,300


Variable Overhead 1,900
Fixed Overhead 900
Total 38,600

The flexible budget required 1,800 direct labour hours for operation at the
monthly activity level used to set the fixed overhead rate.
Compute:
a) Material price Variance
b) Material Usage variance
c) Labour rate variance
d) Labour efficiency variance
e) Variable overhead expenditure variance
f) Variable overhead efficiency variance
g) Fixed overhead expenditure variance
h) Fixed overhead volume variance
i) Fixed overhead capacity variance
j) Fixed overhead efficiency variance
Also reconcile the standard and actual cost of production.

Uniform Costing and Inter-firm comparison


Question No. 10

a) What is uniform costing? What is the scope of uniform costing?

b) Explain the Requisites for inter-firm or intra-firm comparison

Cost control and cost reduction


Question No. 11

Explain the advantage of cost control.

© The Institute of Chartered Accountants of Nepal 10


Paper 5 : Cost and Management Accounting

Answers/Hints:
Costs concepts and costing methods
Question No. 1 (a)

Solution:

Cost plus contract: Under cost plus contract, price is ascertained by adding a percentage of
profit to the total cost of the work. Such types of contracts are entered in to when it is not
possible to estimate the contract cost with reasonable accuracy due to unstable condition
of material, labour service etc.
Following are the advantages of cost plus contract.
(i) The contractor is assured of a fixed percentage of profit. There is no risk of
incurring any loss on the contract.
(ii) It is useful when the work to be done is not definitely fixed at the time of making
the estimate.
(iii) Contractee can ensure himself about the 'cost of contract' as he is empowered to
examine the books and documents of the contractor to ascertain the veracity of the
cost of contract.

Question No. 1 (b)

Solution:
Apportionment of Joint Cost amongst Joint products using:

Market value at the point of separation


This method is used for apportionment of joint products upto the split off point. It is
difficult to apply if the market value of the product at the point of separation is not
available. It is useful method where further processing cost are incurred
disproportionately.

Net realizable value method


From the sales value of joint products ( (at finished stage) the following are deducted:
(i) Estimated profit margins
(ii) Selling & distribution expenses, if any
(iii) Post split off costs.
The resultant figure so obtained is know as net realizable value of joint products. Joint
costs are apportioned in ration of net realizable value.

Question No. 1 (c)

Solution:
Accounting treatment of idle time wages & overtime wages in cost account:
Normal idle time is treated as a part of the cost of production. Thus, in the case of direct
workers, an allowance for normal idle time is built in to the labour cost rates. In the case

© The Institute of Chartered Accountants of Nepal 1


Paper 5 : Cost and Management Accounting

of indirect workers, normal idle time is built into the labour cost rates. In the case of
indirect workers, normal idle time is spread over all the products or jobs through the
process of absorption of factory of factory overheads.

Under Cost Accounting, the overtime premium is treated as follows:


(i) If overtime is resorted to at the desire of the customer, then the overtime
premium may be charged to the job directly.
(ii) If overtime is required to cope with general production program or for
meeting urgent orders, the overtime premium should be treated as overhead
cost of particular department or cost center which works overtime.
(iii) Overtime worked on account of abnormal conditions should be charged to
costing profit & Loss Account.
(iv) If overtime is worked in a department due to the fault of another
department the overtime premium should be charged to the latter
department.

Question No. 1 (d)

Solution:

Cost classification based on variability


(a) Fixed cost – These are the costs which are incurred for a period, and which,
within certain output and turnover limits, tend to be unaffected by fluctuations
in the levels of activity (output or turnover). They do not tend to increase or
decrease with the changes in output. For example, rent, insurance of factory
building etc. remain the same for different levels of production.
(b) Variable Cost – These costs tend to vary with the volume of activity. Any
increase in the activity results in an increase in the variable cost and vice-versa.
For example, cost of direct labour, etc.
(c) Semi – variable Costs- These costs contain both fixed and variable
components and are thus partly affected by fluctuations in the level of activity.
Examples of semi variable costs are telephone bills, gas and electricity etc.

Material Control
Question No. 2 (a)

Solution:

Workings:
Annual production of Product X = Annual demand – Opening stock
= 5,00,000- 12,000 = 4,88,000 units
Annual requirement for raw materials = Annual production ˟ Material per unit- Opening
stock of material

© The Institute of Chartered Accountants of Nepal 2


Paper 5 : Cost and Management Accounting

Material A = 4,88,000 ˟ 4 units – 24,000 units = 19,28,000 units


Material B = 4,88,000 ˟ 16 units – 52,000 units = 77,56,000 units
i) Computation of EOQ when purchase order for the both material is placed
separately
EOQ = √2˟Annual Requirement for material ˟Ordering cost/ Carrying cost per
unit p.a.
Material A = √2˟19,28,000 units ˟ Rs. 15,000/ (13%of Rs. 150)
= √38,56,000 ˟ Rs. 15,000/ Rs. 19.5
= 54,462 units
Material B = √2˟77,56,000 units ˟ Rs. 15,000/ (13%of Rs. 200)
= √1,55,12,000 ˟ Rs. 15,000/ Rs. 26
= 94,600 units

ii) Computation of EOQ when purchase order for the both material is not
placed separately
Material A & B = √2˟(19,28,000+77,56,000) units ˟ Rs. 15,000/ (13%of Rs.
190*)
= √193,68,000 ˟ Rs. 15,000/ Rs. 24.7
= 108,452 units
Material A = (108,452 ˟ 19,28,000)/ 96,84,000
= 21,592 units
Material B = (108,000˟ 7756,000)/ 96,84,000
= 86,860 units
* (Rs. 150 ˟ 19,28,000) + ( Rs. 200˟ 77,56,000)/ (19,28,000+ 77,56,000)
= Rs. 190

Labour Control

Question No. 3 (a)

Solution:
Calculation of earnings
i) Differential piece rate system
= Actual Output ˟ Piece rate per unit ˟ 120%
= 120 Piece ˟ Rs. 4.00 ˟ Rs. 120%
= 120 ˟ Rs. 4.80 = Rs. 576
Efficiency = 120/100 ˟100 = 120 % i.e. above normal output, so 120% of piece
rate is applicable.

© The Institute of Chartered Accountants of Nepal 3


Paper 5 : Cost and Management Accounting

ii) Halsey Premium Scheme


= Time taken ˟ Rate + ½ ( Time Allowed – Time Taken ) ˟ Rate
= 45 hours ˟ Rs. 8 + ½ (( 45/ 100 ˟ 120) – 45 hours) ˟ Rs. 8
= Rs. 360 + Rs. ½ ( 54-45) ˟ Rs. 8.00
=Rs. 360 + Rs. 36
= Rs.396

Question No. 3 (b)

Solution:
i) Labour cost variance ( SH ˟ Std. Rate) – ( AH Paid ˟ AR)
((Rs. 40˟ Rs. 65)/Rs. 2,000 ˟ 1,800) ˟ Rs.45 – ( Rs. 50 ˟ Rs. 40 ˟ Rs. 50)
= ( Rs. 1,05,300 – Rs. 1,00,000)
= Rs. 5,300 ( F)
Labour Rate Variance = AH paid ( SR- AR)
= Rs. 2,000 ( 45-50)
= Rs. 10,000 (A)
Labour Efficiency Variance = SR ( SH-AH worked)
= Rs. 45 ( Rs. 2,340- 1,900)
= 19,800 (F)
Idle time variance = SR ˟ Idle time
=Rs. 45 ˟ Rs. 100
= Rs. 4,500(A)
ii) Reconciliation
Labour Cost Variance = Labour Rate Variance + Labour efficiency variance + Idle
time variance
= Rs. 10,000(A) + Rs. 19,800 (F) + Rs. 4,500(A)
= Rs. 5,300 (F)

Overhead
Question No. 4 (a)

Solution:

© The Institute of Chartered Accountants of Nepal 4


Paper 5 : Cost and Management Accounting

i) Amount of under absorption of production overheads during the period:


Particulars Amount ( Rs.) Amount ( Rs.)
Total production overheads actually incurred 4,84,250
during the period
Less: Expenses on accounting w/o obsolete 40,000
store and wages paid for the strike period
Net production overheads actually incurred 4,44,250
Less: Production overheads absorbed as per 3,84,250
machine hour rate ( 2,65,000 hours × Rs.1.45)
Amount of under absorbed production 60,000
overheads

ii) Accounting treatment of under absorbed production overheads:


As, 30% of the under absorbed overheads were due to factory inefficiency, this being
abnormal, hence should be debited to Costing Profit and Loss Account.
Amount to debited to Costing Profit and Loss Account = (60,000 × 30%)
= Rs. 18,000

Balance of under absorbed production overheads should be distributed over Works in


progress, Finished goods and Cost of sales by applying supplementary rate*.
Amount to be distributed = (60,000 × 70%) = Rs. 42,000

Supplementary rate = Rs. 42,000/ 20,000 units* = 2.10

Apportionment of under absorbed production overheads over WIP, Finished goods and
Cost of sales:
Particulars Equivalent Amount
completed units (Rs.)
Work-in- progress ( 5,000 units × 50% × 2.10) 2,500 5,250
Finished goods ( 5,000 units × 2.10) 5,000 10,500
Cost of sales ( 12,500 × 2.10) 12,500 26,250
Total 20,000 42,000

*( 17,500 units +1/2 of 5,000 units)


Journal Entry:
Particulars Dr. (Rs.) Cr. ( Rs.)
WIP control A/c Dr. 5,250

© The Institute of Chartered Accountants of Nepal 5


Paper 5 : Cost and Management Accounting

Finished goods control A/c Dr. 10,500


Cost of Sales A/c Dr. 26,250
Costing P/L A/c Dr. 18,000
To Overhead Control A/c 60,000

Question No. 4 (b)

Solution:
Particulars Amount (Rs.)
Total factory overheads incurred 51,00,000
Less: Absorbed factory overheads ( Rs. 30×1,50,000) (45,00,000)
Under - absorption of Overheads 6,00,000

60 % of Rs. 6,00,000 i.e. Rs. 3,60,000 would be transferred to Costing P/L Account
40 % of Rs. 6,00,000 i.e. Rs. 2,40,000 would be apportioned over Sales unit and Stock by
using supplementary overheads rate.
Supplementary overheads rate = Rs. 2,40,000/ ( 50,000+5,000+5,000)
= Rs. 4
On Sales ( 50,000 units × Rs. 4) 2,00,000
On Finished Goods ( 5,000× Rs. 4) 20,000
On Work in Progress ( 10,000 × 50% × Rs. 4) 20,000
240,000

Costs Accounts System, Cost Control (Integrated and Non-integrated Accounting System)

Question No. 5

Solution:
i) Costing Profit and Loss Account for the year ended 31st Ashadh, 2076:
Particulars Amount Particulars Amount
(Rs.) (Rs.)
Material Consumed 14,16,000 Sales ( 30,000 units) 30,00,000
Direct wages 7,42,000

© The Institute of Chartered Accountants of Nepal 6


Paper 5 : Cost and Management Accounting

Prime Cost 21,58,000


Works overheads (20% of Prime Cost) 4,31,600
25,89,600
Less: Work in Progress (54,000)
Factory cost 25,35,600
Administration Overheads ( Rs. 5 × 1,60,000
32,000 units)
Cost of production 26,95,600
Less: Finished stock (1,68,475)
Cost of goods sold 25,27,125
Selling and distribution overheads ( Rs. 1,80,000
6 × 30,000)
Cost of sales 27,07,125
Profit ( balancing figure) 2,92,875
30,00,000 30,00,000

ii) Statement reconciling the profit as per costing profit and loss account with the profit
as per financial accounts
Particulars Amount (Rs.) Amount (Rs.)
Profit as per cost records 2,92,875
Add: Overheads over - absorbed:
- Works overheads ( Rs. 4,31,600- Rs. 5,600
4,26,000)
- Administration overheads ( Rs. 160,000- 10,000
Rs. 1,50,000)
- Selling and Distribution ( Rs. 1,80,000- 15,000 30,600
Rs. 1,65,000)
Less: Closing stock overvalued ( Rs. 1,68,475- (975)
Rs. 1,67,500)
Profit as per financial accounts 3,22,500

• It is assumed that the number of units Produced


= Number of units sold +Finished stock = 30,000+2,000 = 32,000 units

© The Institute of Chartered Accountants of Nepal 7


Paper 5 : Cost and Management Accounting

Methods of Costing

Question No. 6 (a)


Solution:
Contract Account
Particulars Rs. Particulars Rs.
To Material Purchased 852,000 By Material ( at site) 38,000
To Wages 10,48,000
To Indirect expenses 92,000
To Administrative charges 1,18,000
Depreciation on 1,08,000
machine(WN-1)
Supervisor's salary 80,000 By Works cost c/f 22,60,000
(Rs.10,000˟12˟2/3) (balancing figure)
22,98,000 22,98,000
To Works cost b/f 22,60,000 By Value of work 22,50,000
certified (50% of
45,00,000
Notional profit c/f 5,55,000 By Cost of work 5,65,000
uncertified (working
Note 2)
28,15,000 28,15,000
To Costing P&L A/c 3,33,000 By Notional profit b/f 5,55,000
(Working Note 3)
To Work- in – progress 2,22,000
(Reserve)
5,55,000 5,55,000
Working Notes:
1. Depreciation = (Rs. 612,000/85%) ˟ 15% = Rs. 1,08,000
Opening value of plant = Rs. 6,12,000 + 108,000 = Rs. 7,20,000
2. The cost of 2/3 rd of the contract is Rs. 22,60,000
Cost of 100% of the contract is Rs. 22,60,000/2 ˟3 = Rs. 33,90,000
Cost of 50% of the contract which has been certified by the architect is Rs. 16,95,000.
Also, the cost of 1/3rd of the contract, which has been completed but not certified by the
architect is Rs. 5,65,000 (22,60,000-16,95,000=5,65,000)
3. 2/3˟ National Profit ˟Cash received/Work certified
Or,2/3˟ 5,55,000˟90%˟ 22,50,000/22,50,000 =Rs. 3,33,000

Question No. 6 (b)

Solution:
Statement of Equivalent Production
Process III
Input Details Units Output Particulars Units Material -A Material - B Labour &
Overhead
% Units % Units % Units
Opening WIP 1,600 Work on Op. WIP 1,600 - - 20 320 40 640

© The Institute of Chartered Accountants of Nepal 8


Paper 5 : Cost and Management Accounting

Process- II 55,400 Introduced & 50,600 100 50,600 100 50,600 100 50,600
Transfer completed during
the month
Normal loss (5% 2,640 - - - - - -
of 52,800 units
Closing WIP 4,200 100 4,200 70 2,940 50 2,100
Abnormal Gain (2,040) 100 (2,040) 100 (2,040) 100 (2,040)
57,000 57,000 52,760 51,820 51,300
Working note:
Production units = Opening units+Units transferred from process-II– Closing Units
= 1,600 - units+ 55,400 units – 4,200 units
= 52,800 units

Statement of Cost
Cost (Rs) Equivalent Cost per
units equivalent
units(Rs)
Material A (Transferred from previous 6,23,250
process)
Less: Scrap value of normal loss (2,640 (13,200)
units ˟ Rs. 5)
6,10,050 52,760 11.5627
Material B 2,12,400 51,820 4.0988
Labour 96,420 51,300 1.8795
Overheads 56,400 51,300 1.0994
9,75,270 18.6404

Statement of apportionment of process Cost


Amount Rs Amount Rs
Opening WIP Material A 24,000
Completed opening Material B (320 units˟ Rs. 1311.62
WIP units – 1600 4.0988
Wages (640 units ˟ Rs. 1.8795) 1202.88
Overheads (640 units ˟ Rs. 703.62 3,218.12
1.0994)
Introduced & 50,600 units ˟Rs. 18.6404 9,43,204.24
Completed -50,600
units
Total cost of 52,200 9,70,422.36
finished goods units
Closing WIP units - Material A (4,200 units ˟ Rs. 48,563.34
4,200 11.5627)
Material B (2,940 units ˟ Rs. 12,050.47
4.0988)
Wages (2,100 units ˟ Rs. 3,946.95
1.8795)
Overheads (2,100 units˟ Rs. 2,308.74
1.09940
66,869.50
Abnormal gain units (2,040 units˟ Rs 18.6404) 38026.42
2,040

© The Institute of Chartered Accountants of Nepal 9


Paper 5 : Cost and Management Accounting

Process III A/c


Particulars Units Amount Particulars Units Amount(Rs.)
(Rs.)
To Balance b/d 1,600 24,000 By Normal loss 2,640 13,200
To Process II A/c 55,400 6,23,250 By Finished 52,200 9,70,422.36
goods
To Direct 2,12,400 By Closing WIP 4,200 66,874.06*
material
To Direct wages 96,420
To production 56,400
overheads
To Abnormal 2,040 38,026.42
gain
59,040 10,50,496.42 59,040 10,50,496.42
*Difference in figure due to rounding off has been adjusted with closing WIP.

Question No. 6 (c)

Solution:
i) Statement showing allocation of Joint Cost
Particulars P Q
No. of units Produced 2,500 1,500
Selling Price Per unit ( Rs.) 80 50
Sales Value ( Rs.) 2,00,000 75,000
Less : Estimated Profit ( P-30% & Q- 25%) (60,000) (18,750)
Cost of Sales 1,40,000 56,250
Less: Selling Expenses ( Ref. W.N. - 1) (25,000) (15,000)
Cost of Production 1,15,000 41,250
Less: Cost after separation (60,000) (30,000)
Joint Cost allocated 55,000 11,250

ii) Statement of Profitability


Particulars M ( Rs.) P ( Rs.) Q ( Rs.)
Sales Value (A) 7,65,000 2,00,000 75,000
(4,500×Rs. 170)
Less: Joint Cost 183,750 55,000 11,250
( 2,50,000-
55,000-11,250)

© The Institute of Chartered Accountants of Nepal 10


Paper 5 : Cost and Management Accounting

- Cost after separation - 60,000 30,000


- Selling Expenses 45,000 25,000 15,000
( Ref. W.N. -1)
(B) 2,28,750 1,40,000 56,250
Profit (A-B) 5,36,250 60,000 18,750
Overall Profit = Rs. 5,36,250+ Rs. 60,000+ Rs. 18,750 = Rs. 6,15,000

iii) If the by -product P is not further processed and is sold at the point of separation
Amount ( Rs.)
Sales value at the point of separation 1,50,000
( 2,500 units × Rs. 60)
Less : Joint Cost 55,000
Profit 95,000
Profit after further processing 60,000
Incremental Profit 35,000

If the by - product P is sold at the point of separation, it will give an additional profit
of Rs. 35,000 to the company, hence the company should sell by - product P without
further processing.

Working Note:
1. Apportionment of selling expenses among M,P and Q
Product M- Rs. (85,000/17) ×9 = Rs. 45,000

By - product P - ( Rs. 85,000/17) ×5 = Rs. 25,000

By- product Q - ( Rs. 85,000/17) × 3 = Rs. 15,000

Question No. 6 (d)

Solution:
a) Calculation of cost and amount chargeable by the Contractor
Particulars Veg. Non-Veg
No. of Meals per Day 180 120
No. of Meals per Month 180×25 = 4,500 120×25 = 3,000
Variable Cost: Rs. Rs.

© The Institute of Chartered Accountants of Nepal 11


Paper 5 : Cost and Management Accounting

Cereals 8 per plate


Veg Items 5 per plate
Cooking Oil 4 per plate
Spices 1 per plate
Total Variable Cost 18×7500(4500+3000 1,35,000
Additional variable cost of Non-veg meal 15 ×3000 45,000
Total Variable Cost 180,000
Fixed Cost:
Salary of Cook 13,000
Salary of Helpers ( 7,000×3) 21,000
Fuel 2,000 36,000
Total Cost 2,16,000
Profit 20% on his taking or 25 % on Cost 54,000
Total amount chargeable by the Contractor 2,70,000

i) No. of Non- Veg Meals 3,000


Equivalent No. of Veg Meals = 3,000×1.5 = 4,500
No. Of Non Veg Meals = 4,500
Total =9,000
Price per Veg Meal = Rs. 270,000/Rs. 9,000 = Rs. 30
Price per Non Veg Meal = Rs. 30×1.5 = Rs. 45
ii) Price per meal when a worker will have to pay
Veg meal Rs. 30 - Subsidy ( 60% of Rs. 30)
= Rs. 30- Rs. 18 = Rs. 12
Non - Veg Meal Rs. 45 - subsidy ( 60% of Rs. 45)
Rs. 45-Rs. 27 = Rs. 18
Note: Cost of Veg and non -veg meal calculated separately and then profit of 20%
on overall takings and 25% of profit on overall Cost is added to determine the total
price to be charged.

Cost Concepts for Decision Making


Question No. 7

Solution:
Particulars Factory X Factory Y
i) Break Even Point:

© The Institute of Chartered Accountants of Nepal 12


Paper 5 : Cost and Management Accounting

Fixed Cost/Contribution 2,00,000/(50-40) 3,00,000/(50-35)


=20,000 units =20,000 units
ii) Cash Break Even point:
(Fixed cost- Depreciation)/Contribution
(2,00,000-40,000)/10 (3,00,000-30,000)/15
= 16,000 units = 18,000 units

iii) BEP as a whole = Complete Fixed Cost/Composite Contribution


= (Rs. 2,00,000+ Rs. 3,00,000)/ ( 10×3/5 + 15×2/5)
= Rs. 5,00,000/(6+6)
= 41,667 units
iv) New Sales Mix = 50,000*2/5 = 20,000 of X
= 50,000*3/5 = 30,000 of Y
Calculation of Composite contribution = 10×2/5 +15*3/5
= 4+9 = Rs. 13
Consequence on profit
Existing Mix New Mix
Contribution 50,000×12 = 6,00,000 50,000×13 = 6,50,000
Less: Fixed Cost 5,00,000 5,00,000
Profit 1,00,000 1,50,000

So, Increase in Profit = Rs. 1,50,000-1,00,000 = Rs. 50,000


Consequence on BEP
New BEP as a whole = Complete Fixed Cost/Composite Contribution
= 5,00,000/13
=500,000/13 = 38,462 units
So, BEP Reduced by 3,205 units (41,667-38462)

Costing for planning and Control –Budgets


Question No. 8

Solution:

i) Production Budget of 'X' for the Second Quarter


Particulars Bags (Nos.)
Budgeted sales 50,000

© The Institute of Chartered Accountants of Nepal 13


Paper 5 : Cost and Management Accounting

Add: desired Closing stock 11,000


Total Requirement 61,000
Less: Opening Stock 15,000
Required Production 46,000

ii) Raw- Materials Purchase Budget in Quantity as well as in Rs. for 46,000 Bags
of 'X'
Particulars 'Y' 'Z' Empty Bags
Nos.
Production Requirements Per 2.5 7.5 1.0
bag of 'X'
Requirement for Production 1,15,000 3,45,000 46,000
(46,000×2.5) (46,000×7.5) (46,000×1)
Add: Desired Closing Stock 26,000 47,000 28,000
Total Requirements 1,41,000 3,92,000 74,000
Less: Opening Stock 32,000 57,000 37,000
Quantity to be purchased 1,09,000 3,35,000 37,000
Cost per Kg./Bag Rs. 120 Rs. 20 Rs. 80
Cost of Purchase (Rs.) 1,30,80,000 67,00,000 29,60,000

iii) Computation of Budgeted Variable Cost of Production of 1 Bag of 'X'


Particulars Rs.
Raw - Material
Y 2.5 Kg. @ 120 300.00
Z 7.5 Kg. @ 20 150.00
Empty Bag 80.00
Direct Labour ( Rs. 50 × 9 minutes/ 60 minutes) 7.50
Variable Manufacturing Overheads 45.00
Variable Cost of Production per bag 582.50

Standard Costing
Question No. 9 (a)
Solution:

Reason for Preference of Standard Costing.


Standard costing system is widely accepted as it serves the different needs of an
organization. The standard costing is preferred for the following reasons:
(a) Predication of future cost for decision making: Standard cost are set after
taking in to account all the future possibilities and can be termed as future cost.
Standard cost is used for calculating profitability from a project/ order/activity
proposed to be undertaken. Hence, standard cost is very useful for decision
making purpose.
(b) Provide target to be achieved: Standard costs are the target cost which should
be no be crossed.

© The Institute of Chartered Accountants of Nepal 14


Paper 5 : Cost and Management Accounting

(c) Used in budgeting and performance evaluation: Standard cost are used to
set budgets and based on these budgets managerial performance is evaluated.
(d) Interim profit measurement and inventory valuation: Few organizations
used to prepare profitability statement for some interim periods as per the
requirement of the management. To arrive at the profitability figure standard
cost are deducted from the revenue.

Question No. 9 (b)

Solution:
a) Material Price Variance:
= ( Standard price - Actual Price) × Actual quantity
= ( Rs. 4 - 4.10) ×5,000 = Rs. 500 Adv.
b) Material usage Variance:
= (Std. quantity of actual output - Actual quantity) × Std. price
= ( 600× 5 - 3,500) ×4 = Rs. 2,000 Adv.
c) Labour Rate Variance:
( Standard Rate - Actual rate) × Actual hours
= ( Rs. 10- Rs. 9 ) × 1,700 = Rs. 1,700 Fav.
d) Labour Efficiency Variance:
= ( Standard hours for actual output - Actual hours) × Standard Rate
= ( 600 × 3 - 1,700) × 10
= Rs. 1,000 Fav.
e) Variable Overhead Expenditure Variance:
= ( Actual Hours × Standard Rate) - Actual Overhead
= ( 1,700 × Rs. 1 ) - Rs. 1,900
= Rs. 200 Adv.
f) Variable Overhead Efficiency Variance:
= Std. hours for Actual output - Actual hours) × Std. Rate
= ( 600× 3 - 1,700) × 1 = Rs. 100 Fav.
g) Fixed Overhead Expenditure Variance:
= ( Budgeted overhead - Actual Overhead)
= ( 1,800× 0.50 - 900) = Nil
h) Fixed Overhead Volume Variance:
= ( Std. hours for actual output - Budgeted hours) × Std. rate
= (600 × 3 - 1,800) × Rs. 0.50 = Nil
i) Fixed Overhead Capacity Variance:
= ( Budgeted hours- Actual Hours) × Standard Rate
= ( 1,800- 1700) × Rs. 0.50 = Rs. 50 Adv.
j) Fixed Overhead Efficiency Variance:
= ( Std. hours for actual output - Actual hours) × Standard rate
= ( 600 × 3 - 1,700 ) × Rs. 0.50 = Rs. 50 Fav.

© The Institute of Chartered Accountants of Nepal 15


Paper 5 : Cost and Management Accounting

Verification Rs. Rs.


Overhead recovered : 600 units @ Rs. 4.50 2,700
Actual Overhead:
Variable 1,900
Fixed 900 2,800
100 Adv.
Variable expenditure variance 200 Adv.
Variable Efficiency variance 100 Fav.
Fixed expenditure variance Nil
Fixed overhead volume variance Nil
100 Adv.

Reconciliation Statement
Standard Cost : 600 units @ Rs. 54. 50 32,700
Actual Cost: 38,600
Less : Material Stock at standard cost: ( 6,000 (32,600) 100 Fav.
1,500× Rs. 4)
Variances: Adv. ( Rs.) Fav ( Rs.)
Material Price 500
Material usage 2,000
Labour Rate 1,700
Labour efficiency 1,000
Variable expenditure 200
Variable efficiency 100
Total 2,700 2,800 100 Fav.

© The Institute of Chartered Accountants of Nepal 16


Paper 5 : Cost and Management Accounting

Uniform Costing and Inter-firm comparison


Question No. 10 (a)

Solution:

Uniform costing is the application of the same accounting and costing principles, methods or

procedures uniformly by various undertakings in the same industry. It is a particular technique

which applies the usual accounting methods like standard costing, marginal costing, and

budgetary control.

Uniform costing method can be advantageously applied:

- In single organisation having number of branches.

- In a number of firms in the same industry who are inter connected through trade association.

- In industries which are similar such as cotton, gas and electricity.

Question No. 10 (b)

Solution:

The followings are the per-requirements for meaningful and effective inter firm or intra firm
comparison.

1. Similarity of Firms or Departments


Similarity means an age, size of business unit or department, character of production, nature
of forms of business organization and the market in which the business units or departments
are catering should be the same. For example, a newly started company cannot be compared
with 10 years old company. Partnership firm cannot be compared with public limited
company.

2. Use of Accounting Ratios


Money values are not useful for comparison. Standard accounting ratios are calculated in
order to find out the strength and weakness of the business unit. Same type of accounting
ratios should be selected for inter firm comparison.

3. Similarity in Accounting Policies


The selected comparing business units should have uniformity in the use of accounting
policies regarding valuation of closing stock, method and rate of depreciation, provision of
gratuity, purchase policy, sales policy and the like. If not so, the comparison does not give
meaningful results.

4. Adjustments for Inflation


Before calculating the ratios, an analyst should consider the inflation. If so, correct reasons
may be find out through comparison and follow corrective actions.

© The Institute of Chartered Accountants of Nepal 17


Paper 5 : Cost and Management Accounting

Cost control and cost reduction


Question No. 11

Solution:

Cost control has the following advantages:


(i) It helps the firm to improve its profitability and competitiveness.
(ii) In the absence of cost control, profits may be drastically reduced despite a large and
increasing sales volume.
(iii) It is indispensable for achieving greater productivity.
(iv) Cost control may also help a firm in reducing its costs and reduce its prices.
(v) If the price of the product is stable and reasonable, it can maintain higher sales and
thus employment of work force.

© The Institute of Chartered Accountants of Nepal 18


Paper 6A : Business Communication

Paper 6A: Business Communication

© The Institute of Chartered Accountants of Nepal 1


Paper 6A : Business Communication

Revision Questions
Chapter 1
Q.N.1. Describe briefly the process of communication that takes place in a business
organization. 10

Q.N. 2. Write short notes on the following. (4*2.5=10)


a) Ethics in business communication
b) Ethical dilemmas and ethical lapses
c) Overcoming group problems
d) Corporate social responsibility

Q.N.3. Define business communication? Discuss different techniques of enhancing


business communication skills. 10

Chapter 2
Q.N.4. What are the important considerations for presentation of information in a
business meeting? Discuss them in terms of the aspects such as business ethics, group
interactions, listening skills, agenda and purpose of meeting. 10

Q.N.5. Negotiation is taken as an important technique of conflict resolution. Define and


describe its procedures. 10

Chapter 3

Q.N.6. What is workforce diversity? Discuss the role of Cross cultural communication in
the diverse workplaces of today’s contexts. 10

Q.N. 7. What is non-verbal communication? What’s the role of graphics in business


communication? Discuss. 10

Chapter 4
Q.N. 8. Write short notes on the following: (4*2.5=10)
a. Paralinguistic features during job interview
b. Stages of job interview
c. Resume
d. Role of interviewer

Q.N. 9. Read the case below, and answer the questions that follow. (4*5=20)

© The Institute of Chartered Accountants of Nepal 2


Paper 6A : Business Communication

Shyam Suwal is a fresh but qualified candidate for the advertised post that he has recently learnt
about through the website of a bank. He has an American degree in business management. He
writes the following e-mail to the prospective employer.

From: discover.ssuwal123@gmail.com
To: himalayan_bank.org.com
subject: Application for the post of IR officer
Date: April 12, 2020

Dear sirs,

I came to learn from your website that your bank, a leading banking institution in
Nepal has been searching for a dynamic resource person as an international
relationship officer. It's a big matter of pleasure for me that I have completed my
MBA degree from a reputed US university with specialization in the international
banking. I had done exactly same nature of job during my internship program as you
have required now. I did my internship at Global Bank in New York for a year under
close supervision of my university professors and bank administrators. I'm fully
convinced that I can be the fittest candidate for your requirement.

Regards,
Shyam Suwal

Questions:
a) Do you think that the given e-mail illustrates one of the effective models of job
application? Why?
b) Before a qualified candidate writes an application for a job, how does he prepare himself
to appropriately respond to the vacancy announcement?
c) Mr. Suwal tries to hide the fact that he does not possess sufficient experience for the job
that he is going to apply. How does he persuade the employer despite this limitation? Do
you agree with his technique?
d) Suppose the employer responded him through an e-mail with some positive remarks
about his application. But, it has been more than one month that the employer has called
neither for interview nor for any other types of tests. Now, as Mr. Suwal, write a follow up
letter to the employer.

Chapter 5

Q.N. 9. What is an analytical report? How is information organized in an analytical


report?

© The Institute of Chartered Accountants of Nepal 3


Paper 6A : Business Communication

Q.N. 10. What is a work plan? What is its significance in a research proposal? Prepare
a sample work plan for your research proposal.10

Q.N. 11. What is an abstract? Write a brief abstract of a business report. 10

© The Institute of Chartered Accountants of Nepal 4


Paper 6A: Business Communication

Answers/ Hints:

Chapter 1

Q.N.1. Describe briefly the process of communication that takes place in a business
organization. 10

Answer:

Communication is a process of exchanging ideas and information between people. It takes place
in different modes and contexts. In business organizations people do various transactions with
various roles basically through communication. Organizational goals are unlikely to be attained
unless there's no effective communication. The process of organizational communication
involves a typical process that basically involves sender, channel and receiver.

In the first step, the sender has an idea, and he or she encodes that idea into message through
linguistic codes or symbols. This process is called encoding. Ideas are encoded into verbal
symbols such as words and sentences. They may be encoded into nonverbal symbols such as
signs, body language, etc. Both verbal and nonverbal codes can also be used simultaneously.
Then in the second step, the encoded message travels through the channel such as air web,
telephone, media, etc. and arrive at the auditory system of the receiver. Then, the receiver
decodes the message. He or she makes right sense of the message. If decoding is not accurate
there can be misunderstanding in communication. The receiver may then take the role of sender.
This process continues until the transaction is successful.

Q.N. 2. Write short notes on the following. (4*2.5=10)


a) Ethics in business communication
b) Ethical dilemmas and ethical lapses
c) Overcoming group problems
d) Corporate social responsibility

Answers:

a) Ethics in business communication generally refers to the set of principles guided for good
conduct of business dealings. They govern a business person or a group so that trust can be
derived from communication as well as from transaction. Ethical people are perceived by
consumers and others as trust worthy, fair and impartial, respecting the rights of others and
showing concern about the impact of their actions on the society. They usually obey the
communicative maxims of cooperative and politeness principles. Ethical communication can
obviously lead the business activities towards success and perfection. Ethical communication
includes all relevant information that excludes false traps and tricks. The massage is true in
every sense, and is not deceptive in any way. In contrast, unethical communication can include
falsehoods and misleading information. Ethical communication is a major key for the success of
a business transaction.

© The Institute of Chartered Accountants of Nepal 1


Paper 6A: Business Communication

b) Ethical behavior is a companywide concern, of course, but every company has


responsibilities to its each stakeholder. In some situations, what is right for a group or a person
can be wrong for another. There can be many alternative solutions for a particular issue, but
they cannot be equally favorable for all people. In such situations, ethical people may not be
able to tell the truth or to take absolutely right decisions. They're forced to think about a better
choice among many different valid alternatives. This is known as ethical dilemma. On the other
hand, the term 'an ethical lapse' refers to a clearly unethical or illegal choice, When a person
(e.g. an official) or a group knows that something is wrong, and yet does it anyway, it is known
as ethical lapses.

c) Overcoming group problems


Working in groups may invite a number of problems though it is believed that team work
contributes to attaining the organizational goals and missions. The group problems are to be
settled so that the expected outcomes can be ensured. Some of the important strategies for
overcoming the group problems are:
• Working with group spirit and identity;
• Discouraging personal skepticism, lobbying, influences and thoughts;
• Training on group autonomy and group dynamism;
• Generating the sense of cooperation, endurance and collaboration…

d) Corporate social responsibility (CSR)


Corporate social responsibility (CSR) is an essential component of an ethical and responsible
business organization. It is a business movement in which commercial organizations address the
social issues identified through different sources, and run the programs for social welfare. They
invest certain amount of money so that their business as well as the society where they have to
survive can grow together. Different infrastructural activities, educational programs, public
health programs, sports events etc. are conducted by the business organizations as their
responsibility to the society.

Q.N.3. Define business communication? Discuss different techniques of enhancing


business communication skills. 10

Answer:

Business communication is defined as a specific variety of communication that is used in the


business transactions. The success of business organizations depend upon many different
factors, one of which is obviously business communication. The skilled business personnel need
to develop in them appropriate business skills so that they can communicate successfully and
get things done accurately. There are various strategies identified for business persons to
develop communication skills. Basically, they need to develop the habit of continuous reading
and writing about their related field, and they need to take part in the interaction programs on
how to make communication more effective. Other common strategies include:

• Listening attentively, cooperatively and actively;

© The Institute of Chartered Accountants of Nepal 2


Paper 6A: Business Communication

• Interacting politely, cooperatively and contextually;


• Behaving with full sense of business ethics;
• Learning to use codes appropriately according to socio-cultural norms and perspectives;
• Developing team spirits, group interactions, conferences and sharing;
• Self-monitoring and collaborating activities;
• Learning to communicate through written as well as electronic modes with accurate
format;
• Communication trainings on intercultural communication and workforce diversity;
• Learning to avoid communication barriers such as mental barriers, physical barriers and
semantic barriers;
• Developing the habit of keeping up-to date knowledge through reading and writing.

Chapter 2

Q.N.4. What are the important considerations for presentation of information in a


business meeting? Discuss them in terms of the aspects such as business ethics, group
interactions, listening skills, agenda and purpose of meeting. 10

Answer:

Presentation in a business meeting requires special skills and strategies for a researcher or a
presenter. It is one of the important forms of business communication. The presenter needs to
take care of several points related with audience, agenda and purpose of meeting.

• It's not ethical in business to falsely present the data in an inconsistent manner. The false
information must be strictly avoided from the presentation and discussion.
• During your presentation you must not look nervous and unplanned. You need to carefully
handle the group interaction with proper turns, arguments and negotiations.
• You need listen to the comments and queries of the other members, but should not impose
your own thoughts unnecessarily.
• Group presentation is not a way of teaching. It’s a type of sharing. So, you need to create
the environment for meaningful discussions.
• Be relevant to the negotiated agenda, and remain patient to listen to other members, too.
• Always be sure that you lead the meeting to its effective conclusion according to the
purpose.

© The Institute of Chartered Accountants of Nepal 3


Paper 6A: Business Communication

Q.N.5. Negotiation is taken as an important technique of conflict resolution. Define and


describe its procedures. 10

Answer:

Conflict is usually regarded as an asset of a living organization. But it must be settled in time
before it invites disaster within the organization. There are several effective techniques for
conflict resolution, including negotiation, mediation, reconciliation, etc. Negotiation is often
taken as a very useful and effective process of conflict resolution. It helps people to eliminate the
basis for conflicts through bilateral discussions, dialogues and compromise. It is the most
preliminary stage in the process for the conflict resolution. The conflicting parties attempt to
negotiate in a common point of understanding. They reflect and judge their own positions and
have a motive of coming to an effective conclusion. Unlike in mediation, there is no clear role of
the third party to bring the conflicting parties into negotiation point, but if there is any trained
agency for creating negotiation, the process of conflict resolution becomes much easier.

It refers to the common effort made by two parties intending to minimize the conflict between
the two. It aims to settle the dispute through intra-group facilitation, compromise, mutual
understanding and co-ordination. The two conflicting parties are required to go through the
situation with certain critical reflections, and they are kept together face-to-face with a kind of
realization about the situation. They are ready to reach the solution and get involved in the
negotiation process. They have open discussion with the motive of negotiation. They try to
make ‘give and take’ results on one hand, and on the other they try to compromise upon certain
bottom line of their views and positions. They reach the ‘win-win’ situation.

Chapter 3

Q.N.6. What is workforce diversity? Discuss the role of Cross cultural communication in
the diverse workplaces of today’s contexts. 10

Answer:

The increasing diversity in the workplace poses challenges to the workers as well their
managers. Along with globalization of business, workplace diversity in the workplace has
become a burning issue in the sector of business and management. Workforce diversity refers to
the diverse situation of the workers in an organization derived from their socio-cultural and
national identities, backgrounds, and behaviors. When people and products move across the
borders, the workplace can be diverse enough because of norms, age, gender, values, education,
conventions, etc. of the workers. People who grew up in the same ethnic and cultural
background are most likely to share the similar patterns of social behavior in their workplace
too. In multinational companies people from different backgrounds might have different ways
of perception, understanding and behaving. Such diversity may invite many problems including
conflicts and misunderstandings. Proper communication can minimize the potential drawbacks
of workforce diversity.

© The Institute of Chartered Accountants of Nepal 4


Paper 6A: Business Communication

As solution has a phenomenon emerged quite lately, i.e. intercultural communication which is
also known as cross-cultural communication. It refers to the way of communication between
the people of different cultures, without posing any kind of threat to any kind of culture. The
international languages and their nativised varieties are commonly used as effective means of
intercultural communication. Basically, the workers of a business house are given trainings on
intercultural norms, values, beliefs and practices. And, they are given guidelines to
communicate each other in line with the perspectives of different cultures.

Q.N. 7. What is non-verbal communication? What’s the role of graphics in business


communication? Discuss. 10
Answer:
Communication can be of two types according to the nature of its means: verbal and non-verbal.
When the means of communication is words or utterances, it’s known as verbal communication.
If the means is not the words but the signs and symbols, the communication is regarded as non-
verbal one. Different forms of this type of communication can be signs, signals, symbols,
gestures, body postures, facial expressions, etc. Non-verbal communication is equally important
to verbal one to express accurately our feelings, attitudes and intentions. In business
communication too, non-verbal devices have crucial role particularly for persuasion, request,
offer, and other several business transactions.
Graphics is one of the highly effective non-verbal tools which is commonly used in business
communication. It refer to different designs, drawings or pictures that we keep in our power
point slides, advertisements, business texts, brochures, instructions, manuals, etc. The
usefulness of graphics in business communication can never be underestimated since graphic
representation of information becomes not only clear and precise but also impressive and
persuasive. It is commonly believed that a picture is worth thousand words. Line graphs,
histograms, bar charts, pie charts, figures, etc. are the common examples of graphics. These
present content more precisely, clearly and accurately than verbal descriptions.

Chapter 4
Q.N. 8. Write short notes on the following: (4*2.5=10)
a. Paralinguistic features during job interview
b. Stages of job interview
c. Resume
d. Role of interviewer

Answers:
a) Paralinguistic features during job interview
Paralinguistic features are those which are not purely the linguistic ones, but specific quality
of the linguistic elements such as words and utterances. Paralinguistic features include
quality of voice, speed of utterances, volume of speech, clarity in talk, and so on. During
interview, linguistic as well as paralinguistic features play an important role in creating the

© The Institute of Chartered Accountants of Nepal 5


Paper 6A: Business Communication

impressions of the interviewee. In the interview the following types of paralinguistic skills
need to be maintained by the interviewee:
• Clarity of sounds, and pronunciation of words and utterances
• Appropriate variations of volume of speech
• Appropriate pauses and speed in discourse
• maintenance of supra-segmental features such as pitch, stress and intonation

b) Stages of job interview

Job interview, an important component of employment communication, displays distinctive


features from other types of interviews. It is usually carried out in three different stages:
warming-up stage, question-answer stage and summing-up stage.
The warming-up stage refers to the stage that exists before the main conversation between the
interviewer and interviewee. In this stage both interviewer and interviewee attempt to be
prepared to lead the interview to a successful communication. The interviewee is prepared with
cheerful appearance and positive thoughts and expectations. The interviewer attempts to make
some kind of attachment with the interviewee with the help of gestures, welcome note, etc.
Similarly, in the question-answer stage, the content based interaction between the interviewee
and the interviewer takes place. This is the largest stage of job interview. The final stage is
known as summing-up stage in which the interviewer signals pre-closing of the conversation.
And finally, the interview gets wrapped up with conventional thank-you note and farewell
exponents.

c) Resume
Resume is one of the important written device of employment communication. Actually it is
a persuasive document which states clearly qualities and qualifications of a person who
applies for a certain post. It is also called bio-data or curriculum-vita (CV). The resume
should show all the strength of the candidate for any job so that the employer can decide the
fittest one among many. So, it should be persuasive in style and content. But, it must not
include false and exaggerated information. It is required to be precise and clear. Usually it
includes academic qualification, experiences, skills, abilities, personal details, and so on. A
candidate should prepare the CV according to the requirement and focus of the job and the
employer.

d) Role of interviewer in a job interview

Interviewer is a person who takes an interview to a candidate for a job. He/she asks
questions, and usually is the employer or any personnel assigned by the employer. The role
of such responsible person must be very effective in the communication event, i.e. job
interview. The interviewer should be focused on finding out the effectiveness and suitability
of the candidate for the given job. So, the questions asked by him/her must be relevant,
contextual and valid. The focus should be concentrated on finding out whether the claimed
features and qualities are there in the candidate or not. Today’s interviewers are more
conversational and friendly to the candidates. They should create for the candidates contexts
for better expressions and interactions.

© The Institute of Chartered Accountants of Nepal 6


Paper 6A: Business Communication

Q.N. 9. Read the case below, and answer the questions that follow. (4*5=20)

Shyam Suwal is a fresh but qualified candidate for the advertised post that he has recently learnt
about through the website of a bank. He has an American degree in business management. He
writes the following e-mail to the prospective employer.

From: discover.ssuwal123@gmail.com
To: himalayan_bank.org.com
subject: Application for the post of IR officer
Date: April 12, 2020

Dear sirs,

I came to learn from your website that your bank, a leading banking institution in
Nepal has been searching for a dynamic resource person as an international
relationship officer. It's a big matter of pleasure for me that I have completed my
MBA degree from a reputed US university with specialization in the international
banking. I had done exactly same nature of job during my internship program as you
have required now. I did my internship at Global Bank in New York for a year under
close supervision of my university professors and bank administrators. I'm fully
convinced that I can be the fittest candidate for your requirement.

Regards,
Shyam Suwal

Questions:
a) Do you think that the given e-mail illustrates one of the effective models of job
application? Why?
b) Before a qualified candidate writes an application for a job, how does he prepare himself
to appropriately respond to the vacancy announcement?
c) Mr. Suwal tries to hide the fact that he does not possess sufficient experience for the job
that he is going to apply. How does he persuade the employer despite this limitation? Do
you agree with his technique?
d) Suppose the employer responded him through an e-mail with some positive remarks about
his application. But, it has been more than one month that the employer has called neither
for interview nor for any other types of tests. Now, as Mr. Suwal, write a follow up letter to
the employer.

Answers

a) The given e-mail is effective means for employment communication in many respects
such as accurate format, conversational tone, precise presentation, etc., but it cannot be a model
for a job application. There are several reasons for this e-mail not being a model. For example:
• It does not clearly state the qualification, specialization and experience of the candidate.
The mentioned qualification, specialization and experience are too sketchy, but not persuasive.

© The Institute of Chartered Accountants of Nepal 7


Paper 6A: Business Communication

• It does not state the commitment of the candidate towards the prospective job.
• The exclusive talent and ability of the candidate are not mentioned at all.
• The contact number is not given. The resume is not attached, either.

b) Before applying for a job, the fresh candidates are required to adopt a number of useful
strategies and considerations. Most primarily, they need to identify the appropriate job for them
individually. In the given case of Mr. Suwal might have identified the nature of this job. Then,
he might have attempted to find some matches between his educational and personal strengths
and the nature of the prospective job. Actually, one needs to judge his or her own specific
qualities, distinct personal competencies, qualifications, communication skills, etc. before
applying for a job.

Then, a candidate needs to prepare a persuasive resume and submit to the employer along with a
short persuasive application. He is required to prepare other application documents such as
reference letters, academic certificates, character certificates, recommendation letters,
experience letters, internship certificate, and so on. More importantly, the person needs to
develop effective interview skills.

c) In the given case, the candidate is trying to hide the fact that he is inexperienced. But the
application looks persuasive. He has mentioned the internship experience. His term ‘ exactly
same nature of job’ is most persuasive, and it may convince the prospective employer about his
competencies in the given job. The name of the American bank is also an additional attraction
in the application. He has also mentioned that he accomplished his duties under the supervision
of university professors and bank officers. This also may accurately ensure the employer about
his performance. I agree with this technique of persuasion, but it would have even been better if
he had mentioned duration of internship and job specification.

d)

New Baneshwor, Kathmandu

May 2, 2020

Sonu Rokaya
Administrative Director
Himalayan Bank Limited
Kathmandu, Nepal

Subject: Follow-up letter

Dear Ms. Rokaya,

I had submitted you a letter of application through an e-mail responding to your online
advertisement for the post of IR Officer. I appreciate your way of responding the email with
detail information and feedbacks. I received a couple of emails from you in which you had

© The Institute of Chartered Accountants of Nepal 8


Paper 6A: Business Communication

shown a favor with my qualifications and skills. I submitted the additional documents that you
had asked for. However, I have not received any response from you now for more than a month.
Would you please inform me how the process of recruitment is getting on?

Sincerely yours,
……………..
Shyam Suwal

Chapter 5
Q.N. 9. What is an analytical report? How is information organized in an analytical
report?

Answer:

An analytical report is usually a research report. It is also called investigative report. It is


prepared on the basis of the information obtained from respondents of the related field. It
requires basically the research tools such as questionnaires, interview, focused group discussion,
observation report, tests, discourse analysis, etc. Scientific analysis and possible interpretations
of the data are made in this type of report. The basic components of an analytical report are:
introduction, background, statement of problem, objectives, methodology, analysis and
interpretation, findings, and recommendations.

The information is the data for your report. It leads one to the conclusion of the identified issue.
So it needs to be organized and analysed so accurately and systematically in the process of
research and report writing. The information can be organized systematically by categorizing it
into different themes such as type of investment, role of IT, desires to take risk, need for
changes, and so on. Then the information is tabulated to ensure more systematic data for the
report. The data can be organized and analysed using tables, graphs, statistical tools such as
percentage, mean, standard deviation, etc. The irrelevant ideas/options are avoided from
analysis. While organizing the data, the extremely high level desires are arranged in one part,
the average level of remarks in the other, and the low level desires for changes and challenges
are organized in the different sub-section.

Q.N. 10. What is a work plan? What is its significance in a research proposal? Prepare
a sample work plan for your research proposal.10

Answer:

Proposal is a systematic plan for a research or for an action to be accomplished. A reliable and
systematic work plan needs to be presented in the proposal about how to carry out the research
or action within a given period of time. In other words, the work plan indicates exactly the
schedule for accomplishing every individual task within the project. It describes how what must
be done will be accomplished. More specifically, the work plan includes: when the work will
begin, how it will be divided into stages, when it will be finished, what methods or resources

© The Institute of Chartered Accountants of Nepal 9


Paper 6A: Business Communication

will be used, and so on. It makes research systematic, timely and purposeful. It gives clear
schedule for the researcher to do what and when. Here is a sample of work plan for a research
proposal.

Actions Time duration


Conceptualization and identification of issue 2 weeks
Setting objectives and writing a proposal 1 week
Preparing tools 1 week
Sampling the population 1 week
Data collection 3 weeks
Data analysis and presentation 2 weeks
Writing report 1 week
Total 11 eeks

Q.N. 11. What is an abstract? Write a brief abstract of a business report. 10

Answer:

Abstract is a component of a business report. It is also seen in the business seminar papers. It is
a very short summary of the report. It appears in the very beginning part of the report. It usually
presents the primary goal of the report. It also includes the major points on introduction,
method, results and discussion, but in a very precise form. The readers readily know the gist of
the report when they read the abstract, so it helps them to decide whether to continue reading
that report or not.
Usually, the abstract of a short report has two paragraphs or parts. In the first part, the topic,
objectives, methods and major findings are precisely presented, and in the second part, the
chapter wise summary is presented. The following text is an example of an abstract of a
business report.

The research report entitled ‘consumers’ attitudes on Honda bike’ is basically aimed at
finding out the attitudes of the Honda riders towards the service and efficiency of Honda bikes.
Total 500 people were sampled to do a survey on their attitudes. They were asked five different
closed ended questions orally, and the responses were analysed using different statistical tools
such as percentage, mean, etc. The survey has an effective conclusion that the consumers have
the positive attitudes about the performance of the bikes, but they are not much satisfied from
the after sale service.

Chapter 1 of the report includes Introduction with background, objectives, statement of


problems and delimitations. Chapter 2 deals with methodology of survey, and the third chapter
presents the analysis and interpretation of the data. The fourth chapter states the conclusion
and recommendations of the research report.

© The Institute of Chartered Accountants of Nepal 10


Paper 7 : Income Tax and VAT

Revision Questions
Income Tax

1. Sunshine Hospitality Pvt. Ltd. is providing the Tourist Class Hotel service from the Hotel
named ‘SunRise Resorts’. During details of the transactions of the company for the income
year is given below. Calculate the tax liability of the company for Income Year (2075.76
with latest provisions of Income Act as amended by Budget for FY 2076.77) based upon
following information:

Particulars Amount (Rs.)


Income
Room Sales 124,500,000.00
Food and Beverage Sales 75,000,000.00
Laundry Sales 2,456,000.00
Forex Gain (netted with Revaluation Loss Rs. 110000) 350,000.00
Gain from sale of old Furniture Sets (book value 450000) 75,000.00
Dividend Income 950,000.00
Sales of Land (book value of Rs. 5090000) 45,000,000.00
Sales of 100% investment in Green Hospitality Pvt. Ltd. (invested Rs.
12500000) 35,000,000.00
Total of Income 283,331,000.00
Cost of Room Sales 32,500,000.00
Cost of Food and Beverage Sales 46,500,000.00
Cost of Laundry Sales 1,245,000.00
Electricity 2,596,000.00
Employee Cost 85,060,000.00
Selling & Other Administrative Expenses 19,750,000.00
Interest Cost 4,560,000.00
Account Receivable not collectible 1,245,600.00
Repair of Elevator 145,000.00
Repair of Building 690,000.00
Repair of Kitchen Boiler and Equipments 456,000.00
Total Expenses 194,747,600.00

Additional Information:

a) The Assets details is as follows

© The Institute of Chartered Accountants of Nepal 1


Paper 7 : Income Tax and VAT

Asset Opening WDV of Asset


Building 12,050,000.00
Elevator, Generator and Kitchen Equipments 24,550,000.00
Bed Furniture and Sets 33,550,000.00
b) The land was purchased 12 years ago, and used as staff quarter till sale. The capital gain
tax of Rs. 598,650 is collected by Land Management Office (Malpot Office) on transfer.
The book value of Staff Quarter was negligible.
c) The company has policy of providing the employee bonus as per Bonus Act 2031.
d) The investment in shares of Green Hospitality is fully disposed off, and capital gain tax
on sale of shares of Rs. 2,250,000 is collected by Company Registrars Office on Share
transfer.
e) Employee cost includes the provision for Gratuity Rs. 22,50,000 not deposited to Fund
(Approved Retirement Fund, for the gratuity cost before implementation of Labor Act
2074).
f) Selling and administrative expenses, includes the donation given to Jumla Institute of
Science & Technology Rs. 25,00,000 with approval from IRD, Rs. 205,000 given to Poor
Welfare Samiti (tax exempt organization) and Rs. 65,000 to Yiti Yoth Club.
g) Salary and wage booked and paid to employees not having PAN is Rs. 375,000, out of
which Rs. 185,000 is wage paid Rs. 900 and 800 each day per labor for miscellaneous
works. The company has employed 165 employees out of which 27 are Indian and 4 are
Pakistani, whole the year.

2. Shivam Plastic Industries has provided the following information for fiscal year 2076.77.
Calculate the tax for the income year.

Particulars Amount (Rs.)


Income
Domestic Sales 187,500,000.00
Export Sales 125,000,000.00
Sales of scraps 1,010,000.00
Total of Income 313,510,000.00

© The Institute of Chartered Accountants of Nepal 2


Paper 7 : Income Tax and VAT

Cost of Materials consumed 177,500,000.00


Wages 48,060,000.00
Heat and Power 18,025,000.00
Packing Materials 2,895,000.00
Employee Cost 14,560,000.00
Sales, Marketing 9,085,000.00
Finance Cost 4,560,000.00
Total of Expenses 274,685,000.00
Factory Building Structure Opening WDV 14,560,000.00
Plant and Machinery Opening WDV 9,085,000.00
Office Equipments Opening WDV 2,275,000.00
Carrying Vehicles Opening WDV 1,259,000.00
Vehicle Opening WDV 3,158,000.00
Addition to Building on Mangshir 2,250,000.00
Addition to Plant on Magh 15 1,385,000.00
Addition to Vehicle on Baisakh 11 1,600,000.00
Particulars Amount (Rs.)
Sales 125,090,000.00
Total 125,090,000.00
Raw Garment Consumed 43,075,000.00
Other Raw materials consumed 12,056,000.00
Factory Rent 1,250,000.00
Wage and Salary 9,860,000.00
Electricity 450,000.00
Insurance of Factory 145,000.00
Insurance of Employees 245,000.00
Carrying and Transportation 2,546,000.00
Total 69,627,000.00

© The Institute of Chartered Accountants of Nepal 3


Paper 7 : Income Tax and VAT

3. Himalyan Breweries Pvt. Ltd. produces “Himalyan Vodka” which is very famous in Nepal
and after launch in Europe and Africa, has gained momentum in the business. For the given
fiscal year it sold 65 % in Nepal and 35% exported. Calculate the tax liability of the
company for the fiscal year based on the information below. The only tax paid by the
company was Rs. 5400000 on Poush end, the final assessment is made on Magh of next
year and tax is also pain on same day, calculate the fine, penalty for this case as per Income
Tax Act 2058.

Particulars Amount (Rs.)


Sales 359,000,000.00
Scrap Sales (within Nepal) 1,518,000.00
Total 360,518,000.00
Raw materials consumed 128,900,000.00
Factory rent 14,500,000.00
Brew Master consultancy expenses 9,085,000.00
Wages 7,500,000.00
Power, water and heat 5,546,000.00
Penalty by Nepal Electricity Authority for Demand charge for 3 phase line 1,275,000.00
Insurance of factory, stock 456,000.00
Administrative Expenses 24,590,000.00
Sale and Business Promotion Expenses 45,600,000.00
Depreciation (as per Income Tax Act) 18,560,000.00
Repair (as per Income Tax Act) 5,260,000.00
Fine and Penalty for Assessment of Excise and VAT by IRD 10,250,000.00
Total 271,522,000.00

4. World Education Foundation Pvt. Ltd. has been operating the school upto class 12 in
Bhaktapur, Nepal. The school is providing the accommodation and fooding facilities to its
students and also the transportation facilities. For the given fiscal year calculate the tax
liability of the company as per Income Tax Act 2058.

Particulars Amount (Rs.)


Tuition Fee 75,600,000.00
Admission Fee of School Students (upto Class 10) 22,500,000.00
Admission Fee of College Students (Class 11 and 12) 15,000,000.00
Transportation Fee 14,700,000.00
Accommodation Fee 16,500,000.00

© The Institute of Chartered Accountants of Nepal 4


Paper 7 : Income Tax and VAT

Medical Fee 750,000.00


Lab Fee 990,000.00
Smart Education Fee 1,012,000.00
Total 147,052,000.00
Leasehold Rent 22,000,000.00
Academic Staff Salary 45,000,000.00
Administrative cost 37,500,000.00
Bank Interest 12,500,000.00
Research & Development Cost 24,560,000.00
Not Collectible Fee Receivable 1,590,000.00
Total 143,150,000.00

Additional information
a) The tuition fee is billed upfront for 3 months. The session for class 1 to 10 starts from
Baisakh and session for class 11 and 12 starts from Ashad. For Amount given above in
tuition fee is cash collected, the account receivable (cash not collected for the bills) is
Rs. 7590000 and Rs. 2350000 is the amount paid by parents in advance before the bill
is sent. The tuition fee for class 1 to 10 is 75% and class 11 and 12 is 25%, the ratio is
same for account receivable and advance fee received.
b) The admission fee is collected on admission to new students only on Baisakh for class 1
to 10 and Ashad for class 11 and 12. Once collected, 10% of admission fee is kept as
security deposit, and returned to the parents/students, if they left the school before the
end of session. In the given year, 10 students upto class 10 and 7 students of class 11
left the school. The total number of students in the school is 1200 and newly admitted
students in the year is 220. The ratio of number of students is also 75% in class 1 to 10
and 25% in class 11 and 12.
c) In the previous year, the research and development cost was Rs. 7500000, and allowed
for deduction as per section 18 was 5000000. The company has opening WDV of Rs.
9200000 for School Furniture, Rs. 10200000 for Research Lab, Rs. 8600000 for
transportation vehicles and Rs. 3500000 for the office furniture and computer/printer.

5. As per the financial statement of Panas Pharma Pvt. Ltd. the following information is
extracted.

© The Institute of Chartered Accountants of Nepal 5


Paper 7 : Income Tax and VAT

Taxable Income Before Pollution Control Cost and Research &


Development Cost 22,054,000.00
Pollution Control Cost for the year 4,596,000.00
Research and Development Cost for the year 6,185,000.00
Cash Incentives paid to Marketing Officers for their target, not
included above 2,574,000.00
Incentives in Kind in the form of refrigerators (10 numbers of Rs.
22500 each) provided to Marketing Officers for their target, not
included above 225,000.00
Last year provision for bonus made Rs. 3789000, distributed to
employees Rs. 2948700 only

Calculate the tax liability of the company, which employs 125 Nepalese for the whole year.
The company is producing and selling different kinds of medicines.

6. Benefit Multipurpose Cooperative Ltd located at Kathmandu metropolis -12, is providing


the saving and deposit service and also providing household consumables trading facility to
its members. Calculate the tax liability for the cooperative for fiscal year based on the
financial information given.

Particulars Amount (Rs.)


Interest income 99,085,000.00
Interest Expenses 67,580,900.00
Net Interest income 31,504,100.00
Sales of household consumables 37,589,000.00
Cost of household consumables 26,789,400.00
Profit on trading activity 10,799,600.00
Add
Service Fee 9,045,600.00
commission and charges 2,245,600.00
Less
Administrative Expenses 9,245,860.00
Provision for Loan Loss made for the year 3,278,900.00
Net Income 41,070,140.00
Gross Loan on Year end 358,600,000.00
Loan loss provision upto last year 16,560,000.00

© The Institute of Chartered Accountants of Nepal 6


Paper 7 : Income Tax and VAT

7. The financial statement of DJ Recreation Club Pvt. Ltd. shows total sales of Rs. 940,00,000
before VAT for the fiscal year, but the tax return of the company has shown less in the
business income (turnover) for the same year. While applying for tax clearance, tax officer
has told you to clarify the same to tax officer. As you inquired with the finance officer,
finance officer has told that the amount is including service charge but before VAT. Show
your clarification workings to the tax officer.

8. As per given information, calculate the tax applicable to Mrs. Shanti Amatya for the
income year. She has contributed to Social Security Fund (SSF) as per requirement of SSF
Act.

Basic Salary is Rs. 405,000 per month, the employer has provided the required 20%
contribution to SSF and remaining 11% is contributed by Mrs. Shanti. Festival and medical
allowance of one month each is provided by the employer. Further she has been using the
office provided vehicle and accommodation facilities. Her Foreign trip allowance for the
year was USD 5,100 (1 USD = 113 Nrs) for which she have to submit the bills of travel to
company. The employer has made arrangement of group personal health insurance with
total cost of Rs. 230,000 for the employees, further 45% part of cost of life policy of Rs.
275,000 is paid by the company, and 55% need to be paid by her. The education merit
facility by employer has covered the monthly school fee of her daughter Rs. 22,500. Her
husband is employed in other organization, so she opted as single for tax calculation. As
part of social responsibility, she had donated to Hospic Nepal (which is tax exempt) Rs.
175,000 during the year.

9. Mrs. Niranjana Chalise has worked with RR Nepal as Social Improvement consultant till
Poush of year with following facilities
Basic Salary of Rs. 275,000 per month, the 10% provident fund contribution by employer
and Mrs. Niranjana. She has contributed Rs 20000 per month to approved retirement fund
from her pocket.
From Magh she joined another employer SR Nepal as Social Improvement consultant with
facility of 450,000 per month. The employer SR Nepal is listed with Social Security Fund

© The Institute of Chartered Accountants of Nepal 7


Paper 7 : Income Tax and VAT

(SSF) and 31% of income need to be deposited with the fund with 20% by employer and 11
by the employee contribution, which is not included in per month 350,000 amount.

The previous employer has deposited the Withholding taxes but the ETDS is not made
(return is not submitted), the current employer has regularly deposited the withholding
taxes and TDS return is submitted regularly. Calculate the net tax liability of Mrs. Chalise
with best option either as couple or single, and discuss the legal implication of not filing the
ETDS return by the previous employer.

10. Shyam Bhandari has following transactions for the given year, calculate the tax applicable
to him as per applicable provisions of Income Tax Act 2058.

He has commercial complex at Anamnagar, with 110 shutters, which are rented out for Rs.
22,500 per month, and tax is paid to concerned ward office. The rent collection and
complex management is done by his staff Mr. Ramesh with monthly salary of Rs. 25,000.
Repair of complex for the year is Rs 324,000.

He has provided loan to Bhairabkunda Hydropower Rs. 450,00,000 for interest of 15% p.a.,
and withholding tax is deposited by the company to concerned tax office.

He has purchased 2 sports vehicles and rented out to ‘Ride Ventures Pvt. Ltd.’ for per
month rental of Rs. 35,000 each, the tax is deposited by the company at concerned tax
office.

Dividend received from listed companies is Rs. 425,000 and interest received from
Debentures of Siddhartha Bank Ltd is Rs. 95,000 during the year.

The opening WDV of sports vehicles is Rs. 240,00,000 and repair of the vehicles is Rs.
275,000 for the year.

He opted as couple for tax calculation.

© The Institute of Chartered Accountants of Nepal 8


Paper 7 : Income Tax and VAT

11. Tamakoshi Hydropower Ltd has contracted M/S Spectro Solutions Norway for its penstock
pipe of tunnel fitting works. Spectro Solutions has deputed its head engineer Mr. Smith
from 24.Sep.2019 to do the necessary works, and the works is handed over to Tamakoshi
on 12.Feb.2020. Tamakoshi has paid Rs. 12,00,00,000 to the contractor and as per the
details the field mobilization expenses of Spector Solutions in Nepal is Rs. 9,00,00,000.
What is the tax applicable for Specto Solutions, calculate with provision of Income Tax Act
regarding Foreign Permanent Establishment.

12. Define the term Company as per Income Tax Act 2058.
13. Discuss the following as per Income Tax Act 2058.
a) Couples
b) Basis of Tax Accounting

14. Write Short Notes on following as per Income Tax Act 2058.
a) Trustee
b) Royalty

15. ABC Ltd has issued the zero coupon ‘Debenture ABC’ on 2075.04.01 with 1 year term,
interest payable on each Nepali month end. The debentures are listed with Nepal Stock
Exchange. XYZ Company Ltd has paid Rs. 99 lacs for the par value of Rs. 100 lacs
debentures on the same day. The Finance Manager of ABC company has claimed that the
company has issued zero coupon bond, and thus no interest cost arrises in the books of
accounts for the related fiscal year. Give your opinion with relevant provisions of Income
Tax Act 2058.

Value Added Tax

16. Capital Construction Pvt. Ltd. has constructed one of fine city building in Thamel of
Kathmandu. The space is used for operation of “Sheer World” branded hotel. It started the
hotel service form 1st of Magh, and remaining area is rented out to various businesses. The
details of the financial information is given below, calculate the Value Added Tax (VAT)

© The Institute of Chartered Accountants of Nepal 9


Paper 7 : Income Tax and VAT

payable/receivable of the company for the year. The given amounts are excluding VAT (if
applicable).

Particulars Amount (Rs.)


Cost of Civil Construction (55 % of cost is VAT attractive raw materials
and remaining is non - VAT attractive raw materials) 217,500,000.00
Financial Cost (interest during construction) 22,500,000.00
Electrification & plumbing (7% of civil cost) (80% is Vat attractive
items, and remaining non -VAT attractive raw materials/ wages) 15,225,000.00
Hotel Equipments 27,500,000.00
Items such as Elevator, Generator (all VAT Attractive) (used for Hotel as
well as the other businesses located in the building) 32,500,000.00
Vehicles (Approved from Department of Tourism) 12,500,000.00
Vehicle for Managing Director (not approved from Department of
Tourism) 5,500,000.00
Office Equipments (used for office of building as well as the Hotel) 27,500,000.00
Cost of Diesel & Gas (Fuel used for Hotel Kitchen) 7,590,000.00
Cost of High Speed Internet (used for building and hotel) 1,245,000.00
Groceries Purchased for Hotel Operation (65% Vat Attractive) 9,580,000.00
Ticket purchased from Airlines 9,085,000.00
Cost of National Park Permit for Hotel Guests 2,075,000.00
Cost paid to VAT not registered contractor for various building works 5,035,000.00
Management Fee paid to Foreign Hotel Management Service provider
company 12,000,000.00
Revenue
Space leased rental income 66,500,000.00
Food, Beverage and Laundry Sales 127,500,000.00
Tour Package Sold to Foreign Trip Advisor Companies (Out Bound tour
package) 22,560,000.00
Tour Package Sold to Nepali Trip Advisor Companies (In Bound tour
package) 17,500,000.00

17. 3D Designers and Printers Pvt. Ltd is using latest cutting edge designing and printing
technology, and thus has attracted most of corporate as its clients. For the latest year, it has
following transactions. Calculate the VAT payable/receivable for the company for the
given period.

Particulars Amount (Rs.)


Designing services to Nepal Government 64,580,000.00

© The Institute of Chartered Accountants of Nepal 10


Paper 7 : Income Tax and VAT

Security printing services to Nepal Rastra Bank 129,860,000.00


Printing services to different corporate houses 119,560,000.00
Design and consultancy of security printing to Government of Maldives 85,060,000.00
Total 399,060,000.00
Purchase cost of printing raw materials 46,050,000.00
Import of Latest Technology equipped Machine from Sweden (This is
equivalent of foreign currency, custom has imposed 21% custom on the cost 145,600,000.00
including the transportation, insurance of 2% of cost)
High Quality chemicals used for printing 12,045,000.00
Alcohol which is mixed with chemical while printing 6,590,000.00
Total 210,285,000.00

18. Shreeram Stores is general grocery store located at Koteshwor, The tax office Koteshwor has
inspected the store on Shrawan 11 2076 and found less physical stock than shown in stock
register. Discuss and show the implications of VAT Act/ Rule.
Short inventory
Sugar Rs. 18,000, Rice Rs. 21,000, Dal Rs. 17,000.
As per the system, the taxpayer VAT registered person and tax is in credit (in the system there is
tax receivable from the Government by the taxpayer)

19. Medicare Pvt. Ltd. has started the commercial operation of its new antibiotic medicine from 1st
of Shrawan 2076. On month of Poush 2076, it has sold the old furnitures, old machineries and
old vechiles at price of 6,715,000. After filing the tax return for the fiscal year, the tax office has
assessed the Value Added Tax on the sales of old fixed assets. But the company has opinion that
the medicines are tax exempt (listed in Annex 1). Give your opinion regarding the applicability
of tax. (No need to calculate the fine/penalty).

20. Discuss the provision of VAT Act/Rule on the following:


a) Proportionate credit
b) Market Value and determination of market value of transaction.
c) Fine for Late payment of VAT amount.

© The Institute of Chartered Accountants of Nepal 11


Paper 7 : Income Tax and VAT

Answers/ Hints:
Income Tax
Q. No. 1

Investment Income
(Non Business
Incomes Business Income Total
Chargeable Assets
Disposal)
Room Sales 124,500,000.00 124,500,000.00
Food and Beverage Sales 75,000,000.00 75,000,000.00
Laundry Sales 2,456,000.00 2,456,000.00
Forex Gain 460,000.00 460,000.00
Gain from sale of old Furniture
- -
Sets (book value 450000)
Dividend Income - -
Sales of Land (book value of
45,000,000.00 45,000,000.00
Rs. 5090000)
Sales of 100% investment in
Green Hospitality Pvt. Ltd. 35,000,000.00 35,000,000.00
(invested Rs. 12500000)
Total of Income 247,416,000.00 35,000,000.00 282,416,000.00
Allowed for Allowed for
Expenses Total
Business Income Investment Income
Cost of Room Sales 32,500,000.00 32,500,000.00
Cost of Food and Beverage
46,500,000.00 46,500,000.00
Sales
Cost of Laundry Sales 1,245,000.00 1,245,000.00
Electricity 2,596,000.00 2,596,000.00
Employee Cost 82,810,000.00 82,810,000.00
Selling & Other Administrative
16,980,000.00 16,980,000.00
Expenses
Interest Cost 4,560,000.00 4,560,000.00
Account Receivable not
- -
collectible
Cost of Land Purchase 5,090,000.00 5,090,000.00
Cost of Investment in unlisted
- 12,500,000.00 12,500,000.00
securities
Repair 1,291,000.00 1,291,000.00
Depreciation 9,238,750.00 9,238,750.00
Employee Bonus 7,213,150.00 7,213,150.00
Total Deduction 210,023,900.00 12,500,000.00 222,523,900.00
Donation (with approval from 1,000,000.00 - 1,000,000.00

© The Institute of Chartered Accountants of Nepal 1


Paper 7 : Income Tax and VAT

IRD)
Donation (to tax exempt
100,000.00 - 100,000.00
organization)
Taxable Income 36,292,100.00 22,500,000.00 58,792,100.00
Tax Rate 25% 10%
Tax 9,073,025.00 2,250,000.00 11,323,025.00
Advance Tax 598,650.00 2,250,000.00 2,848,650.00
Tax Liability 8,474,375.00 - 8,474,375.00

Calculation for Depreciation and Repair


Opening WDV of
Pool Less : Disposal Proceeds
Asset
A 12,050,000.00 -
D 58,100,000.00 525,000.00
Depreciation Base
Dep rate (%) Depreciation
Value
12,050,000.00 5 602,500.00
57,575,000.00 15 8,636,250.00
Total 9,238,750.00
Allowed repair (Minimum of Actual or
Actual Repair 7% of DBV
7%)
690,000.00 843,500.00 690,000.00
601,000.00 4,030,250.00 601,000.00
1,291,000.00 4,873,750.00 1,291,000.00

Profit before tax for employee bonus


Accounting Profit (given in question) 88,583,400.00
Depreciation 9,238,750.00
Profit before tax 79,344,650.00
10/110 of profit before tax 7,213,150.00

a) The sale of land is sale of business assets, so the sales proceeds of the land is added in
business income and cost of land (book value) is cost of land sales, the capital gain tax on
sale of land collected by Malpot office (1.5%) is claimed as advance tax. However, the
net gain on disposal of unlisted securities (share) is disposal of Non Business Chargeable
Assets and the net gain is filed in income tax return separately. The capital gain tax
collected by Company Registrars Office (10%) is claimed as advance tax.

© The Institute of Chartered Accountants of Nepal 2


Paper 7 : Income Tax and VAT

b) The forex gain in netted off with Revaluation forex loss, so the total forex gain is
included in income, but the revaluation loss is not allowed as deduction.
c) The dividend income is final withholding, so not included in income.
d) The gratuity cost before implementation of Labor Act 2074 not deposited into fund
(approved retirement fund) is not allowed for deduction.
e) The donation is not allowed as normal business expense, but allowed as reduction. The
donation with approval of IRD is limited to Rs. 10 lacs and to tax exempted organization
is limited to Rs. 1 lacs.
f) The account receivable not collectible is not allowed for deduction.
g) Repair and Deprecation is calculated separately. The furniture/bed sets are core assets of
Hotel, so need to be treated into Pool D for depreciation calculation.
h) For employee bonus, accounting profit before tax (not the taxable income) is taken as
base (to calculate 10/110).

Q. No. 2
The tax benefit for special industry is on inflation of rate of depreciation by 1/3rd, the rate of tax
for special industry is 20% and rate of tax for export is also 20%, so there is no difference on tax
rates, so no need to separately calculate the tax for domestic sales and export sales.

Particulars Amount (Rs.)


Domestic Sales 187,500,000.00
Export Sales 125,000,000.00
Sales of scraps 1,010,000.00
Total of Income 313,510,000.00
Interest Cost (u/s 14) 4,560,000.00
Cost of Disposal of Trading Stock (u/s 15)
Cost of Materials consumed 177,500,000.00
Wages 48,060,000.00
Heat and Power 18,025,000.00
Packing Materials 2,895,000.00
Repair (u/s 16) -
Depreciation (u/s 19) 5,116,822.22
Others (u/s 13)
Employee Cost 14,560,000.00

© The Institute of Chartered Accountants of Nepal 3


Paper 7 : Income Tax and VAT

Sales, Marketing 9,085,000.00


Total deduction 256,156,822.22
Taxable income 57,353,177.78
Tax rate 20%
Tax Liability 11,470,635.56

Particulars Pool A Pool B Pool C Pool D


Machine and
Office
Assets Factory Shed Vehicle Transport
Equipments
Vehicle
Opening WDV 14,560,000.00 2,275,000.00 3,158,000.00 10,344,000.00
Addition 2,250,000.00 1,600,000.00 1,385,000.00
Capitalized Portion 1.00 0.33 0.67
Allowed Addition 2,250,000.00 - 533,333.33 923,333.33
Depreciation Base
16,810,000.00 2,275,000.00 3,691,333.33 11,267,333.33
Value
Depreciation Rate 6.67 33.33 26.67 20.00
Depreciation 1,120,666.67 758,333.33 984,355.56 2,253,466.67

Q. No. 3

Particulars Export Sales Domestic Sales


Sales 125,650,000.00 233,350,000.00
Scrap Sales 1,518,000.00
Total Income 125,650,000.00 233,350,000.00
% Sales 35.00% 65.00%
Deductible Expenditure
Cost of Goods Sold (Section 15)
Raw materials consumed 45,115,000.00 83,785,000.00
Factory rent 5,075,000.00 9,425,000.00
Brew Master consultancy expenses 3,179,750.00 5,905,250.00
wages 2,625,000.00 4,875,000.00
Power, water and heat 1,941,100.00 3,604,900.00
Penalty by Nepal Electricity Authority for
Demand charge for 3 phase line 446,250.00 828,750.00
Insurance of factory, stock 159,600.00 296,400.00
Repair 1,841,000.00 3,419,000.00
Depreciation (Section 19) 6,496,000.00 12,064,000.00
Other Expenses (section 13 & 21)
Administrative Expenses 8,606,500.00 15,983,500.00

© The Institute of Chartered Accountants of Nepal 4


Paper 7 : Income Tax and VAT

Sale and Business Promotion Expenses 15,960,000.00 29,640,000.00


Fine and Penalty for Assessment of Excise
and VAT by IRD (This is disallowed) - -
Total Deduction 91,445,200.00 169,826,800.00
Taxable Income 34,204,800.00 63,523,200.00
Tax Rate 30% 30%
Facility as per Section (3Nga) (Kha) 6% 0%
Rate after this facility 24% 30%
Facility as per Section (3Nga) (Ga) 6% 0%
Final Applicable Rate 18% 30%
Tax Liability 6,156,864.00 19,056,960.00
Add : Fine, Penalty 2,490,952.51
Total Payable 27,704,776.51

Section 117
Fine for not submitting Estimated Tax Return 2,000.00
Fine for not submitting the Tax Return within Ashoj ((Rs. 100 per
month or 0.1% per annum of turnover for each month of delay,
take maximum) (360518000*.1%*4/12 or 100*4 whichever is
higher) 120,172.67
Total for 117 122,172.67
Section 118
Tax Liability 25,213,824.00
First Installment (40%) 10,085,529.60
Paid 5,400,000.00
Deficit 4,685,529.60
Interest on Deficit for 3 months @ 15% p.a. 175,707.36
Second Installment (70%) 17,649,676.80
Paid 5,400,000.00
Deficit 12,249,676.80
Interest on Deficit for 3 months @ 15% p.a. 459,362.88
Last Installment (100%) 25,213,824.00
Paid 5,400,000.00
Deficit 19,813,824.00
Interest on Deficit for 3 months @ 15% p.a. 743,018.40
Total for 118 1,378,088.64
Section 119
Total to be paid till Ashoj 25,213,824.00
Paid till Ashoj 5,400,000.00
Deficit 19,813,824.00
Remaining is paid on Magh (so delay is for 4 month) (Interest for
4 month @ 15% p.a.) 990,691.20

© The Institute of Chartered Accountants of Nepal 5


Paper 7 : Income Tax and VAT

Total for 119 990,691.20


Total for 117, 118 & 119 2,490,952.51

Q. No. 4
Particulars Amount (Rs.)
Total Tuition fee collected 75,600,000.00
Part of Class 1 to 10 56,700,000.00
Add : account receivable for tuition fee 5,692,500.00
Less : amount received in advance (1,762,500.00)
Total Tuition fee to be booked as income for class 1 to 10 79,530,000.00
Part of Class 11 and 12 18,900,000.00
Out of this the amount is received on Ashad for 3 months
(Ashad, Sharawan and Bhadra, so amount relating to 2 (3,150,000.00)
months is amount received in advance)
Add : account receivable for tuition fee 1,897,500.00
Less : amount received in advance (587,500.00)
Total Tuition fee to be booked as income for class 11 & 12 17,060,000.00
Tuition Fee Income 96,590,000.00
Admission Fee of School Students (upto Class 10) 22,500,000.00
10 students left the school, so the amount refunded is
deducted (amount refunded per student is 10% of (136,363.64)
22500000/(220*75%))

Admission Fee of College Students (Class 11 and 12) 15,000,000.00

7 students left the school, so the amount refunded is


deducted (amount refunded per student is 10% of (190,909.09)
15000000/(220*25%))
Transportation Fee 14,700,000.00
Accommodation Fee 16,500,000.00
Medical Fee 750,000.00
Lab Fee 990,000.00
Smart Education Fee 1,012,000.00
Total 167,714,727.27
Leasehold Rent 22,000,000.00
Academic Staff Salary 45,000,000.00
Administrative Staff cost 37,500,000.00
Bank Interest 12,500,000.00
Not Collectible Fee Receivable (Not Allowed) -
Depreciation 5,450,000.00

© The Institute of Chartered Accountants of Nepal 6


Paper 7 : Income Tax and VAT

Total Deduction 122,450,000.00


Adjusted Taxable Income 45,264,727.27
50% of adjusted taxable income 22,632,363.64
Actual Research & Development Cost 24,560,000.00
Allowed R & D 22,632,363.64
Taxable Income 22,632,363.64
Tax Rate 0.25
Tax 5,658,090.91
Remaining Rs. 1 927,636.36 in R & D is allowed to be carried forward to be added in the
Opening WDV of Pool D of Assets

Depreciation Calculation Pool B Pool D


Research Lab,
Office Furniture Transportation Vehicle,
School Furniture
Opening WDV 3,500,000.00 28,000,000.00
Previous Year unused Research and
2,500,000.00
Development Cost
DBV 3,500,000.00 30,500,000.00
Rate 25.00 15.00
Depreciation 875,000.00 4,575,000.00

Q. No. 5
Particulars Amount
Income before PCC (section 17) and R & D (section 18) 22,054,000.00
Less : Cash incentives to employees 2,574,000.00
Less : Kind incentives to employees 225,000.00
Add : Bonus not distributed to employees (provision for bonus allowed for
deduction for the year, but the same need to be distributed within 8 months of 840,300.00
next fiscal year, the not distributed amount should be included in income)
Adjusted Taxable Income 20,095,300.00
50% of Adjusted Taxable Income 10,047,650.00
Actual Pollution Control Cost 4,596,000.00
Actual Research and Development Cost 6,185,000.00
Allowed PCC (section 17) 4,596,000.00
Allowed R & D (section 18) 6,185,000.00
Taxable Income 9,314,300.00
Tax Rate 25%
Facility for Special Industry (Section 11(2 Kha) (Kha) 5%

© The Institute of Chartered Accountants of Nepal 7


Paper 7 : Income Tax and VAT

Reduced Rate 20%


Facility for Nepali employees more than 100 (Section 11(3) (Ka) 2%
Final Applicable Rate 18%
Tax 1,676,574.00

Q. No. 6
Particulars Cooperative Business Trading Business Total
Interest Income 99,085,000.00 - 99,085,000.00
Sales Revenue 37,589,000.00 37,589,000.00
Service Fee 9,045,600.00 - 9,045,600.00
Commission and charges 2,245,600.00 - 2,245,600.00
Total Income 110,376,200.00 37,589,000.00 147,965,200.00
Ratio of Income 0.75 0.25 1.00
Deductions -
Interest Expenses 67,580,900.00 67,580,900.00
Cost of Goods sold 26,789,400.00 26,789,400.00
Administrative Expenses 6,897,046.69 2,348,813.31 9,245,860.00
Loan Loss provision 1,370,000.00 - 1,370,000.00
Total Deduction 75,847,946.69 29,138,213.31 104,986,160.00
Taxable Income 34,528,253.31 8,450,786.69 42,979,040.00
Tax Rate 0.10 0.25
Tax 3,452,825.33 2,112,696.67 5,565,522.00

Calculation of Allowable LLP


Gross Loan 358,600,000.00
5% of Gross Loan 17,930,000.00
Loan Loss provision claimed upto last year 16,560,000.00
This year allowed Loan Loss Provision (min of this year expenses
1,370,000.00
amount or upto 5% of gross loan)

Q. No. 7
Particulars Amount (Rs.)
Sales before VAT 94,000,000.00
Rate of Service charge on each invoice 10%
So the sales before service charge is 100%, total being 110% 85,454,545.45
Service charge amount 8,545,454.55

Out of total service charge, 71 % is allocated to employees, 24 % is


provided to management and thus need to be included as income, and 2,050,909.09
5% is distributed to Trade Union and Hotel Association of Nepal

© The Institute of Chartered Accountants of Nepal 8


Paper 7 : Income Tax and VAT

Total Turnover as per VAT 94,000,000.00


Total Turnover (Income from Business) of the company for the year
87,505,454.55
(need to be filed as income from business in tax return)

Q. No. 8

Total Amount
Particulars Amount Unit
(Rs.)
Basic Salary 405,000.00 12.00 4,860,000.00
Contribution to Social Security Fund (20% of basic salary
81,000.00 12.00 972,000.00
contributed by employer)
Festival Allowance 405,000.00 1.00 405,000.00
Medical Allowance 405,000.00 1.00 405,000.00
Foreign Trip Allowance (not taxable) - - -
Education Fee of daughter 22,500.00 12.00 270,000.00
Group Health Insurance (not taxable) - - -
Cost of life policy paid by employer (275000*45%) 123,750.00 1.00 123,750.00
Quantification of Accommodation Facility (2% of 4860000) 97,200.00 1.00 97,200.00
Quantification of Vehicle Facility (0.5% of 4860000) 24,300.00 1.00 24,300.00
Assessable income (AI) 7,157,250.00
Reduction
Donation (Min of below three conditions) 75,000.00
5% of AI (5% of 7157250) 357,862.50
Rs. 100,000 100,000.00 100,000.00
Rs. 175,000 actual payment 175,000.00
Life Policy (Minimum of below two conditions) -
Rs. 25,000 25,000.00 25,000.00
Actual paid 275,000.00
Approved Retirement Fund (Minimum of below three
500,000.00
conditions)
Rs. 500,000 (As per amendment of Income Tax Rule 2059,
500,000.00 500,000.00
updated on 2076.11.12 dated Nepal Gazettee)
1/3rd of AI 2,385,750.00
Actual payment (Rs. 405000*31%*12) 1,506,600.00
Taxable income 6,582,250.00
No Social Security Tax for 400,000 -
10% for Next 100,000 10,000.00
20% for Next 200,000 40,000.00
30% for Next 5882250 1,764,675.00
20% Surcharge for more than 20 lacs (4582250*30%*20%) 274,935.00
Total Tax 2,089,610.00

© The Institute of Chartered Accountants of Nepal 9


Paper 7 : Income Tax and VAT

Less : 10% rebate for female 208,961.00


Net Tax 1,880,649.00

Q. No. 9
Amount
Particulars Amount Unit
(Rs.)
Income from RR Nepal (Previous Employer)
Salary 275,000.00 6 1,650,000.00
10% PF contributed by employer 27,500.00 6 165,000.00
Total Income 1,815,000.00
Reduction -
Approved Retirement Fund -
Rs. 300000 300,000.00 1 300,000.00
1/3rd of Assessable Income 605,000.00 1 605,000.00
Actual (27500*6*2+20000*6) 75,000.00 6 450,000.00
Minimum 300,000.00
Taxable Income 1,515,000.00
Tax as Single
First 400000 400,000.00 1% 4,000.00
next 100000 100,000.00 10% 10,000.00
next 200000 200,000.00 20% 40,000.00
remaining 815000 815,000.00 30% 244,500.00
Rebate for female 298,500.00 10% 29,850.00
Final Tax from previous employer (tax as single is
268,650.00
beneficial)
Per month TDS to be deducted, deposited and
44,775.00
return to be filed
Total income calculation for whole the year
salary from RR Nepal 275,000.00 6.00 1,650,000.00
PF from RR Nepal 27,500.00 6.00 165,000.00
Income from SR Nepal 450000 6 2,700,000.00
SSF contribution by SR Nepal 90000 6 540,000.00
Assessable Income 5,055,000.00
Reduction -
Reduction Facility of Approved Retirement Fund
(this limit is for PF and ARF contribution from -
Previous employer)
Rs. 300000 300,000.00 1 300,000.00
1/3rd of Assessable Income 1,685,000.00 1 1,685,000.00
Actual (27500*6*2+20000*6) 75,000.00 6 450,000.00
Minimum 300,000.00

© The Institute of Chartered Accountants of Nepal 10


Paper 7 : Income Tax and VAT

Reduction Facility of Social Security Fund (the


final limit is Rs. 500,000, but from previous
employer is taken Rs. 300,000 so the remaining Rs.
200,000 is only allowed for reduction)
Rs. (500000 - 300000) 200000 1 200,000.00
1/3rd of Assessable Income 1,685,000.00 1 1,685,000.00
Actual (450000*6*31%) 837,000.00 1 837,000.00
Take minimum 200,000.00
Allowed Total Reduction 500,000.00
Taxable Income 4,555,000.00
First 400000 400,000.00 0% -
next 100000 100,000.00 10% 10,000.00
next 200000 200,000.00 20% 40,000.00
remaining 3855000 3,855,000.00 30% 1,156,500.00
Surcharge of 20% on 30% applied 2,555,000.00 6% 153,300.00
Total Tax before rebate 1,359,800.00
Rebate for female 135,980.00
Tax applicable 1,223,820.00
Tax already deposited 268,650.00
Tax to be deducted by SR Nepal 955,170.00
Per month TDS 159,195.00
ETDS not made by RR Nepal (previous employer) for 6 month, which need to be filed
within 25th of next of each Nepali month end.
Section 117 (3), 2.5% p.a. for each month and part of month delay fine is applied.
For example, for the TDS amount of Rs. 44775 for Shrawan, the ETDS return need to be
filed within Bhadra 25
If not filed, and say filed on 17th of Magh, then 44775*2.5%*6/12 = 559.68 is applied
(delay for 6 months) to employee and employer (joint and severally liable).
In totality also, the single option is beneficial.
She has to file the income tax return (being the income more than Rs. 40 lacs and being
employed by more than one employer during the year). The TDS fine is jointly and severally
liable to the employer (previous) and employee.

Q. No. 10
Tax Amount
Particulars Nature of Income Income (Rs.)
(NRs.)
Not to be included on Income from
rent (Rent by natural person is not
Rent 29,700,000.00 -
tax applicable, the tax is payable to
local government as per their

© The Institute of Chartered Accountants of Nepal 11


Paper 7 : Income Tax and VAT

respective legislation, For Income


Tax the income is neither inclusible
nor taxable)
Interest from
Bhairabkunda Inclusive (15% TDS advance tax) 6,750,000.00 1,012,500.00
Hydropower
Rental Income from
Inclusive (10% TDS advance tax) 840,000.00 84,000.00
Sport Vehicles
Final Withholding (tax not allowed
Dividend 447,368.42 22,368.42
for credit)
Interest from Final Withholding (tax not allowed
100,000.00 5,000.00
Debenture for credit)
Total Inclusive Only the non final income are
7,590,000.00 1,096,500.00
Income added
Less: Deduction
Expenses related to non inclusive
Salary of Staff 300,000.00 -
income not allowed (not allowed)
Expenses related to non inclusive
Repair of Building 324,000.00
income not allowed (allowed)
Depreciation of Sport 24000000*20% for pool C
4,800,000.00
Vehicle (allowed)
actual Rs. 275000 or 7% of
Repair of Sport
34000000 (minimum to be taken) 275,000.00
Vehicle
(allowed)
Total Allowed
5,075,000.00
Deduction
Taxable Income 2,515,000.00
First (Social Security
Tax not applicable for 450,000.00 1% -
investment income
Next 100,000.00 10% 10,000.00
Next 200,000.00 20% 40,000.00
Next 1,765,000.00 30% 529,500.00
Surcharge 3,075,000.00 6% 184,500.00
Total 764,000.00
Claim for Advance
1,096,500.00
Tax
Net tax payable this is carried as advance tax (332,500.00)

Q. No 11

As per Section 2 (bb) Permanent Establishment means a place from where a person fully or
partially conducts its business and the term includes place where it conducts business, operates

© The Institute of Chartered Accountants of Nepal 12


Paper 7 : Income Tax and VAT

machineries, or provided technical or professional service. The permanent establishment of the


institution or organization of Entities which is not situated in the country in which it is resident is
called Foreign Permanent Establishment (section 2(x) and section 2(ay)).
The Spectro Solutions has provided the construction service in Nepal for more than 120 days, so
it is FPE for the fiscal year. The FPE is entity and taxed at rate of entity and on repatriation of
profit, the repatriation tax is also levied.
So
Revenue 12,00,00,000
Cost of Revenue 9,00,00,000
Profit 3,00,00,000
Tax Rate 25% = 75,00,000
Net Profit after tax = 2,25,00,000
Repatriation tax = 5% = 1,125,000
Net amount transferred to Head Office of Spectro = 2,13,75,000

Q. No 12
As per section 2(m) of Income Tax act 2058, company means a company established under the
company laws for the time being in force and the following institutions shall also be treated as
company for tax purpose:-
(1) Corporate body established under the laws for the time being in force;
(2) any unincorporated association, committee, institution, society, or group of persons other
than a partnership or a proprietorship firm (whether or not registered) or a trust;
(3) a partnership firm (whether or not registered under the laws for the time being in force) that
has 20 or more partners, a retirement fund, a co-operative, a unit trust, or a joint venture;
(4) Foreign company; and (5) any foreign institution prescribed by the Director-General.

Q. No 13

a) As per section 50,


i) A resident natural person and a resident spouse of the person may, by notice in
writing elect to be treated as a single individual for a particular income year.
ii) Each spouse of a couple making an election as above with respect to an income-year
is jointly and severally liable with the other spouse for any tax payable by the couple
for the year.

© The Institute of Chartered Accountants of Nepal 13


Paper 7 : Income Tax and VAT

iii) What so ever mentioned in above (i) and (ii) resident widow or widower responsible
to take care of dependents shall be treated as couple.

b) Basis of Tax Accounting


As similar in accounting, basis of accounting is also cash basis of accounting and
accrual basis of accounting. Both cash basis and accrual basis are not conceptually
same for both tax and accountings.
Statutory cash basis for tax: Income from employment and income from investments
in case of a natural person has to be accounted in cash basis of accounting.
Statutory accrual basis for tax: Company should keep its tax accounts on accrual
basis of accounting.
Banking business, as licensed from Nepal Rastra Bank can keep its accounts based on
directives. Again, co-peratives can maintain its interest income in cash basis.
Optional basis for tax: Income from business of a natural person and income of entity
other than a company (partnership and trust) may opt either basis for taxation.

Q. No 14

a) As per sec 2(u) of Income tax Act, 2058, A "Trustee" means an individual or Goothi or
corporate body holding assets in a fiduciary capacity, whether held alone or jointly with
other individuals or corporate bodies, and includes the following persons-

(i) any executor or administrator of a deceased individual's estate;


(ii) any liquidator, receiver, or trustee;
(iii) any person having, either in a private or official capacity, the
possession, direction, control, or management of the assets of an
incapacitated person;

(iv) any person who manages assets under a private foundation or other similar
arrangement; and
(v) any person in a similar position to a person mentioned in subparagraphs (i), (ii), (iii)
and (iv).

b)
As per sec 2(ak) of Income tax Act, 2058, "Royalty" means any payment made under a
lease
of an intangible asset and includes any payment made for the following purpose:-

© The Institute of Chartered Accountants of Nepal 14


Paper 7 : Income Tax and VAT

(i) the use of, or the right to use, a copyright, patent, design, model, plan, secret
formula or process, or trademark;
(ii) the supply of know-how;
(iii) the use of, or right to use, a cinematography film, video tape, sound recording,
or any other like medium and the supply of information concerning
industrial, commercial, or scientific experience;
(iv) the supply of assistance ancillary to a matter referred to in paragraphs (i), (ii)
or (iii); or
(v) a total or partial forbearance with respect to a matter referred to in paragraphs
(i), (ii), (iii) or (iv).

Q. No 15
As per section 2(as), "Interest" means the following payment or profit:
(1) Payment under debt liability except the principal,
(2) Profit made from concession, exemption, premium under loan liability, alteration payment or
from similar payment, and
(3) The amounts referred to in Section 32 receivable as an interest out of the payment to be made
by a person who acquires any property under annuities or installment sale or of the payment
made to any person for the use of any property under a financial lease.

As per the arrangement ABC company’s Rs. 100 lacs par value debenture is sold at Rs. 99 lacs,
and on maturity the investor (XYZ Company Ltd) will get Rs. 100 lacs. The 1 lacs received from
the issuer by the investor is interest, i.e. interest cost for ABC Ltd and interest income for XYZ
ltd. Thus the view of finance manager is incorrect, and the amount received on sale of debenture
is source of fund and the extra amount paid to the investor on maturity is interest cost. In the
above example Rs. 1 lacs paid on maturity is interest cost for the ABC Ltd.

Value Added Tax (VAT) Act


Q. No 16

Particulars (Output Tax and


Ratio of taxable and non
taxable sales) Total Sales Taxable Non Taxable Tax
Space leased rental income

© The Institute of Chartered Accountants of Nepal 15


Paper 7 : Income Tax and VAT

66,500,000.00 - 66,500,000.00 -
Food, Beverage and
Laundry Sales 127,500,000.00 127,500,000.00 16,575,000.00
Tour Package Sold to
Foreign Trip Advisor
Companies (Out Bound tour
package) (Sold at 0% Tax
Rate for Export) 22,560,000.00 22,560,000.00 -
Tour Package Sold to
Nepali Trip Advisor
Companies (In Bound tour
package) 17,500,000.00 17,500,000.00 2,275,000.00

Total 234,060,000.00 167,560,000.00 66,500,000.00 18,850,000.00


Ratio 100.00% 71.59% 28.41%

Tax need to pay as


Allowed amount
Particulars Amount Tax Paid Reverse Charge Remarks
for credit
(Section 8Ka)
For non vat
Cost of Civil attractive cost
Construction (55 % of capitalised, the
cost is VAT attractive Reverse charge
raw materials and 217,500,000.00 15,551,250.00 12,723,750.00 11,132,903.74 VAT need to be
remaining is non - assessed and paid.
VAT attractive raw Allowed for credit
materials) in the taxable sales
ratio
No VAT (and
Financial Cost reverse charge
(interest during 22,500,000.00 - 0 0 VAT) applied
construction) interest during
construction
For non vat
Electrification & attractive cost
plumbing (7% of civil capitalised, the
cost) (80% is Vat Reverse charge
attractive items, and 15,225,000.00 1,583,400.00 395,850.00 1,583,400.00 VAT need to be
remaining non -VAT assessed and paid.
attractive raw Allowed for credit
materials/ wages) in the taxable sales
ratio
These are hotel
Hotel Equipments 27,500,000.00 3,575,000.00 0 3,575,000.00 equipments, so all
allowed for credit
Items such as
Elevator, Generator Allowed in the
(all VAT Attractive) 32,500,000.00 4,225,000.00 0 3,024,613.35 ratio of taxable
(used for Hotel as well sales
as the other buinsesses

© The Institute of Chartered Accountants of Nepal 16


Paper 7 : Income Tax and VAT

located in the
building)
Vehicles (Approved
Full amount
from Department of 12,500,000.00 1,625,000.00 0 1,625,000.00
allowed for credit
Tourism)
Only 40% allowed,
further the
Vehicle for Managing
Managing Director
Director (not approved
5,500,000.00 715,000.00 0 204,743.06 allocates the time
from Department of
for building and
Tourism)
hotel, allowed for
taxable sales ratio
Office Equipments
Allowed in the
(used for office of
27,500,000.00 3,575,000.00 0 2,559,288.22 ratio of taxable
building as well as the
sales
Hotel)
Cost of Diesel & Gas
(Fuel used for Hotel 7,590,000.00 986,700.00 0 0 Not allowed
Kitchen)
Cost of High Speed Allowed in the
Internet (used for 1,245,000.00 161,850.00 0 115,865.96 ratio of taxable
building and hotel) sales
65% are Vat
Groceries Purchased
Attractive, only
for Hotel Operation 9,580,000.00 809,510.00 0 809,510.00
that portion
(65% Vat Attractive)
allowed
Ticket purchased from
9,085,000.00 - 0 0 Not taxable
Airlines
Full amount
allowed for credit,
Cost of National Park
because the
Permit for Hotel 2,075,000.00 269,750.00 0 269,750.00
national park
Guests
permit is for hotel
business
For non vat
attractive cost
capitalised, the
Cost paid to VAT not
Reverse charge
registered contractor
5,035,000.00 654,550.00 0 VAT need to be
for various building
assessed and paid.
works
Allowed for credit
in the taxable sales
ratio
For foreign service
Management Fee paid provider, Reverse
to Foreign Hotel Charge VAT need
12,000,000.00 1,560,000.00 1,560,000.00
Management Service to be assessed and
provider company paid, the same is
allowed for credit
Total 407,335,000.00 33,077,460.00 15,334,150.00 26,460,074.32
Tax allowed for credit 26,460,074.32
Tax collected on sales 18,850,000.00

© The Institute of Chartered Accountants of Nepal 17


Paper 7 : Income Tax and VAT

Net Receivable 7,610,074.32


the amount reverse
charge need to be paid
separately

Q. No 17
No
n
Particulars Amount Tax Taxable Tax
abl
e
Designing services to Nepal Government 64,580,000.00 64,580,000.00 8,395,400.00
Security printing services to Nepal Rastra
129,860,000.00 129,860,000.00 16,881,800.00
Bank
Printing services to different corporate
119,560,000.00 119,560,000.00 15,542,800.00
houses
Design and consultancy of security
printing to Government of Maldives 85,060,000.00 85,060,000.00 -
(Taxed at rate of 0% - Export)
Total 399,060,000.00 - 399,060,000.00 40,820,000.00
Purchase cost of printing raw materials 46,050,000.00 46,050,000.00 5,986,500.00
Import of Latest Technology equipped
Machine from Sweden (The taxable
amount at Custom point is cost plus 2% 179,699,520.00 179,699,520.00 23,360,937.60
for transportation, insurance i.e. 102% of
cost, plus 21% custom)
High Quality chemicals used for printing 12,045,000.00 12,045,000.00 1,565,850.00
Alcohol which is mixed with chemical
while printing (the alcohol is direct raw 6,590,000.00 6,590,000.00 342,680.00
material, so the VAT is allowed as credit)
Total 244,384,520.00 - 244,384,520.00 31,255,967.60
Total Output tax 40,820,000.00
Total Input tax 31,255,967.60
Tax to be paid 9,564,032.40

Q. No 18
The sugar is taxable, and rice and dal is not taxable. So, the short inventory of sugar should be
treated as sales and tax as well as the fine/penalty should be collected.
Short taxable item Rs. 18,000

© The Institute of Chartered Accountants of Nepal 18


Paper 7 : Income Tax and VAT

The normal value addition in the taxable item is added as value addition for calculation of
taxable amount. In question, no information is given, so lets take 12% profit margin on sugar. So
the taxable amount is 18,000 *1.1 = 19,800
Tax to be collected = 19,800 * 13% = 2,574
100% penalty as per section 29(2)(Ka) =2574
Since, the latest tax return shown the credit (tax is receivable by the taxpayer from the
government).
Total tax to be collected from the taxpayer 5,148.

Q. No. 19

As per VAT Act and Rule, the person which deals in goods/services listed in Annex 1 (tax
exempted items), need not be registered with Inland Revenue Department for VAT. Since the
company is dealing with medicines which is tax exempted, so no need to register for VAT. But
old furniture, vehicles and machineries on sale are taxable (tax attractive). So the tax office’s
move of assessment on sale of old fixed assets is as per law, and the company need to pay the tax
on sale of furnitures, machineries and vehicles. The taxable amount is the amount recovered on
sale of old fixed assets, 13% on the amount need to be paid. Further, the fine/penalty as per
section 29 is also applicable.

Q. No. 20
a) Proportionate credit

Value Added Tax (VAT) paid on purchase is allowed as full set off if output of registered person
is VAT attractive only. Similarly, VAT paid on purchase is not allowed as credit in full if the
output is VAT exempted. There can be cases, where registered person deals both VAT attractive
and VAT exempt items at a time; in such situation VAT credit is allowed on the VAT paid on
purchase of:
Raw material for VAT attractive output full credit Rule 40(3) Raw material for VAT exempted
output no credit Rule 40(3) Common cost (Raw material or overhead) proportionate credit
Rule 40(4)
As per Rule 40(4), the tax payer dealing in both taxable and tax exempt goods and services shall
apportion the common cost in the ratio of taxable sales value to total sales value. The tax payer
can claim credit for VAT paid on purchase or import proportionally in the ratio of taxable sales
to total sales value.

© The Institute of Chartered Accountants of Nepal 19


Paper 7 : Income Tax and VAT

b) As per section 2(k) of Value Added Tax Act 2052, "Market Value" means the price as
determined pursuant to Section 13;
As per section 13 of Value Added Tax Act 2052, market value related provisions are:
(1) The market value of goods or services shall be determined as the consideration in money
which the supply of these goods or services would generally be agreed on if the transaction
were made under similar circumstances at that date in Nepal taking into consideration the
characteristics, quality, quantity, materials, and any other relevant factor, being a supply
freely offered and made between persons who are unrelated.

(2) For the purpose of this section the method for the determination of market value hall be
as prescribed.
(3) Where the market value of goods or services could not be determined under subsection 1)
and
(2), it shall be determined in accordance with a process determined by the Director General.

In addition to this section, Section 22 of Value Added Rules 2053 mention that, for
determining the market value under Section 13 of the Act, the tax officer shall determine the
market value by studying the transactions and value of other vendors registered in regard to
the transaction of the same nature. In cases where the market value of any goods or services
cannot be determined as set forth in sub-section (3) of Section 13 of the Act, the Director
General shall determine the value on the basis also of the information received in that regard
by him from the registered persons of the same nature.

c) Fine for Late payment of VAT amount


Additional fee: On late payment of VAT amount to Inland Revenue, additional fee of
10% annually shall be imposed on payable VAT amount (Section 19 (2)). Interest On
late payment of VAT amount to Inland Revenue, 15% annual interest shall be imposed in
such outstanding amount. (Section 26).

© The Institute of Chartered Accountants of Nepal 20

You might also like